module 5

Lakukan tugas rumah & ujian kamu dengan baik sekarang menggunakan Quizwiz!

Which information obtained by the nurse interviewing a 30-year-old male patient is most important to communicate to the health care provider? a. The patient has a history of constipation. b. The patient has noticed blood in the stools. c. The patient had an appendectomy at age 27. d. The patient smokes a pack/day of cigarettes.

ANS: B Blood in the stools is a possible clinical manifestation of colorectal cancer and requires further assessment by the health care provider. The other patient information will also be communicated to the health care provider, but does not indicate an urgent need for further testing or intervention

When the nurse is assessing the health perception-health maintenance pattern as related to GI function, an appropriate question to ask is a. "What is your usual bowel elimination pattern?" b. "What percentage of your income is spent on food?" c. "Have you traveled to a foreign country in the last year?" d. "Do you have diarrhea when you are under a lot of stress?"

Correct answer: c Rationale: When assessing gastrointestinal function in relation to the health perception-health management pattern, the nurse should ask the patient about recent foreign travel with possible exposure to hepatitis, parasitic infestation, or bacterial infection.

The patient has peritonitis, which is a major complication of appendicitis. What treatment will the nurse plan to include? a. Peritoneal lavage c. IV fluid replacement b. Peritoneal dialysis d. Increased oral fluid intake

c. IV fluid replacement along with antibiotics, NG suction, analgesics, and surgery would be expected. Peritoneal lavage may be used to determine abdominal trauma. Peritoneal dialysis would not be performed. Oral fluids would be avoided with peritonitis.

The RN directs the LPN/LVN to remove a Foley catheter at 1300. The nurse would check if the patient has voided by: A) 1400. B) 1600 C) 1700. D) 2300.

1700.

The patient with right upper quadrant abdominal pain has an abdominal ultrasound that reveals cholelithiasis. What should the nurse expect to do for this patient? A) Prevent all oral intake. B) Control abdominal pain. C) Provide enteral feedings. D) Avoid dietary cholesterol.

B) Patients with cholelithiasis can have severe pain, so controlling pain is important until the problem can be treated. NPO status may be needed if the patient will have surgery but will not be used for all patients with cholelithiasis. Enteral feedings should not be needed, and avoiding dietary cholesterol is not used to treat cholelithiasis.

An important nursing intervention for a patient with a small intestinal obstruction who has an NG tube is to a. offer ice chips to suck PRN. b. provide mouth care every 1 to 2 hours. c. irrigate the tube with normal saline every 8 hours. d. keep the patient supine with the head of the bed elevated 30 degrees.

b. Mouth care should be done frequently for the patient with a small intestinal obstruction who has an NG tube because of vomiting, fecal taste and odor, and mouth breathing. No ice chips are allowed when a patient is NPO because of a bowel obstruction. The NG tube should be checked for patency and irrigated as ordered. The position of the patient should be one of comfort.

The nurse is reviewing the home medication list for a 44-year-old man admitted with suspected hepatic failure. Which medication could cause hepatotoxicity? a. Nitroglycerin b. Digoxin (Lanoxin) c. Ciprofloxacin (Cipro) d. Acetaminophen (Tylenol)

Correct answer: d Rationale: Many chemicals and drugs are potentially hepatotoxic (see Table 39-6) and result in significant patient harm unless monitored closely. For example, chronic high doses of acetaminophen and nonsteroidal antiinflammatory drugs (NSAIDs) may be hepatotoxic.

A patient complains of nausea. When administering a dose of metoclopramide (Reglan), the nurse should teach the patient to report which potential adverse effect? a) Tremors b) Constipation c) Double vision d) Numbness in fingers and toes

A Extrapyramidal side effects, including tremors and tardive dyskinesias, may occur as a result of metoclopramide (Reglan) administration. Constipation, double vision, and numbness in fingers and toes are not adverse effects of metoclopramide.

A patient is suspected of having a large intestine obstruction. What is the best indication that an obstruction is present? A. Lack of flatus B. Nausea C. Temperature of 100.4° F (38° C) D. Thirst

A Rationale Inability to pass gas or constipation is a common manifestation of a large intestinal obstruction. Reference: 1032

You perform a detailed assessment of the abdomen of a patient with a possible bowel obstruction, knowing that manifestations of an obstruction in the large intestine include (select all that apply) A. a largely distended abdomen. B. diarrhea that is loose or liquid. C. persistent, colicky abdominal pain. D. profuse vomiting that relieves abdominal pain.

A, C Rationale Persistent, colicky abdominal pain occurs with lower intestinal obstruction. Abdominal distention is markedly increased in lower intestinal obstructions. Onset of a large intestine obstruction is gradual, and vomiting is rare. Absolute constipation usually is associated with large intestine obstructions. Reference: 1033

Which assessment information will be most important for the nurse to report to the health care provider about a patient with acute cholecystitis? a. The patient's urine is bright yellow. b. The patient's stools are tan colored. c. The patient has increased pain after eating. d. The patient complains of chronic heartburn.

ANS: B Tan or grey stools indicate biliary obstruction, which requires rapid intervention to resolve. The other data are not unusual for a patient with this diagnosis, although the nurse would also report the other assessment information to the health care provider

A 66-year-old patient has a body mass index (BMI) of 31 kg/m2, a normal C-reactive protein level, and low transferrin and albumin levels. The nurse will plan patient teaching to increase the patient's intake of foods that are high in a. iron. b. protein. c. calories. d. carbohydrate.

ANS: B The patient's C-reactive protein and transferrin levels indicate low protein stores. The BMI is in the obese range, so increasing caloric intake is not indicated. The data do not indicate a need for increased carbohydrate or iron intake.

After bariatric surgery, a patient who is being discharged tells the nurse, "I prefer to be independent. I am not interested in any support groups." Which response by the nurse is best? a. "I hope you change your mind so that I can suggest a group for you." b. "Tell me what types of resources you think you might use after this surgery." c. "Support groups have been found to lead to more successful weight loss after surgery." d. "Because there are many lifestyle changes after surgery, we recommend support groups."

ANS: B This statement allows the nurse to assess the individual patient's potential needs and preferences. The other statements offer the patient more information about the benefits of support groups, but fail to acknowledge the patient's preferences

A woman with irritable bowel syndrome is prescribed alosetron (Lotronex). The patient should be taught to discontinue the drug if which symptom occurs? A. Diarrhea B. Constipation C. Urinary burning D. Renal calculi

B Rationale A potential complication of alosetron is severe constipation with ischemic colitis. The drug should be discontinued if constipation occurs. Diarrhea is the reason the drug is prescribed. The drug does not affect the urinary system. Reference: 1018

You are planning care for a 68-year-old patient with an abdominal mass and suspected bowel obstruction. Which factor in the patient's history increases the patient's risk for colorectal cancer? A. Osteoarthritis B. History of rectal polyps C. History of lactose intolerance D. Use of herbs as dietary supplements

B Rationale A history of rectal polyps places this patient at risk for colorectal cancer. This tissue can degenerate over time and become malignant. The other factors identified do not pose an additional risk to the patient. Reference: 1035

Which food is recommended for a patient 2 weeks after having a colostomy? A. Cabbage B. Popcorn C. Applesauce D. Dried fruit

C Rationale Patients are initially on low-residue and low-fiber diets until the intestine adjusts. The eventual goal is to return to a normal presurgical diet. The patient who had an ileostomy is susceptible to obstruction. Foods such as popcorn and dried fruits must be chewed extremely well so particles are small when swallowed. Patients with ostomies are encouraged to eliminate the cabbage family foods because they cause odor. Reference: 1045

What may occur with failure of the sodium-potassium pump during severe protein depletion? a. Ascites b. Anemia c. Hyperkalemia d. Hypoalbuminemia

Correct answer: c Rationale: The sodium-potassium pump uses 20% to 50% of all calories ingested. When energy sources are decreased, the pump fails to function, sodium and water are left in the cell, and potassium remains in extracellular fluids. Hyperkalemia, as well as hyponatremia, can occur.

What extraintestinal manifestations are seen in both ulcerative colitis and Crohn's disease? a. Celiac disease and gallstones c. Conjunctivitis and colonic dilation b. Peptic ulcer disease and uveitis d. Erythema nodosum and osteoporosis

d. Ulcerative colitis and Crohn's disease have many of the same extraintestinal symptoms, including erythema nodosum and osteoporosis, as well as gallstones, uveitis, and conjunctivitis. Colonic dilation and celiac disease are not extraintestinal.

After administering a dose of promethazine (Phenergan) to a patient with nausea and vomiting, the nurse explains that which of the following may be experienced as a common temporary adverse effect of the medication? A) Drowsiness B) Reduced hearing C) Sensation of falling D) Photosensitivity

A) Drowsiness (Although being given to this patient as an antiemetic, promethazine also has sedative and amnesic properties. For this reason, the patient is likely to experience drowsiness as an adverse effect of the medication.)

The nurse is admitting a 67-year-old patient with new-onset steatorrhea. Which question is most important for the nurse to ask? a. "How much milk do you usually drink?" b. "Have you noticed a recent weight loss?" c. "What time of day do your bowels move?" d. "Do you eat meat or other animal products?"

ANS: B Although all of the questions provide useful information, it is most important to determine if the patient has an imbalance in nutrition because of the steatorrhea

The nurse will plan to teach the patient diagnosed with acute hepatitis B about a. side effects of nucleotide analogs. b. measures for improving the appetite. c. ways to increase activity and exercise. d. administering a-interferon (Intron A).

ANS: B Maintaining adequate nutritional intake is important for regeneration of hepatocytes. Interferon and antivirals may be used for chronic hepatitis B, but they are not prescribed for acute hepatitis B infection. Rest is recommended

Which prescribed intervention for a 61-year-old female patient with chronic short bowel syndrome will the nurse question? a. Ferrous sulfate (Feosol) 325 mg daily b. Senna (Senokot) 1 tablet every day c. Psyllium (Metamucil) 2.1 grams 3 times daily d. Diphenoxylate with atropine (Lomotil) prn loose stools

ANS: B Patients with short bowel syndrome have diarrhea because of decreased nutrient and fluid absorption and would not need stimulant laxatives. Iron supplements are used to prevent iron-deficiency anemia, bulk-forming laxatives help make stools less watery, and opioid antidiarrheal drugs are helpful in slowing intestinal transit time

Which information will the nurse include when teaching a patient with peptic ulcer disease about the effect of ranitidine (Zantac)? a. "Ranitidine absorbs the gastric acid." b. "Ranitidine decreases gastric acid secretion." c. "Ranitidine constricts the blood vessels near the ulcer." d. "Ranitidine covers the ulcer with a protective material."

ANS: B Ranitidine is a histamine-2 (H2) receptor blocker, which decreases the secretion of gastric acid. The response beginning, "Ranitidine constricts the blood vessels" describes the effect of vasopressin. The response "Ranitidine absorbs the gastric acid" describes the effect of antacids. The response beginning "Ranitidine covers the ulcer" describes the action of sucralfate (Carafate).

At his first postoperative checkup appointment after a gastrojejunostomy (Billroth II), a patient reports that dizziness, weakness, and palpitations occur about 20 minutes after each meal. The nurse will teach the patient to a. increase the amount of fluid with meals. b. eat foods that are higher in carbohydrates. c. lie down for about 30 minutes after eating. d. drink sugared fluids or eat candy after meals.

ANS: C The patient is experiencing symptoms of dumping syndrome, which may be reduced by lying down after eating. Increasing fluid intake and choosing high carbohydrate foods will increase the risk for dumping syndrome. Having a sweet drink or hard candy will correct the hypoglycemia that is associated with dumping syndrome but will not prevent dumping syndrome. DIF: Cognitive Level: Apply (application) REF: 949-950 TOP: Nursing Process: Implementation MSC: NCLEX: Physiological Integrity

A 72-year-old male patient with dehydration caused by an exacerbation of ulcerative colitis is receiving 5% dextrose in normal saline at 125 mL/hour. Which assessment finding by the nurse is most important to report to the health care provider? a. Patient has not voided for the last 4 hours. b. Skin is dry with poor turgor on all extremities. c. Crackles are heard halfway up the posterior chest. d. Patient has had 5 loose stools over the last 6 hours.

ANS: C The presence of crackles in an older patient receiving IV fluids at a high rate suggests volume overload and a need to reduce the rate of the IV infusion. The other data will also be reported, but are consistent with the patient's age and diagnosis and do not require a change in the prescribed treatment

The nurse is caring for a 54-year-old female patient on the first postoperative day after a Roux-en-Y gastric bypass procedure. Which assessment finding should be reported immediately to the surgeon? a. Bilateral crackles audible at both lung bases b. Redness, irritation, and skin breakdown in skinfolds c. Emesis of bile-colored fluid past the nasogastric (NG) tube d. Use of patient-controlled analgesia (PCA) several times an hour for pain

ANS: C Vomiting with an NG tube in place indicates that the NG tube needs to be repositioned by the surgeon to avoid putting stress on the gastric sutures. The nurse should implement actions to decrease skin irritation and have the patient cough and deep breathe, but these do not indicate a need for rapid notification of the surgeon. Frequent PCA use after bariatric surgery is expected

Which assessment finding would the nurse need to report most quickly to the health care provider regarding a patient with acute pancreatitis? a. Nausea and vomiting b. Hypotonic bowel sounds c. Abdominal tenderness and guarding d. Muscle twitching and finger numbness

ANS: D Muscle twitching and finger numbness indicate hypocalcemia, which may lead to tetany unless calcium gluconate is administered. Although the other findings should also be reported to the health care provider, they do not indicate complications that require rapid action

Which change in renal or urinary functioning as a result of the normal aging process increases the older client's risk for infection? A. Decreased glomerular filtration B. Decreased filtrate reabsorption C. Weakened sphincter muscles D. Urinary retention

ANS: D Incomplete bladder emptying for whatever reason increases the client's risk for urinary tract infections as a result of urine stasis providing an excellent culture medium that promotes the growth of microorganisms.

A male patient returned from the operating room 6 hours ago with a cast on his right arm. He has not yet voided. Which action would be the most beneficial in assisting the patient to void? A) Suggest he stand at the bedside B) Stay with the patient C) Give him the urinal to use in bed D) Tell him that, if he doesn't urinate, he will be catheterized

A) Suggest he stand at the bedside

Vasopressin (Pitressin) 0.2 units/min infusion is prescribed for a patient with acute arterial gastrointestinal (GI) bleeding. The vasopressin label states vasopressin 20 units/50 mL normal saline. How many mL/min will the nurse infuse? ____________________

ANS: 0.5 There are 0.4 units/1 mL. An infusion of 0.5 mL/min will result in the patient receiving 0.2 mL/min as prescribed.

A 30-year-old man is being admitted to the hospital for elective knee surgery. Which assessment finding is most important to report to the health care provider? a. Tympany on percussion of the abdomen b. Liver edge 3 cm below the costal margin c. Bowel sounds of 20/minute in each quadrant d. Aortic pulsations visible in the epigastric area

ANS: B Normally the lower border of the liver is not palpable below the ribs, so this finding suggests hepatomegaly. The other findings are within normal range for the physical assessment

When a patient's urine dipstick test indicates a small amount of protein, the nurse's next action should be to a. send a urine specimen to the laboratory to test for ketones. b. obtain a clean-catch urine for culture and sensitivity testing. c. inquire about which medications the patient is currently taking. d. ask the patient about any family history of chronic renal failure.

ANS: C Normally the urinalysis will show zero to trace amounts of protein, but some medications may give false-positive readings. The other actions by the nurse may be appropriate, but checking for medications that may affect the dipstick accuracy should be done first

Which information from a patient's urinalysis requires that the nurse notify the health care provider? a. pH 6.2 b. Trace protein c. WBC 20 to 26/hpf d. Specific gravity 1.021

ANS: C The increased number of white blood cells (WBCs) indicates the presence of urinary tract infection or inflammation. The other findings are normal

Two hours after a closed percutaneous kidney biopsy, the client reports a dramatic increase in pain. What is the nurse's best first action? A. Reposition the client on the operative side. B. Administer prescribed opioid analgesic. C. Assess pulse rate and blood pressure. D. Check the Foley catheter for kinks.

ANS: C An increase in the intensity of pain after a percutaneous kidney biopsy is a symptom of internal hemorrhage.

The nurse will anticipate teaching a patient experiencing frequent heartburn about a. a barium swallow. b. radionuclide tests. c. endoscopy procedures. d. proton pump inhibitors.

ANS: D Because diagnostic testing for heartburn that is probably caused by gastroesophageal reflux disease (GERD) is expensive and uncomfortable, proton pump inhibitors are frequently used for a short period as the first step in the diagnosis of GERD. The other tests may be used but are not usually the first step in diagnosis

The patient is incontinent, and a condom catheter is placed. The nurse should take which action? A) Secure the condom with adhesive tape B) Change the condom every 48 hours C) Assess the patient for skin irritation D) Use sterile technique for placement

Assess the patient for skin irritation

A 33-year-old male patient with a gunshot wound to the abdomen undergoes surgery, and a colostomy is formed as shown in the accompanying figure. Which information will be included in patient teaching? a. Stool will be expelled from both stomas. b. This type of colostomy is usually temporary. c. Soft, formed stool can be expected as drainage. d. Irrigations can regulate drainage from the stomas.

B

The results of a patient's recent endoscopy indicate the presence of peptic ulcer disease (PUD). Which teaching point should the nurse provide to the patient based on this new diagnosis? a) "You'll need to drink at least two to three glasses of milk daily." b) "It would likely be beneficial for you to eliminate drinking alcohol." c) "Many people find that a minced or pureed diet eases their symptoms of PUD." d) "Your medications should allow you to maintain your present diet while minimizing symptoms."

B Alcohol increases the amount of stomach acid produced so it should be avoided. Although there is no specific recommended dietary modification for PUD, most patients find it necessary to make some sort of dietary modifications to minimize symptoms. Milk may exacerbate PUD.

The nurse identifies that which patient is at highest risk for developing colon cancer? A. A 28-year-old male who has a body mass index of 27 kg/m2 B. A 32-year-old female with a 12-year history of ulcerative colitis C. A 52-year-old male who has followed a vegetarian diet for 24 years D. A 58-year-old female taking prescribed estrogen replacement therapy

B. A 32-year-old female with a 12-year history of ulcerative colitis Risk for colon cancer includes personal history of inflammatory bowel disease (especially ulcerative colitis for longer than 10 years); obesity (body mass index ≥ 30 kg/m2); family (first-degree relative) or personal history of colorectal cancer, adenomatous polyposis, hereditary nonpolyposis colorectal cancer syndrome; red meat (=7 servings/week); cigarette use; and alcohol (=4 drinks/week).

The nurse and a licensed practical/vocational nurse (LPN/LVN) are working together to care for a patient who had an esophagectomy 2 days ago. Which action by the LPN/LVN requires that the nurse intervene? a. The LPN/LVN uses soft swabs to provide for oral care. b. The LPN/LVN positions the head of the bed in the flat position. c. The LPN/LVN encourages the patient to use pain medications before coughing. d. The LPN/LVN includes the enteral feeding volume when calculating intake and output.

B. The LPN/LVN positions the head of the bed in the flat position

A 71-year-old patient had an abdominal-perineal resection for colon cancer. Which nursing action is most important to include in the plan of care for the day after surgery? a. Teach about a low-residue diet. b. Monitor output from the stoma. c. Assess the perineal drainage and incision. d. Encourage acceptance of the colostomy stoma.

C

A female patient has a sliding hiatal hernia. What nursing interventions will prevent the symptoms of heartburn and dyspepsia that she is experiencing? a) Keep the patient NPO. b) Put the bed in the Trendelenberg position. c) Have the patient eat 4 to 6 smaller meals each day. d) Give various antacids to determine which one works for the patient.

C Eating smaller meals during the day will decrease the gastric pressure and the symptoms of hiatal hernia. Keeping the patient NPO or in a Trendelenberg position are not safe or realistic for a long period of time for any patient. Varying antacids will only be done with the care provider's prescription, so this is not a nursing intervention.

The patient with suspected pancreatic cancer is having many diagnostic studies done. Which one can be used to establish the diagnosis of pancreatic adenocarcinoma and for monitoring the response to treatment? Spiral CT scan A PET/CT scan Incorrect Abdominal ultrasound Cancer-associated antigen 19-9

Cancer-associated antigen 19-9 Correct The cancer-associated antigen 19-9 (CA 19-9) is the tumor marker used for the diagnosis of pancreatic adenocarcinoma and for monitoring the response to treatment. Although a spiral CT scan may be the initial study done and provides information on metastasis and vascular involvement, this test and the PET/CT scan or abdominal ultrasound do not provide additional information.

After administration of a dose of metoclopramide, which patient assessment finding would show the medication was effective? a. Decreased blood pressure b. Absence of muscle tremors c. Relief of nausea and vomiting d. No further episodes of diarrhea

Correct answer: C Metoclopramide is classified as a prokinetic and antiemetic medication. If it is effective, the patient's nausea and vomiting should resolve. Metoclopramide does not affect blood pressure, muscle tremors, or diarrhea.

This bariatric surgical procedure involves creating a stoma and gastric pouch that is reversible, and no malabsorption occurs. What surgical procedure is this? a. Vertical gastric banding b. Biliopancreatic diversion c. Roux-en-Y gastric bypass d. Adjustable gastric banding

Correct answer: d Rationale: With adjustable gastric banding (AGB), the stomach size is limited by an inflatable band placed around the fundus of the stomach. The band is connected to a subcutaneous port and can be inflated or deflated to change the stoma size to meet the patient's needs as weight is lost. The procedure is performed laparoscopically and, if necessary, can be modified or reversed after the initial procedure.

When teaching the patient about the diet for diverticular disease, which foods should the nurse recommend? A. White bread, cheese, and green beans B. Fresh tomatoes, pears, and corn flakes C. Oranges, baked potatoes, and raw carrots D. Dried beans, All Bran (100%) cereal, and raspberries

D A high fiber diet is recommended for diverticular disease. Dried beans, All Bran (100%) cereal, and raspberries all have higher amounts of fiber than white bread, cheese, green beans, fresh tomatoes, pears, corn flakes, oranges, baked potatoes, and raw carrots.

The nurse would question the use of which cathartic agent in a patient with renal insufficiency? A. Bisacodyl (Dulcolax) B. Lubiprostone (Amitiza) C. Cascara sagrada (Senekot) D. Magnesium hydroxide (Milk of Magnesia)

D Milk of Magnesia may cause hypermagnesemia in patients with renal insufficiency. The nurse should question this order with the health care provider. Bisacodyl, lubiprostone, and cascara sagrada are safe to use in patients with renal insufficiency as long as the patient is not currently dehydrated.

The patient with a history of lung cancer and hepatitis C has developed liver failure and is considering liver transplantation. After the comprehensive evaluation, the nurse knows that which factor discovered may be a contraindication for liver transplantation? A) Has completed a college education B) Has been able to stop smoking cigarettes C) Has well-controlled type 1 diabetes mellitus D) The chest x-ray showed another lung cancer lesion.

D) Contraindications for liver transplant include severe extrahepatic disease, advanced hepatocellular carcinoma or other cancer, ongoing drug and/or alcohol abuse, and the inability to comprehend or comply with the rigorous post-transplant course.

A pt reports having dry mouth and asks for some liquid to drink. The nurse reasons that this symptom can most likely be attributed to a common adverse effect of which of the following medications? A) Digoxin (Lanoxin) B) Cefotetan (Cefotan) C) Famotidine (Pepcid) D) Promethazine (Phenergan)

D) Promethazine (Phenergan) A common adverse effect of promethazine, an antihistamine antiemetic agent, is dry mouth; another is blurred vision.

When teaching the patient with acute hepatitis C (HCV), the patient demonstrates understanding when the patient makes which statement? A) "I will use care when kissing my wife to prevent giving it to her." B) "I will need to take adofevir (Hepsera) to prevent chronic HCV." C) "Now that I have had HCV, I will have immunity and not get it again." D) "I will need to be checked for chronic HCV and other liver problems."

D) The majority of patients who acquire HCV usually develop chronic infection, which may lead to cirrhosis or liver cancer. HCV is not transmitted via saliva, but percutaneously and via high-risk sexual activity exposure. The treatment for acute viral hepatitis focuses on resting the body and adequate nutrition for liver regeneration. Adofevir (Hepsera) is taken for severe hepatitis B (HBV) with liver failure. Chronic HCV is treated with pegylated interferon with ribavirin. Immunity with HCV does not occur as it does with HAV and HBV, so the patient may be reinfected with another type of HCV.

The nurse is admitting a 68-year-old man with severe dehydration and frequent watery diarrhea. He just completed a 10-day outpatient course of antibiotic therapy for bacterial pneumonia. It is most important for the nurse to take which action? A. Wear a mask to prevent transmission of infection. B. Wipe equipment with ammonia-based disinfectant. C. Instruct visitors to use the alcohol-based hand sanitizer. D. Don gloves and gown before entering the patient's room.

D. Don gloves and gown before entering the patient's room. Clostridium difficile is an antibiotic-associated diarrhea transmitted by contact, and the spores are extremely difficult to kill. Patients with suspected or confirmed infection with C. difficile should be placed in a private room and gloves and gowns should be worn by visitors and health care providers. Alcohol-based hand cleaners and ammonia-based disinfectants are ineffective and do not kill all of the spores. Equipment cannot be shared with other patients, and a disposable stethoscope and individual patient thermometer are kept in the room. Objects should be disinfected with a 10% solution of household bleach.

The patient is to have an intravenous pyelogram (IVP). Which of the following apply to this procedure? (Select all that apply.) A) Note any allergies. B) Monitor intake and output. C) Provide for perineal hygiene. D) Assess vital signs. E) Encourage fluids after the procedure.

Note any allergies. Encourage fluids after the procedure.

A patient is told to take replacement pancreatic enzymes to prevent malabsorption of fat. How should the enzymes be administered? A. Mixed in fruit juice B. Mixed in chocolate milk C. Enema D. IV infusion

a Rationale Pancreatic enzymes are mixed in fruit juice or applesauce. They should not be mixed with a protein. They are administered by mouth. Reference: 1093-1094

The patient calls the clinic and describes a bump at the site of a previous incision that disappears when he lies down. The nurse suspects that this is which type of hernia (select all that apply)? a. Ventral d. Reducible b. Inguinal e. Incarcerated c. Femoral f. Strangulated

a, d. The ventral or incisional hernia is due to a weakness of the abdominal wall at the site of a previous incision. It is reducible when it returns to the abdominal cavity. Inguinal hernias are at the weak area of the abdominal wall where the spermatic cord in men or the round ligament in women emerges. A femoral hernia is a protrusion through the femoral ring into the femoral canal. Incarcerated hernias do not reduce.

When obtaining a nursing history from the patient with colorectal cancer, the nurse should specifically ask the patient about a. dietary intake. b. sports involvement. c. environmental exposure to carcinogens. d. long-term use of nonsteroidal antiinflammatory drugs (NSAIDs).

a. A diet high in red meat and low fruit and vegetable intake is associated with development of colorectal cancer (CRC), as are alcohol intake and smoking. Family and personal history of CRC also increases the risk. Other environmental agents are not known to be related to colorectal cancer. Long-term use of nonsteroidal antiinflammatory drugs (NSAIDs) is associated with reduced CRC risk.

Two days following a colectomy for an abdominal mass, a patient reports gas pains and abdominal distention. The nurse plans care for the patient based on the knowledge that the symptoms are occurring as a result of a. impaired peristalsis. b. irritation of the bowel. c. nasogastric suctioning. d. inflammation of the incision site.

a. impaired peristalsis. Until peristalsis returns to normal following anesthesia, the patient may experience slowed gastrointestinal motility leading to gas pains and abdominal distention. Irritation of the bowel, nasogastric suctioning, and inflammation of the surgical site do not cause gas pains or abdominal distention.

The nurse determines that the goals of dietary teaching have been met when the patient with celiac disease selects from the menu: a. scrambled eggs and sausage b. buckwheat pancakes with syrup c. oatmeal, skim milk, and orange juice d. yogurt, strawberries, and rye toast with butter

a. scrambled eggs and sausage Rationale: Celiac disease is treated with lifelong avoidance of dietary gluten. Wheat, barley, oats, and rye products must be avoided. Although pure oats do not contain gluten, oat products can become contaminated with wheat, rye, and barley during the milling process. Gluten is also found in some medications and in many food additives, preservatives, and stabilizers.

A physician just told a patient that she has a volvulus. When the patient asks the nurse what this is, what is the best description for the nurse to give her? a. Bowel folding on itself c. Emboli of arterial supply to the bowel b. Twisting of bowel on itself d. Protrusion of bowel in weak or abnormal opening

b. Volvulus is the bowel twisting on itself. The bowel folding on itself is intussusception. Emboli of arterial blood supply to the bowel is vascular obstruction. Protrusion of bowel in a weak or abnormal opening is a hernia

A patient with stage I colorectal cancer is scheduled for surgery. Patient teaching for this patient would include an explanation that: a. chemotherapy will begin after the patient recovers from the surgery b. both chemotherapy and radiation can be used as palliative treatments c. follow-up colonoscopies will be needed to ensure that the cancer does not occur d. a wound, ostomy, and continence nurse will visit the patient to identify an abdominal site for the ostomy

c. follow-up colonoscopies will be needed to ensure that the cancer does not occur Rationale: Stage 1 colorectal cancer is treated with surgical removal of the tumor and reanastomosis, and so there is no ostomy. Chemotherapy is not recommended for stage I tumors. Follow-up colonoscopy is recommended because colorectal cancer can recur.

Following a hemorrhoidectomy, what should the nurse advise the patient to do? a. Use daily laxatives to facilitate bowel emptying. b. Use ice packs to the perineum to prevent swelling. c. Avoid having a bowel movement for several days until healing occurs. d. Take warm sitz baths several times a day to promote comfort and cleaning.

d. Warm sitz baths provide comfort, healing, and cleansing of the area following all anorectal surgery and may be done three or four times a day for 1 to 2 weeks. Stool softeners may be prescribed for several days postoperatively to help keep stools soft for passage but laxatives may cause irritation and trauma to the anorectal area and are not used postoperatively. Early passage of a bowel movement, although painful, is encouraged to prevent drying and hardening of stool, which would result in an even more painful bowel movement.

A 73-year-old patient with diverticulosis has a large bowel obstruction. The nurse will monitor for a. referred back pain. b. metabolic alkalosis. c. projectile vomiting. d. abdominal distention.

ANS: D Abdominal distention is seen in lower intestinal obstruction. Referred back pain is not a common clinical manifestation of intestinal obstruction. Metabolic alkalosis is common in high intestinal obstruction because of the loss of HCl acid from vomiting. Projectile vomiting is associated with higher intestinal obstruction.

A 24-year-old female contracts hepatitis from contaminated food. During the acute (icteric) phase of the patient's illness, the nurse would expect serologic testing to reveal a. antibody to hepatitis D (anti-HDV). b. hepatitis B surface antigen (HBsAg). c. anti-hepatitis A virus immunoglobulin G (anti-HAV IgG). d. anti-hepatitis A virus immunoglobulin M (anti-HAV IgM).

ANS: D Hepatitis A is transmitted through the oral-fecal route, and antibody to HAV IgM appears during the acute phase of hepatitis A. The patient would not have antigen for hepatitis B or antibody for hepatitis D. Anti-HAV IgG would indicate past infection and lifelong immunity.

A 60-year-old African American patient is afraid she might have anal cancer. What assessment finding puts her at high risk for anal cancer? a. Alcohol use c. Human papillomavirus (HPV) b. Only one sexual partner d. Use of a condom with sexual intercourse

c. Human papillomavirus (HPV) is associated with about 80% of anal cancer cases. Other risk factors include multiple sexual partners, smoking, receptive anal sex, and HIV infection, as well as being female, age 60, and African American. The other options are not considered risk factors for anal cancer.

When teaching the patient with acute hepatitis C (HCV), the patient demonstrates understanding when the patient makes which statement? "I will use care when kissing my wife to prevent giving it to her." "I will need to take adofevir (Hepsera) to prevent chronic HCV." "Now that I have had HCV, I will have immunity and not get it again." "I will need to be checked for chronic HCV and other liver problems."

"I will need to be checked for chronic HCV and other liver problems." The majority of patients who acquire HCV usually develop chronic infection, which may lead to cirrhosis or liver cancer. HCV is not transmitted via saliva, but percutaneously and via high-risk sexual activity exposure. The treatment for acute viral hepatitis focuses on resting the body and adequate nutrition for liver regeneration. Adofevir (Hepsera) is taken for severe hepatitis B (HBV) with liver failure. Chronic HCV is treated with pegylated interferon with ribavirin. Immunity with HCV does not occur as it does with HAV and HBV, so the patient may be reinfected with another type of HCV.

All of the following orders are received for a patient who has vomited 1500 mL of bright red blood. Which order will the nurse implement first? a. Insert a nasogastric (NG) tube and connect to suction. b. Administer intravenous (IV) famotidine (Pepcid) 40 mg. c. Draw blood for typing and crossmatching. d. Infuse 1000 mL of lactated Ringer's solution.

ANS: D Because the patient has vomited a large amount of blood, correction of hypovolemia and prevention of hypovolemic shock are the priorities. The other actions also are important to implement quickly but are not the highest priorities.

An older male patient states that he is having problems starting and stopping his stream of urine and he feels the urgency to void. The best way to assist this patient is to: A) Help him stand to void. B) Place a condom catheter. C) Have him practice Credé's method. D) Initiate Kegel exercises.

Initiate Kegel exercises.

The nurse should teach the patient with chronic constipation that which food has the highest dietary fiber? a. Peach c. Dried beans b. Popcorn d. Shredded wheat

c. Of the foods listed, dried beans contain the highest amount of dietary fiber and are an excellent source of soluble fiber. Bran and berries also have large amounts of fiber.

The patient states that she "loses urine" every time she laughs or coughs. The nurse teaches the patient measures to regain urinary control. The nurse recognizes the need for further teaching when the patient states: A) "I will perform my Kegel exercises every day." B) "I joined weight watchers." C) "I drink two glasses of wine with dinner." D) "I have tried urinating every 3 hours."

"I drink two glasses of wine with dinner."

When administered a dose of metoclopramide (Reglan), a patient complains of nausea. The nurse would teach the patient to report which of the following potential adverse effects? A) Tremors B) Constipation C) Double vision D) Numbness in the fingers and dose

A) Tremors (Extrapyramidal side effects, including tremors and dyskinesias, may occur as a result of metoclopramide (Reglan) administration.)

Which menu choice by the patient with diverticulosis is best for preventing diverticulitis? a. Navy bean soup and vegetable salad b. Whole grain pasta with tomato sauce c. Baked potato with low-fat sour cream d. Roast beef sandwich on whole wheat bread

ANS: A A diet high in fiber and low in fats and red meat is recommended to prevent diverticulitis. Although all of the choices have some fiber, the bean soup and salad will be the highest in fiber and the lowest in fat

Select the results (in italics) that are normal in a urinalysis. A. pH 6 B. Specific gravity 1.015 C. Protein small D. Sugar negative E. Nitrate small F. Leukocyte esterase positive G. Bilirubin negative

A, B, D, G Rationale: The abnormal values are indicative of a urinary tract infection. As a result of protein, nitrates, and leukoesterase in the urine, the nurse can expect the laboratory to analyze microscopic sediment including evaluating the sample for the presence of crystals, casts, WBCs, and RBCs.

A 50-year-old female patient calls the clinic to report a new onset of severe diarrhea. The nurse anticipates that the patient will need to a. collect a stool specimen. b. prepare for colonoscopy. c. schedule a barium enema. d. have blood cultures drawn.

ANS: A Acute diarrhea is usually caused by an infectious process, and stool specimens are obtained for culture and examined for parasites or white blood cells. There is no indication that the patient needs a colonoscopy, blood cultures, or a barium enema.

After assisting with a needle biopsy of the liver at a patient's bedside, the nurse should a. put pressure on the biopsy site using a sandbag. b. elevate the head of the bed to facilitate breathing. c. place the patient on the right side with the bed flat. d. check the patient's postbiopsy coagulation studies.

ANS: C After a biopsy, the patient lies on the right side with the bed flat to splint the biopsy site. Coagulation studies are checked before the biopsy. A sandbag does not exert adequate pressure to splint the site

The nurse will be teaching self-management to patients after gastric bypass surgery. Which information will the nurse plan to include? a. Drink fluids between meals but not with meals. b. Choose high-fat foods for at least 30% of intake. c. Developing flabby skin can be prevented by exercise. d. Choose foods high in fiber to promote bowel function.

ANS: A Intake of fluids with meals tends to cause dumping syndrome and diarrhea. Food choices should be low in fat and fiber. Exercise does not prevent the development of flabby skin

Which laboratory test result will the nurse monitor when evaluating the effects of therapy for a 62-year-old female patient who has acute pancreatitis? a. Calcium b. Bilirubin c. Amylase d. Potassium

ANS: C Amylase is elevated in acute pancreatitis. Although changes in the other values may occur, they would not be useful in evaluating whether the prescribed therapies have been effective

A hospitalized patient with possible renal insufficiency after coronary artery bypass surgery is scheduled for a creatinine clearance test. Which equipment will the nurse need to obtain? a. Urinary catheter b. Cleaning towelettes c. Large container for urine d. Sterile urine specimen cup

ANS: C Because creatinine clearance testing involves a 24-hour urine specimen, the nurse should obtain a large container for the urine collection. Catheterization, cleaning of the perineum with antiseptic towelettes, and a sterile specimen cup are not needed for this test

What is important to teach a patient who is learning to manage a colostomy? A. Change the drainage bag when it is one-third full. B. Cut the skin protective wafer about 1 inch larger than the stoma. C. Poke a hole in the collection bag if it is tight and taunt. D. Maintain sterile procedure during the drainage bag change.

A Rationale The weight of drainage from the stoma pulls the wafer away from the skin so ostomy bags should be emptied when one-third full. The wafer is 1/8 to 1/16 larger than the stoma to prevent drainage on the skin that would cause irritation. Holes are not poked in the bag. A distended bag is full of flatus, which should be expelled by opening the bag. The gastrointestinal system is not sterile, and clean protocol is maintained. Reference: 1042

The family member of a patient who has suffered massive abdominal trauma in an automobile accident asks the nurse why the patient is receiving famotidine (Pepcid). The nurse will explain that the medication will a. prevent aspiration of gastric contents. b. inhibit the development of stress ulcers. c. lower the chance for H. pylori infection. d. decrease the risk for nausea and vomiting.

ANS: B Famotidine is administered to prevent the development of physiologic stress ulcers, which are associated with a major physiologic insult such as massive trauma. Famotidine does not decrease nausea or vomiting, prevent aspiration, or prevent H. pylori infection.

The nurse is caring for a woman recently diagnosed with viral hepatitis A. Which individual should the nurse refer for an immunoglobin (IG) injection? A caregiver who lives in the same household with the patient A friend who delivers meals to the patient and family each week A relative with a history of hepatitis A who visits the patient daily A child living in the home who received the hepatitis A vaccine 3 months ago

A caregiver who lives in the same household with the patient IG is recommended for persons who do not have anti-HAV antibodies and are exposed as a result of close contact with persons who have HAV or foodborne exposure. Persons who have received a dose of HAV vaccine more than 1 month previously or who have a history of HAV infection do not require IG.

The patient has an obstruction high in the small intestine. What patient assessment do you anticipate finding? A. No bowel sounds B. Metabolic acidosis C. Flank pain D. Vomiting

D Rationale A patient with a high small intestinal obstruction is likely to have vomiting, which can be profuse. Lower intestinal obstruction is associated with a greater risk of metabolic acidosis. In small intestinal obstructions bowel sounds can still be heard in the large intestine. Flank pain is typically related to renal calculi. Reference: 1032

The nurse is preparing to administer a dose of bisacodyl (Dulcolax). In explaining the medication to the patient, the nurse would explain that it acts in what way? A. Increases bulk in the stool B. Lubricates the intestinal tract to soften feces C. Increases fluid retention in the intestinal tract D. Increases peristalsis by stimulating nerves in the colon wall

D Bisacodyl is a stimulant laxative that aids in producing a bowel movement by irritating the colon wall and stimulating enteric nerves. It is available in oral and suppository forms. Fiber and bulk forming drugs increase bulk in the stool; water and stool softeners soften feces, and saline and osmotic solutions cause fluid retention in the intestinal tract.

A patient with a Foley catheter carries the collection bag at waist level when ambulating. The nurse tells the patient that he or she is at risk for: (Select all that apply.) A) Infection. B) Retention. C) Stagnant urine. D) Reflux of urine.

Infection. Reflux of urine.

The nurse assesses that the patient has a full bladder, and the patient states that he or she is having difficulty voiding. The nurse would teach the patient to: A) Use the double-voiding technique. B) Perform Kegel exercises. C) Use Credé's method. D) Keep a voiding diary.

Use Credé's method.

A female patient reports that she is experiencing burning on urination, frequency, and urgency. The nurse notes that a clean-voided urine specimen is markedly cloudy. The probable cause of these symptoms and findings is: A) Cystitis. B) Hematuria. C) Pyelonephritis. D) Dysuria.

A) Cystitis.

Vasopressin (Pitressin) 0.2 units/min infusion is prescribed for a patient with acute arterial gastrointestinal (GI) bleeding. The vasopressin label states vasopressin 100 units/250 mL normal saline. How many mL/hr will the nurse infuse?

ANS: 30 There are 0.4 units/1 mL. An infusion of 30 mL/hr will result in the patient receiving 0.2 units/min as prescribed.

A nurse is caring for a client who will perform fecal occult blood testing at home. Which of the following should the nurse include when explaining the procedure to the client? A. Eating more protein is recommended prior to testing. B. One stool specimen is sufficient for testing. C. A red color change indicates a positive test. D. The specimen cannot be contaminated with urine.

D. The specimen cannot be contaminated with urine. For fecal occult blood testing at home, the stool specimens cannot be contaminated with water or urine; three specimens from three different bowel movements are required; some proteins such as red meat, fish, and poultry can alter the test results; and a blue color indicates a positive guaiac or presence of fecal occult blood not red. Chapter 45; Page 1046

The health care provider orders lactulose for a patient with hepatic encephalopathy. The nurse will monitor for effectiveness of this medication for this patient by assessing what? Relief of constipation Relief of abdominal pain Decreased liver enzymes Decreased ammonia levels

Decreased ammonia levels Hepatic encephalopathy is a complication of liver disease and is associated with elevated serum ammonia levels. Lactulose traps ammonia in the intestinal tract. Its laxative effect then expels the ammonia from the colon, resulting in decreased serum ammonia levels and correction of hepatic encephalopathy.

The nurse assessing the urinary system of a 45-year-old female would use auscultation to a. determine kidney position. b. identify renal artery bruits. c. check for ureteral peristalsis. d. assess for bladder distention.

ANS: B The presence of a bruit may indicate problems such as renal artery tortuosity or abdominal aortic aneurysm. Auscultation would not be helpful in assessing for the other listed urinary tract information

Which of the following should a patient be taught after a hemorrhoidectomy? A. take mineral oil prior to bedtime B eat a low fiber diet to rest the colon C. administer oil retention enema to empty the colon D. use prescribed pain medication before a bowel movement

D

The medications prescribed for the patient with inflammatory bowel disease include cobalamin and iron injections. What is the rationale for using these drugs? a. Alleviate stress c. Correct malnutrition b. Combat infection d. Improve quality of life

c. Cobalamin and iron injections will help to correct malnutrition. Correcting malnutrition will also indirectly help to improve quality of life and fight infections.

The family of a patient newly diagnosed with hepatitis A asks the nurse what they can do to prevent becoming ill themselves. Which response by the nurse is most appropriate? "The hepatitis vaccine will provide immunity from this exposure and future exposures." "I am afraid there is nothing you can do since the patient was infectious before admission." "You will need to be tested first to make sure you don't have the virus before we can treat you." "An injection of immunoglobulin will need to be given to prevent or minimize the effects from this exposure."

"An injection of immunoglobulin will need to be given to prevent or minimize the effects from this exposure." Immunoglobulin provides temporary (1-2 months) passive immunity and is effective for preventing hepatitis A if given within 2 weeks after exposure. It may not prevent infection in all persons, but it will at least modify the illness to a subclinical infection. The hepatitis vaccine is only used for preexposure prophylaxis.

What is a classic diagnostic finding in a patient with appendicitis? A. Elevated white blood cell (WBC) count B. Elevated level of lipase C. Left lower quadrant tenderness D. Positive Kernig's sign

A Rationale The WBC count is mildly to moderately elevated in about 90% of cases. The lipase level is elevated in patients with pancreatitis. Left lower quadrant tenderness is typically seen in diverticulitis. The classic location for appendicitis is McBurney's point in the right lower quadrant. Positive Kernig's sign indicates meningitis. Reference: 1020

The patient has fecal incontinence. You are working to promote bowel training. What is the best time to have the patient attempt to defecate every day? A. Thirty minutes after breakfast B. Before going to bed C. Before noon meal D. After performing exercise

A Rationale The gastrocolic reflex is strongest in most people right after breakfast. A good time for many persons to schedule elimination is within 30 minutes after breakfast. The other times are not as conducive; exercise often makes fecal incontinence worse. Reference: 1012

The nurse is caring for a group of patients. Which patient is at highest risk for pancreatic cancer? A 38-year-old Hispanic female who is obese and has hyperinsulinemia A 23-year-old who has cystic fibrosis-related pancreatic enzyme insufficiency A 72-year-old African American male who has smoked cigarettes for 50 years A 19-year-old who has a 5-year history of uncontrolled type 1 diabetes mellitus

A 72-year-old African American male who has smoked cigarettes for 50 years Risk factors for pancreatic cancer include chronic pancreatitis, diabetes mellitus, age, cigarette smoking, family history of pancreatic cancer, high-fat diet, and exposure to chemicals such as benzidine. African Americans have a higher incidence of pancreatic cancer than whites. The most firmly established environmental risk factor is cigarette smoking. Smokers are two or three times more likely to develop pancreatic cancer as compared with nonsmokers. The risk is related to duration and number of cigarettes smoked.

When providing discharge teaching for the patient after a laparoscopic cholecystectomy, what information should the nurse include? A lower-fat diet may be better tolerated for several weeks. Correct Do not return to work or normal activities for 3 weeks. Bile-colored drainage will probably drain from the incision. Keep the bandages on and the puncture site dry until it heals.

A lower-fat diet may be better tolerated for several weeks. Correct Although the usual diet can be resumed, a low-fat diet is usually better tolerated for several weeks following surgery. Normal activities can be gradually resumed as the patient tolerates. Bile-colored drainage or pus, redness, swelling, severe pain, and fever may all indicate infection. The bandage may be removed the day after surgery, and the patient can shower.

Elimination changes that result from inability of the bladder to empty properly may cause which of the following? (Select all that apply.) A) Incontinence B) Frequency C) Urgency D) Urinary retention E) Urinary tract infection

A) Incontinence B) Frequency C) Urgency D) Urinary retention E) Urinary tract infection ---- Any condition resulting in urinary retention increases the risk for urinary tract infection. As retention progresses, retention with overflow develops. Pressure in the bladder builds to a point at which the external urethral sphincter is unable to hold back urine. With retention the patient may void small amounts of urine 2 to 3 times an hour and have urgency. He or she may continually dribble urine. Urinary retention results from inability of the bladder to empty.

A pt with a history of peptic ulcer disease has presented to the emergency department with complaints of severe abdominal pain and a rigid, boardlike abdomen, prompting the health care team to suspect a perforated ulcer. Which of the following actions should the nurse anticipate? A) Providing IV fluids and inserting a nasogastric tube B) Administering oral bicarbonate and testing the patient's gastric pH level C) Performing a fecal occult blood test and administering IV calcium gluconate D) Starting parenteral nutrition and placing the patient in a high-Fowler's position

A) Providing IV fluids and inserting a nasogastric tube A perforated peptic ulcer requires IV replacement of fluid losses and continued gastric aspiration by NG tube. Nothing is given by mouth and gastric pH testing is not a priority. Calcium gluconate is not a medication directly relevant to the patient's suspected diagnosis and parenteral nutrition is not a priority in the short term.

When caring for a patient with a biliary obstruction, the nurse will anticipate administering which of the following vitamin supplements (select all that apply)? A. Vitamin A B. Vitamin D C. Vitamin E D. Vitamin K E. Vitamin B

A,B,C,D) Biliary obstruction prevents bile from entering the small intestine and thus prevents the absorption of fat-soluble vitamins. Vitamins A, D, E, and K are all fat-soluble and thus would need to be supplemented in a patient with biliary obstruction.

A nurse is caring for a client for whom a tap water enema is prescribed, to be repeated until the return is clear. Which of the following actions should the nurse take? A. Clarify the order with the provider. B. Explain the procedure to the client. C. Ensure that the tap water is not too hot. D. Keep the amount per enema to less than 1,000 mL.

A. Clarify the order with the provider. Tap water is a hypotonic solution that can cause water toxicity. It should not be repeated. The nurse should clarify the order with the provider. Explaining the procedure to the client, ensuring that the tap water is not too hot, and keeping the amount to less than 1,000 mL are not pertinent if the enema should not be repeated. If you got this question wrong, I am judging you...just kidding :)

A 58-year-old woman has just returned to the nursing unit after an esophagogastroduodenoscopy (EGD). Which action by unlicensed assistive personnel (UAP) requires that the registered nurse (RN) intervene? a. Offering the patient a drink of water b. Positioning the patient on the right side c. Checking the vital signs every 30 minutes d. Swabbing the patient's mouth with cold water

ANS: A Immediately after EGD, the patient will have a decreased gag reflex and is at risk for aspiration. Assessment for return of the gag reflex should be done by the RN. The other actions by the UAP are appropriate

While caring for a comatose patient who is receiving continuous enteral nutrition through a soft nasogastric tube, the nurse notes the presence of new crackles in the patient's lungs. In which order will the nurse take the following actions? Put a comma and space between each answer choice (a, b, c, d, etc.) ____________________ a. Turn off the tube feeding. b. Obtain the patient's oxygen saturation. c. Check the tube feeding residual volume. d. Notify the patient's health care provider.

ANS: A, B, C, D The assessment data indicate that aspiration may have occurred, and the nurse's first action should be to turn off the tube feeding to avoid further aspiration. The next action should be to check the oxygen saturation because this may indicate the need for immediate respiratory suctioning or oxygen administration. The residual volume should be obtained because it provides data about possible causes of aspiration. Finally, the health care provider should be notified and informed of all the assessment data the nurse has just obtained.

In which order will the nurse take the following actions when caring for a patient who develops watery diarrhea and a fever after prolonged omeprazole (Prilosec) therapy? (Put a comma and a space between each answer choice [A, B, C, D].) a. Contact the health care provider. b. Assess blood pressure and heart rate. c. Give the PRN acetaminophen (Tylenol). d. Place the patient on contact precautions.

ANS: D, B, A, C Proton pump inhibitors including omeprazole (Prilosec) may increase the risk of Clostridium difficile-associated colitis. Because the patient's history and symptoms are consistent with C. difficile infection, the initial action should be initiation of infection control measures to protect other patients. Assessment of blood pressure and pulse is needed to determine whether the patient has symptoms of hypovolemia and/or shock. The health care provider should be notified so that actions such as obtaining stool specimens and antibiotic therapy can be started. Tylenol may be administered, but is the lowest priority of the actions.

When teaching a patient about testing to diagnose metabolic syndrome, which topic would the nurse include? a. Blood glucose test b. Cardiac enzyme tests c. Postural blood pressures d. Resting electrocardiogram

ANS: A A fasting blood glucose test >100 mg/dL is one of the diagnostic criteria for metabolic syndrome. The other tests are not used to diagnose metabolic syndrome although they may be used to check for cardiovascular complications of the disorder

A 34-year-old female patient with a new ileostomy asks how much drainage to expect. The nurse explains that after the bowel adjusts to the ileostomy, the usual drainage will be about _____ cups. a. 2 b. 3 c. 4 d. 5

ANS: A After the proximal small bowel adapts to reabsorb more fluid, the average amount of ileostomy drainage is about 500 mL daily. One cup is about 240 mL.

A patient complains of gas pains and abdominal distention two days after a small bowel resection. Which nursing action is best to take? a. Encourage the patient to ambulate. b. Instill a mineral oil retention enema. c. Administer the ordered IV morphine sulfate. d. Offer the ordered promethazine (Phenergan) suppository.

ANS: A Ambulation will improve peristalsis and help the patient eliminate flatus and reduce gas pain. A mineral oil retention enema is helpful for constipation with hard stool. A return-flow enema might be used to relieve persistent gas pains. Morphine will further reduce peristalsis. Promethazine (Phenergan) is used as an antiemetic rather than to decrease gas pains or distention

Which nursing action will be included in the plan of care for a 27-year-old male patient with bowel irregularity and a new diagnosis of irritable bowel syndrome (IBS)? a. Encourage the patient to express concerns and ask questions about IBS. b. Suggest that the patient increase the intake of milk and other dairy products. c. Educate the patient about the use of alosetron (Lotronex) to reduce symptoms. d. Teach the patient to avoid using nonsteroidal antiinflammatory drugs (NSAIDs).

ANS: A Because psychologic and emotional factors can affect the symptoms for IBS, encouraging the patient to discuss emotions and ask questions is an important intervention. Alosetron has serious side effects, and is used only for female patients who have not responded to other therapies. Although yogurt may be beneficial, milk is avoided because lactose intolerance can contribute to symptoms in some patients. NSAIDs can be used by patients with IBS.

The health care provider prescribes the following therapies for a patient who has been admitted with dehydration and hypotension after 3 days of nausea and vomiting. Which order will the nurse implement first? a. Infuse normal saline at 250 mL/hr. b. Administer IV ondansetron (Zofran). c. Provide oral care with moistened swabs. d. Insert a 16-gauge nasogastric (NG) tube.

ANS: A Because the patient has severe dehydration, rehydration with IV fluids is the priority. The other orders should be accomplished as quickly as possible after the IV fluids are initiated.

When assessing a patient who is a vegan, which finding may indicate the need for cobalamin supplementation? a. Paresthesias b. Ecchymoses c. Dry, scaly skin d. Gingival swelling

ANS: A Cobalamin (vitamin B12) cannot be obtained from foods of plant origin, so the patient will be most at risk for signs of cobalamin deficiency, such as anemia and peripheral neuropathy. The other symptoms listed are associated with other nutritional deficiencies but would not be associated with a vegan diet.

Which patient statement indicates that the nurse's teaching following a gastroduodenostomy has been effective? a. "Vitamin supplements may prevent anemia." b. "Persistent heartburn is common after surgery." c. "I will try to drink more liquids with my meals." d. "I will need to choose high carbohydrate foods."

ANS: A Cobalamin deficiency may occur after partial gastrectomy, and the patient may need to receive cobalamin via injections or nasal spray. Although peptic ulcer disease may recur, persistent heartburn is not expected after surgery and the patient should call the health care provider if this occurs. Ingestion of liquids with meals is avoided to prevent dumping syndrome. Foods that have moderate fat and low carbohydrate should be chosen to prevent dumping syndrome

Cobalamin injections have been prescribed for a patient with chronic atrophic gastritis. The nurse determines that teaching regarding the injections has been effective when the patient states, a. "The cobalamin injections will prevent me from becoming anemic." b. "These injections will increase the hydrochloric acid in my stomach." c. "These injections will decrease my risk for developing stomach cancer." d. "The cobalamin injections need to be taken until my inflamed stomach heals."

ANS: A Cobalamin supplementation prevents the development of pernicious anemia. The incidence of stomach cancer is higher in patients with chronic gastritis, but cobalamin does not reduce the risk for stomach cancer. Chronic gastritis may cause achlorhydria, but cobalamin does not correct this. The loss of intrinsic factor secretion with chronic gastritis is permanent, and the patient will need lifelong supplementation with cobalamin.

A patient has just arrived on the postoperative unit after having a laparoscopic esophagectomy for treatment of esophageal cancer. Which nursing action should be included in the postoperative plan of care? a. Elevate the head of the bed to at least 30 degrees. b. Reposition the nasogastric (NG) tube if drainage stops or decreases. c. Notify the doctor immediately about bloody NG drainage. d. Start oral fluids when the patient has active bowel sounds.

ANS: A Elevation of the head of the bed decreases the risk for reflux and aspiration of gastric secretions. The NG tube should not be repositioned without consulting with the health care provider. Bloody NG drainage is expected for the first 8 to 12 hours. A swallowing study is needed before oral fluids are started.

The nurse will anticipate preparing a 71-year-old female patient who is vomiting "coffee-ground" emesis for a. endoscopy. b. angiography. c. barium studies. d. gastric analysis.

ANS: A Endoscopy is the primary tool for visualization and diagnosis of upper gastrointestinal (GI) bleeding. Angiography is used only when endoscopy cannot be done because it is more invasive and has more possible complications. Barium studies are helpful in determining the presence of gastric lesions, but not whether the lesions are actively bleeding. Gastric analysis testing may help with determining the cause of gastric irritation, but it is not used for acute GI bleeding. DIF: Cognitive Level: Apply (application) REF: 954 TOP: Nursing Process: Planning MSC: NCLEX: Physiological Integrity

A patient is hospitalized with vomiting of "coffee-ground" emesis. The nurse will anticipate preparing the patient for a. endoscopy. b. angiography. c. gastric analysis testing. d. barium contrast studies.

ANS: A Endoscopy is the primary tool for visualization and diagnosis of upper gastrointestinal (GI) bleeding. Angiography is used only when endoscopy cannot be done because it is more invasive and has more possible complications. Gastric analysis testing may help with determining the cause of gastric irritation, but it is not used for acute GI bleeding. Barium studies are helpful in determining the presence of gastric lesions, but not whether the lesions are actively bleeding.

A 36-year-old male patient in the outpatient clinic is diagnosed with acute hepatitis C (HCV) infection. Which action by the nurse is appropriate? a. Schedule the patient for HCV genotype testing. b. Administer the HCV vaccine and immune globulin. c. Teach the patient about ribavirin (Rebetol) treatment. d. Explain that the infection will resolve over a few months.

ANS: A Genotyping of HCV has an important role in managing treatment and is done before drug therapy is initiated. Because most patients with acute HCV infection convert to the chronic state, the nurse should not teach the patient that the HCV will resolve in a few months. Immune globulin or vaccine is not available for HCV. Ribavirin is used for chronic HCV infection

A patient has elevated blood urea nitrogen (BUN) and serum creatinine levels. Which bowel preparation order would the nurse question for this patient who is scheduled for a renal arteriogram? a. Fleet enema b. Tap-water enema c. Senna/docusate (Senokot-S) d. Bisacodyl (Dulcolax) tablets

ANS: A High-phosphate enemas, such as Fleet enemas, should be avoided in patients with elevated BUN and creatinine because phosphate cannot be excreted by patients with renal failure. The other medications for bowel evacuation are more appropriate.

A 50-year-old patient who underwent a gastroduodenostomy (Billroth I) earlier today complains of increasing abdominal pain. The patient has no bowel sounds and 200 mL of bright red nasogastric (NG) drainage in the last hour. The highest priority action by the nurse is to a. contact the surgeon. b. irrigate the NG tube. c. monitor the NG drainage. d. administer the prescribed morphine.

ANS: A Increased pain and 200 mL of bright red NG drainage 12 hours after surgery indicate possible postoperative hemorrhage, and immediate actions such as blood transfusion and/or return to surgery are needed. Because the NG is draining, there is no indication that irrigation is needed. Continuing to monitor the NG drainage is not an adequate response. The patient may need morphine, but this is not the highest priority action

Twelve hours after undergoing a gastroduodenostomy (Billroth I), a patient complains of increasing abdominal pain. The patient has absent bowel sounds and 200 mL of bright red nasogastric (NG) drainage in the last hour. The most appropriate action by the nurse at this time is to a. notify the surgeon. b. irrigate the NG tube. c. administer the prescribed morphine. d. continue to monitor the NG drainage.

ANS: A Increased pain and 200 mL of bright red NG drainage 12 hours after surgery indicate possible postoperative hemorrhage, and immediate actions such as blood transfusion and/or return to surgery are needed. Because the NG is draining, there is no indication that irrigation is needed. The patient may need morphine, but this is not the highest priority action. Continuing to monitor the NG drainage is not an adequate response.

A patient who has just been started on continuous tube feedings of a full-strength commercial formula at 100 mL/hr using a closed system method has six diarrhea stools the first day. Which action should the nurse plan to take? a. Slow the infusion rate of the tube feeding. b. Check gastric residual volumes more frequently. c. Change the enteral feeding system and formula every 8 hours. d. Discontinue administration of water through the feeding tube.

ANS: A Loose stools indicate poor absorption of nutrients and indicate a need to slow the feeding rate or decrease the concentration of the feeding. Water should be given when patients receive enteral feedings to prevent dehydration. When a closed enteral feeding system is used, the tubing and formula are changed every 24 hours. High residual volumes do not contribute to diarrhea.

A 51-year-old male patient has a new diagnosis of Crohn's disease after having frequent diarrhea and a weight loss of 10 pounds (4.5 kg) over 2 months. The nurse will plan to teach about a. medication use. b. fluid restriction. c. enteral nutrition. d. activity restrictions.

ANS: A Medications are used to induce and maintain remission in patients with inflammatory bowel disease (IBD). Decreased activity level is indicated only if the patient has severe fatigue and weakness. Fluids are needed to prevent dehydration. There is no advantage to enteral feedings

A patient who has had several episodes of bloody diarrhea is admitted to the emergency department. Which action should the nurse anticipate taking? a. Obtain a stool specimen for culture. b. Administer antidiarrheal medications. c. Teach about adverse effects of nonsteroidal anti-inflammatory drugs (NSAIDs). d. Provide education about antibiotic therapy.

ANS: A Patients with bloody diarrhea should have a stool culture for E. coli O157:H7. NSAIDs may cause occult blood in the stools, but not diarrhea. Antidiarrheal medications usually are avoided for possible infectious diarrhea to avoid prolonging the infection. Antibiotic therapy in the treatment of infectious diarrhea is controversial because it may precipitate kidney complications.

The nurse is caring for a 68-year-old hospitalized patient with a decreased glomerular filtration rate who is scheduled for an intravenous pyelogram (IVP). Which action will be included in the plan of care? a. Monitor the urine output after the procedure. b. Assist with monitored anesthesia care (MAC). c. Give oral contrast solution before the procedure. d. Insert a large size urinary catheter before the IVP.

ANS: A Patients with impaired renal function are at risk for decreased renal function after IVP because the contrast medium used is nephrotoxic, so the nurse should monitor the patient's urine output. MAC sedation and retention catheterization are not required for the procedure. The contrast medium is given IV, not orally.

The health care provider orders intravenous (IV) ranitidine (Zantac) for a patient with gastrointestinal (GI) bleeding caused by peptic ulcer disease. When teaching the patient about the effect of the medication, which information will the nurse include? a. "Ranitidine decreases secretion of gastric acid." b. "Ranitidine neutralizes the acid in the stomach." c. "Ranitidine constricts the blood vessels in the stomach and decreases bleeding." d. "Ranitidine covers the ulcer with a protective material that promotes healing."

ANS: A Ranitidine is a histamine-2 (H2) receptor blocker, which decreases the secretion of gastric acid. The response beginning, "Ranitidine constricts the blood vessels" describes the effect of vasopressin. The response beginning "Ranitidine neutralizes the acid" describes the effect of antacids. And the response beginning "Ranitidine covers the ulcer" describes the action of sucralfate (Carafate).

The nurse is planning care for a 48-year-old woman with acute severe pancreatitis. The highest priority patient outcome is a. maintaining normal respiratory function. b. expressing satisfaction with pain control. c. developing no ongoing pancreatic disease. d. having adequate fluid and electrolyte balance.

ANS: A Respiratory failure can occur as a complication of acute pancreatitis, and maintenance of adequate respiratory function is the priority goal. The other outcomes would also be appropriate for the patient

A 51-year-old woman with Crohn's disease who is taking infliximab (Remicade) calls the nurse in the outpatient clinic about new symptoms. Which symptom is most important to communicate to the health care provider? a. Fever b. Nausea c. Joint pain d. Headache

ANS: A Since infliximab suppresses the immune response, rapid treatment of infection is essential. The other patient complaints are common side effects of the medication, but they do not indicate any potentially life-threatening complications

When counseling a patient with a family history of stomach cancer about ways to decrease risk for developing stomach cancer, the nurse will teach the patient to avoid a. smoked foods such as bacon and ham. b. foods that cause abdominal distention. c. chronic use of H2 blocking medications. d. emotionally or physically stressful situations.

ANS: A Smoked foods such as bacon, ham, and smoked sausage increase the risk for stomach cancer. Use of H2 blockers, stressful situations, and abdominal distention are not associated with an increased incidence of stomach cancer.

What glomerular filtration rate (GFR) would the nurse estimate for a 30-year-old patient with a creatinine clearance result of 60 mL/min? a. 60 mL/min b. 90 mL/min c. 120 mL/min d. 180 mL/min

ANS: A The creatinine clearance approximates the GFR. The other responses are not accurate

To palpate the liver during a head-to-toe physical assessment, the nurse a. places one hand on the patient's back and presses upward and inward with the other hand below the patient's right costal margin. b. places one hand on top of the other and uses the upper fingers to apply pressure and the bottom fingers to feel for the liver edge. c. presses slowly and firmly over the right costal margin with one hand and withdraws the fingers quickly after the liver edge is felt. d. places one hand under the patient's lower ribs and presses the left lower rib cage forward, palpating below the costal margin with the other hand.

ANS: A The liver is normally not palpable below the costal margin. The nurse needs to push inward below the right costal margin while lifting the patient's back slightly with the left hand. The other methods will not allow palpation of the liver

Which medication taken at home by a 47-year-old patient with decreased renal function will be of most concern to the nurse? a. ibuprofen (Motrin) b. warfarin (Coumadin) c. folic acid (vitamin B9) d. penicillin (Bicillin LA)

ANS: A The nonsteroidal antiinflammatory medications (NSAIDs) are nephrotoxic and should be avoided in patients with impaired renal function. The nurse also should ask about reasons the patient is taking the other medications, but the medication of most concern is the ibuprofen.

A patient recovering from a gastrojejunostomy (Billroth II) for treatment of a duodenal ulcer develops dizziness, weakness, and palpitations about 20 minutes after eating. To avoid recurrence of these symptoms, the nurse teaches the patient to a. lie down for about 30 minutes after eating. b. choose foods that are high in carbohydrates. c. increase the amount of fluid intake with meals. d. drink sugared fluids or eat candy after each meal.

ANS: A The patient is experiencing symptoms of dumping syndrome, which may be reduced by lying down after eating. Increasing fluid intake and choosing high carbohydrate foods will increase the risk for dumping syndrome. Having a sweet drink or hard candy will correct the hypoglycemia that is associated with dumping syndrome but will not prevent dumping syndrome.

A patient is being scheduled for endoscopic retrograde cholangiopancreatography (ERCP) as soon as possible. Which actions from the agency policy for ERCP should the nurse take first? a. Place the patient on NPO status. b. Administer sedative medications. c. Ensure the consent form is signed. d. Teach the patient about the procedure.

ANS: A The patient will need to be NPO for 8 hours before the ERCP is done, so the nurse's initial action should be to place the patient on NPO status. The other actions can be done after the patient is NPO

A 23-year-old has been admitted with acute liver failure. Which assessment data are most important for the nurse to communicate to the health care provider? a. Asterixis and lethargy b. Jaundiced sclera and skin c. Elevated total bilirubin level d. Liver 3 cm below costal margin

ANS: A The patient's findings of asterixis and lethargy are consistent with grade 2 hepatic encephalopathy. Patients with acute liver failure can deteriorate rapidly from grade 1 or 2 to grade 3 or 4 hepatic encephalopathy and need early transfer to a transplant center. The other findings are typical of patients with hepatic failure and would be reported but would not indicate a need for an immediate change in the therapeutic plan.

A patient with a bleeding duodenal ulcer has a nasogastric (NG) tube in place, and the health care provider orders 30 mL of aluminum hydroxide/magnesium hydroxide (Maalox) to be instilled through the tube every hour. To evaluate the effectiveness of this treatment, the nurse a. periodically aspirates and tests gastric pH. b. monitors arterial blood gas values on a daily basis. c. checks each stool for the presence of occult blood. d. measures the amount of residual stomach contents hourly.

ANS: A The purpose for antacids is to increase gastric pH. Checking gastric pH is the most direct way of evaluating the effectiveness of the medication. Arterial blood gases may change slightly, but this does not directly reflect the effect of antacids on gastric pH. Because the patient has upper gastrointestinal (GI) bleeding, occult blood in the stools will appear even after the acute bleeding has stopped. The amount of residual stomach contents is not a reflection of resolution of bleeding or of gastric pH.

Which finding indicates to the nurse that lactulose (Cephulac) is effective for a 72-year-old man who has advanced cirrhosis? a. The patient is alert and oriented. b. The patient denies nausea or anorexia. c. The patient's bilirubin level decreases. d. The patient has at least one stool daily.

ANS: A The purpose of lactulose in the patient with cirrhosis is to lower ammonia levels and prevent encephalopathy. Although lactulose may be used to treat constipation, that is not the purpose for this patient. Lactulose will not decrease nausea and vomiting or lower bilirubin levels.

A patient who is receiving continuous enteral nutrition through a small-bore silicone feeding tube has a computed tomography (CT) scan ordered and will have to be placed in a flat position for the scan. Which action by the nurse is best? a. Shut the feeding off 30 to 60 minutes before the scan. b. Ask the health care provider to reschedule the CT scan. c. Connect the feeding tube to continuous suction during the scan. d. Send the patient to CT scan with oral suction in case of aspiration.

ANS: A The tube feeding should be shut off 30 to 60 minutes before any procedure requiring the patient to lie flat. Because the CT scan is ordered for diagnosis of patient problems, rescheduling is not usually an option. Prevention, rather than treatment, of aspiration is needed. Small-bore feeding tubes are soft and collapse easily with aspiration or suction, making nasogastric suction of gastric contents unreliable.

The nurse administering a-interferon and ribavirin (Rebetol) to a patient with chronic hepatitis C will plan to monitor for a. leukopenia. b. hypokalemia. c. polycythemia. d. hypoglycemia.

ANS: A Therapy with ribavirin and a-interferon may cause leukopenia. The other problems are not associated with this drug therapy

The client scheduled to have an intravenous urogram is a diabetic and taking the antidiabetic agent metformin. What should the nurse tell this client? A. "Call your diabetes doctor and tell him or her that you are having an intravenous urogram performed using dye." B. "Do not take your metformin the morning of the test because you are not going to be eating anything and could become hypoglycemic." C. "You must start on an antibiotic before this test because your risk of infection is greater as a result of your diabetes." D. "You must take your metformin immediately before the test is performed because the IV fluid and the dye contain a significant amount of sugar."

ANS: A Metformin can cause a lactic acidosis and renal impairment as an interaction with the dye. This drug must be discontinued for 48 hours before the procedure and not started again after the procedure until urine output is well established.

What would be the response if a person's nephrons were not able to filter normally due to scarring of the proximal convoluted tubule leading to inhibition of reabsorption? A. Increased urine output, fluid volume deficit B. Decreased urine output, fluid volume deficit C. Increased urine output, fluid volume overload D. Decreased urine output, fluid volume overload

ANS: A The nephrons filter about 120 mL/min. Most of this filtrate is reabsorbed in the proximal convoluted tubule. If the tubule were not able to reabsorb the fluid that has been filtered, urine output would greatly increase, leading to rapid and severe dehydration.

After an unimmunized individual is exposed to hepatitis B through a needle-stick injury, which actions will the nurse plan to take (select all that apply)? a. Administer hepatitis B vaccine. b. Test for antibodies to hepatitis B. c. Teach about a-interferon therapy. d. Give hepatitis B immune globulin. e. Teach about choices for oral antiviral therapy.

ANS: A, B, D The recommendations for hepatitis B exposure include both vaccination and immune globulin administration. In addition, baseline testing for hepatitis B antibodies will be needed. Interferon and oral antivirals are not used for hepatitis B prophylaxis

Which information will the nurse include when teaching a patient how to avoid chronic constipation (select all that apply)? a. Many over-the-counter (OTC) medications can cause constipation. b. Stimulant and saline laxatives can be used regularly. c. Bulk-forming laxatives are an excellent source of fiber. d. Walking or cycling frequently will help bowel motility. e. A good time for a bowel movement may be after breakfast.

ANS: A, C, D, E Stimulant and saline laxatives should be used infrequently. Use of bulk-forming laxatives, regular early morning timing of defecation, regular exercise, and avoiding many OTC medications will help the patient avoid constipation

During a busy day, the nurse admits all of the following patients to the medical-surgical unit. Which patients are most important to refer to the dietitian for a complete nutritional assessment (select all that apply)? a. A 24-year-old who has a history of weight gains and losses b. A 53-year-old who complains of intermittent nausea for the past 2 days c. A 66-year-old who is admitted for débridement of an infected surgical wound d. A 45-year-old admitted with chest pain and possible myocardial infarction (MI) e. A 32-year-old with rheumatoid arthritis who takes prednisone (Deltasone) daily

ANS: A, C, E Weight fluctuations, use of corticosteroids, and draining or infected wounds all suggest that the patient may be at risk for malnutrition. Patients with chest pain or MI are not usually poorly nourished. Although vomiting that lasts 5 days places a patient at risk, nausea that has persisted for 2 days does not always indicate poor nutritional status or risk for health problems caused by poor nutrition.

The nurse will plan to monitor a patient with an obstructed common bile duct for a. melena. b. steatorrhea. c. decreased serum cholesterol levels. d. increased serum indirect bilirubin levels.

ANS: B A common bile duct obstruction will reduce the absorption of fat in the small intestine, leading to fatty stools. Gastrointestinal (GI) bleeding is not caused by common bile duct obstruction. Serum cholesterol levels are increased with biliary obstruction. Direct bilirubin level is increased with biliary obstruction

The nurse recognizes that teaching a 44-year-old woman following a laparoscopic cholecystectomy has been effective when the patient states which of the following? a. "I can expect yellow-green drainage from the incision for a few days." b. "I can remove the bandages on my incisions tomorrow and take a shower." c. "I should plan to limit my activities and not return to work for 4 to 6 weeks." d. "I will always need to maintain a low-fat diet since I no longer have a gallbladder."

ANS: B After a laparoscopic cholecystectomy, the patient will have Band-Aids in place over the incisions. Patients are discharged the same (or next) day and have few restrictions on activities of daily living. Drainage from the incisions would be abnormal, and the patient should be instructed to call the health care provider if this occurs. A low-fat diet may be recommended for a few weeks after surgery but will not be a life-long requirement

Parenteral nutrition (PN) containing amino acids and dextrose was ordered and hung 24 hours ago for a malnourished patient. The nurse observes that about 50 mL remain in the PN container. Which action is best for the nurse to take? a. Ask the health care provider to clarify the written PN order. b. Add a new container of PN using the current tubing and filter. c. Hang a new container of PN and change the IV tubing and filter. d. Infuse the remaining 50 mL and then hang a new container of PN.

ANS: B All PN solutions are changed at 24 hours. PN solutions containing dextrose and amino acids require a change in tubing and filter every 72 hours rather than daily. Infusion of the additional 50 mL will increase patient risk for infection. Changing the IV tubing and filter more frequently than required will unnecessarily increase costs. The nurse (not the health care provider) is responsible for knowing the indicated times for tubing and filter changes.

A 74-year-old patient preparing to undergo a colon resection for cancer of the colon asks about the elevated carcinoembryonic antigen (CEA) test result. The nurse explains that the test is used to a. identify any metastasis of the cancer. b. monitor the tumor status after surgery. c. confirm the diagnosis of a specific type of cancer. d. determine the need for postoperative chemotherapy.

ANS: B CEA is used to monitor for cancer recurrence after surgery. CEA levels do not help to determine whether there is metastasis of the cancer. Confirmation of the diagnosis is made on the basis of biopsy. Chemotherapy use is based on factors other than CEA

A patient being admitted with an acute exacerbation of ulcerative colitis reports crampy abdominal pain and passing 15 or more bloody stools a day. The nurse will plan to a. administer IV metoclopramide (Reglan). b. discontinue the patient's oral food intake. c. administer cobalamin (vitamin B12) injections. d. teach the patient about total colectomy surgery.

ANS: B An initial therapy for an acute exacerbation of inflammatory bowel disease (IBD) is to rest the bowel by making the patient NPO. Metoclopramide increases peristalsis and will worsen symptoms. Cobalamin (vitamin B12) is absorbed in the ileum, which is not affected by ulcerative colitis. Although total colectomy is needed for some patients, there is no indication that this patient is a candidate

Which information given by a 70-year-old patient during a health history indicates to the nurse that the patient should be screened for hepatitis C? a. The patient had a blood transfusion in 2005. b. The patient used IV drugs about 20 years ago. c. The patient frequently eats in fast-food restaurants. d. The patient traveled to a country with poor sanitation.

ANS: B Any patient with a history of IV drug use should be tested for hepatitis C. Blood transfusions given after 1992 (when an antibody test for hepatitis C became available) do not pose a risk for hepatitis C. Hepatitis C is not spread by the oral-fecal route and therefore is not caused by contaminated food or by traveling in underdeveloped countries

Which nursing action should be included in the postoperative plan of care for a patient after a laparoscopic esophagectomy? a. Notify the doctor about bloody nasogastric (NG) drainage. b. Elevate the head of the bed to at least 30 degrees. c. Reposition the NG tube if drainage stops. d. Start oral fluids when the patient has active bowel sounds.

ANS: B Elevation of the head of the bed decreases the risk for reflux and aspiration of gastric secretions. The NG tube should not be repositioned without consulting with the health care provider. Bloody NG drainage is expected for the first 8 to 12 hours. A swallowing study is needed before oral fluids are started

The nurse is caring for a 73-year-old man who has cirrhosis. Which data obtained by the nurse during the assessment will be of most concern? a. The patient complains of right upper-quadrant pain with palpation. b. The patient's hands flap back and forth when the arms are extended. c. The patient has ascites and a 2-kg weight gain from the previous day. d. The patient's skin has multiple spider-shaped blood vessels on the abdomen.

ANS: B Asterixis indicates that the patient has hepatic encephalopathy, and hepatic coma may occur. The spider angiomas and right upper quadrant abdominal pain are not unusual for the patient with cirrhosis and do not require a change in treatment. The ascites and weight gain indicate the need for treatment but not as urgently as the changes in neurologic status

The nurse preparing for the annual physical exam of a 50-year-old man will plan to teach the patient about a. endoscopy. b. colonoscopy. c. computerized tomography screening. d. carcinoembryonic antigen (CEA) testing.

ANS: B At age 50, individuals with an average risk for colorectal cancer (CRC) should begin screening for CRC. Colonoscopy is the gold standard for CRC screening. The other diagnostic tests are not recommended as part of a routine annual physical exam at age 50.

Which nursing action will the nurse include in the plan of care for a 35-year-old male patient admitted with an exacerbation of inflammatory bowel disease (IBD)? a. Restrict oral fluid intake. b. Monitor stools for blood. c. Ambulate four times daily. d. Increase dietary fiber intake.

ANS: B Because anemia or hemorrhage may occur with IBD, stools should be assessed for the presence of blood. The other actions would not be appropriate for the patient with IBD. Because dietary fiber may increase gastrointestinal (GI) motility and exacerbate the diarrhea, severe fatigue is common with IBD exacerbations, and dehydration may occur.

How will the nurse assess for flank tenderness in a 30-year-old female patient with suspected pyelonephritis? a. Palpate along both sides of the lumbar vertebral column. b. Strike a flat hand covering the costovertebral angle (CVA). c. Push fingers upward into the two lowest intercostal spaces. d. Percuss between the iliac crest and ribs along the midaxillary line.

ANS: B Checking for flank pain is best performed by percussion of the CVA and asking about pain. The other techniques would not assess for flank pain

A patient who has been NPO during treatment for nausea and vomiting caused by gastric irritation is to start oral intake. Which of these should the nurse offer to the patient? a. A glass of orange juice b. A dish of lemon gelatin c. A cup of coffee with cream d. A bowl of hot chicken broth

ANS: B Clear liquids are usually the first foods started after a patient has been nauseated. Acidic foods such as orange juice, very hot foods, and coffee are poorly tolerated when patients have been nauseated.

The nurse determines that teaching regarding cobalamin injections has been effective when the patient with chronic atrophic gastritis states which of the following? a. "The cobalamin injections will prevent gastric inflammation." b. "The cobalamin injections will prevent me from becoming anemic." c. "These injections will increase the hydrochloric acid in my stomach." d. "These injections will decrease my risk for developing stomach cancer."

ANS: B Cobalamin supplementation prevents the development of pernicious anemia. Chronic gastritis may cause achlorhydria, but cobalamin does not correct this. The loss of intrinsic factor secretion with chronic gastritis is permanent, and the patient will need lifelong supplementation with cobalamin. The incidence of stomach cancer is higher in patients with chronic gastritis, but cobalamin does not reduce the risk for stomach cancer. DIF: Cognitive Level: Apply (application) REF: 941-942 TOP: Nursing Process: Evaluation MSC: NCLEX: Physiological Integrity

Which information will the nurse prioritize in planning preoperative teaching for a patient undergoing a Roux-en-Y gastric bypass? a. Educating the patient about the nasogastric (NG) tube b. Instructing the patient on coughing and breathing techniques c. Discussing necessary postoperative modifications in lifestyle d. Demonstrating passive range-of-motion exercises for the legs

ANS: B Coughing and deep breathing can prevent major postoperative complications such as carbon monoxide retention and hypoxemia. Information about passive range of motion, the NG tube, and postoperative modifications in lifestyle will also be discussed, but avoidance of respiratory complications is the priority goal after surgery

The nurse is assessing a 31-year-old female patient with abdominal pain. Th nurse,who notes that there is ecchymosis around the area of umbilicus, will document this finding as a. Cullen sign. b. Rovsing sign. c. McBurney sign. d. Grey-Turner's signt.

ANS: B Cullen sign is ecchymosis around the umbilicus. Rovsing sign occurs when palpation of the left lower quadrant causes pain in the right lower quadrant. Deep tenderness at McBurney's point (halfway between the umbilicus and the right iliac crest), known as McBurney's sign, is a sign of acute appendicitis

Which of these assessment findings in a patient with a hiatal hernia who returned from a laparoscopic Nissen fundoplication 4 hours ago is most important for the nurse to address immediately? a. The patient is experiencing intermittent waves of nausea. b. The patient has absent breath sounds throughout the left lung. c. The patient has decreased bowel sounds in all four quadrants. d. The patient complains of 6/10 (0 to 10 scale) abdominal pain.

ANS: B Decreased breath sounds on one side may indicate a pneumothorax, which requires rapid diagnosis and treatment. The abdominal pain and nausea also should be addressed but they are not as high priority as the patient's respiratory status. The patient's decreased bowel sounds are expected after surgery and require ongoing monitoring but no other action.

Which action should the nurse take first in order to improve calorie and protein intake for a patient who eats only about 50% of each meal because of "feeling too tired to eat much." a. Teach the patient about the importance of good nutrition. b. Serve multiple small feedings of high-calorie, high-protein foods. c. Obtain an order for enteral feedings of liquid nutritional supplements. d. Consult with the health care provider about providing parenteral nutrition (PN).

ANS: B Eating small amounts of food frequently throughout the day is less fatiguing and will improve the patient's ability to take in more nutrients. Teaching the patient may be appropriate, but will not address the patient's inability to eat more because of fatigue. Tube feedings or PN may be needed if the patient is unable to take in enough nutrients orally, but increasing the oral intake should be attempted first.

Which assessment action will help the nurse determine if an obese patient has metabolic syndrome? a. Take the patient's apical pulse. b. Check the patient's blood pressure. c. Ask the patient about dietary intake. d. Dipstick the patient's urine for protein.

ANS: B Elevated blood pressure is one of the characteristics of metabolic syndrome. The other information also may be obtained by the nurse, but it will not assist with the diagnosis of metabolic syndrome

After a total proctocolectomy and permanent ileostomy, the patient tells the nurse, "I cannot manage all these changes. I don't want to look at the stoma." What is the best action by the nurse? a. Reassure the patient that ileostomy care will become easier. b. Ask the patient about the concerns with stoma management. c. Develop a detailed written list of ostomy care tasks for the patient. d. Postpone any teaching until the patient adjusts to the ileostomy.

ANS: B Encouraging the patient to share concerns assists in helping the patient adjust to the body changes. Acknowledgment of the patient's feelings and concerns is important rather than offering false reassurance. Because the patient indicates that the feelings about the ostomy are the reason for the difficulty with the many changes, development of a detailed ostomy care plan will not improve the patient's ability to manage the ostomy. Although detailed ostomy teaching may be postponed, the nurse should offer teaching about some aspects of living with an ostomy.

Which action should the nurse take to evaluate treatment effectiveness for a patient who has hepatic encephalopathy? a. Request that the patient stand on one foot. b. Ask the patient to extend both arms forward. c. Request that the patient walk with eyes closed. d. Ask the patient to perform the Valsalva maneuver.

ANS: B Extending the arms allows the nurse to check for asterixis, a classic sign of hepatic encephalopathy. The other tests might also be done as part of the neurologic assessment but would not be diagnostic for hepatic encephalopathy.

A family member of a 28-year-old patient who has suffered massive abdominal trauma in an automobile accident asks the nurse why the patient is receiving famotidine (Pepcid). The nurse will explain that the medication will a. decrease nausea and vomiting. b. inhibit development of stress ulcers. c. lower the risk for H. pylori infection. d. prevent aspiration of gastric contents.

ANS: B Famotidine is administered to prevent the development of physiologic stress ulcers, which are associated with a major physiologic insult such as massive trauma. Famotidine does not decrease nausea or vomiting, prevent aspiration, or prevent H. pylori infection

A 34-year old patient with chronic hepatitis C infection has several medications prescribed. Which medication requires further discussion with the health care provider before administration? a. Ribavirin (Rebetol, Copegus) 600 mg PO bid b. Pegylated a-interferon (PEG-Intron, Pegasys) SQ daily c. Diphenhydramine (Benadryl) 25 mg PO every 4 hours PRN itching d. Dimenhydrinate (Dramamine) 50 mg PO every 6 hours PRN nausea

ANS: B Pegylated a-interferon is administered weekly. The other medications are appropriate for a patient with chronic hepatitis C infection

A 54-year-old critically ill patient with sepsis is frequently incontinent of watery stools. What action by the nurse will prevent complications associated with ongoing incontinence? a. Apply incontinence briefs. b. Use a fecal management system c. Insert a rectal tube with a drainage bag. d. Assist the patient to a commode frequently.

ANS: B Fecal management systems are designed to contain loose stools and can be in place for as long as 4 weeks without causing damage to the rectum or anal sphincters. Although incontinence briefs may be helpful, unless they are changed frequently, they are likely to increase the risk for skin breakdown. Rectal tubes are avoided because of possible damage to the anal sphincter and ulceration of the rectal mucosa. A critically ill patient will not be able to tolerate getting up frequently to use the commode or bathroom.

A 24-year-old woman with Crohn's disease develops a fever and symptoms of a urinary tract infection (UTI) with tan, fecal-smelling urine. What information will the nurse add to a general teaching plan about UTIs in order to individualize the teaching for this patient? a. Bacteria in the perianal area can enter the urethra. b. Fistulas can form between the bowel and bladder. c. Drink adequate fluids to maintain normal hydration. d. Empty the bladder before and after sexual intercourse.

ANS: B Fistulas between the bowel and bladder occur in Crohn's disease and can lead to UTI. Teaching for UTI prevention in general includes good hygiene, adequate fluid intake, and voiding before and after intercourse

Which information will the nurse include in teaching a patient who had a proctocolectomy and ileostomy for ulcerative colitis? a. Restrict fluid intake to prevent constant liquid drainage from the stoma. b. Use care when eating high-fiber foods to avoid obstruction of the ileum. c. Irrigate the ileostomy daily to avoid having to wear a drainage appliance. d. Change the pouch every day to prevent leakage of contents onto the skin.

ANS: B High-fiber foods are introduced gradually and should be well chewed to avoid obstruction of the ileostomy. Patients with ileostomies lose the absorption of water in the colon and need to take in increased amounts of fluid. The pouch should be drained frequently but is changed every 5 to 7 days. The drainage from an ileostomy is liquid and continuous, so control by irrigation is not possible

The nurse caring for a patient after cystoscopy plans that the patient a. learns to request narcotics for pain. b. understands to expect blood-tinged urine. c. restricts activity to bed rest for a 4 to 6 hours. d. remains NPO for 8 hours to prevent vomiting.

ANS: B Pink-tinged urine and urinary frequency are expected after cystoscopy. Burning on urination is common, but pain that requires opioids for relief is not expected. A good fluid intake is encouraged after this procedure. Bed rest is not required following cystoscopy

A patient gives the nurse health information before a scheduled intravenous pyelogram (IVP). Which item has the most immediate implications for the patient's care? a. The patient has not had food or drink for 8 hours. b. The patient lists allergies to shellfish and penicillin. c. The patient complains of costovertebral angle (CVA) tenderness. d. The patient used a bisacodyl (Dulcolax) tablet the previous night.

ANS: B Iodine-based contrast dye is used during IVP and for many computed tomography (CT) scans. The nurse will need to notify the health care provider before the procedures so that the patient can receive medications such as antihistamines or corticosteroids before the procedures are started. The other information is also important to note and document but does not have immediate implications for the patient's care during the procedures

Which information will the nurse teach a 23-year-old patient with lactose intolerance? a. Ice cream is relatively low in lactose. b. Live-culture yogurt is usually tolerated. c. Heating milk will break down the lactose. d. Nonfat milk is a better choice than whole milk.

ANS: B Lactose-intolerant individuals can usually eat yogurt without experiencing discomfort. Ice cream, nonfat milk, and milk that has been heated are all high in lactose

After change-of-shift report, which patient should the nurse assess first? a. 40-year-old male with celiac disease who has frequent frothy diarrhea b. 30-year-old female with a femoral hernia who has abdominal pain and vomiting c. 30-year-old male with ulcerative colitis who has severe perianal skin breakdown d. 40-year-old female with a colostomy bag that is pulling away from the adhesive wafer

ANS: B Pain and vomiting with a femoral hernia suggest possible strangulation, which will necessitate emergency surgery. The other patients have less urgent problems

Which action will the nurse in the gastrointestinal clinic include in the plan of care? a. Obtain blood samples for DNA analysis. b. Schedule the patient for yearly colonoscopy. c. Provide preoperative teaching about total colectomy. d. Discuss lifestyle modifications to decrease cancer risk.

ANS: B Patients with FAP should have annual colonoscopy starting at age 16 and usually have total colectomy by age 25 to avoid developing colorectal cancer. DNA analysis is used to make the diagnosis, but is not needed now for this patient. Lifestyle modifications will not decrease cancer risk for this patient

A new 19-year-old male patient has familial adenomatous polyposis (FAP). Which action will the nurse in the gastrointestinal clinic include in the plan of care? a. Obtain blood samples for DNA analysis. b. Schedule the patient for yearly colonoscopy. c. Provide preoperative teaching about total colectomy. d. Discuss lifestyle modifications to decrease cancer risk.

ANS: B Patients with FAP should have annual colonoscopy starting at age 16 and usually have total colectomy by age 25 to avoid developing colorectal cancer. DNA analysis is used to make the diagnosis, but is not needed now for this patient. Lifestyle modifications will not decrease cancer risk for this patient.

For a patient with cirrhosis, which of the following nursing actions can the registered nurse (RN) delegate to unlicensed assistive personnel (UAP)? a. Assessing the patient for jaundice b. Providing oral hygiene after a meal c. Palpating the abdomen for distention d. Assisting the patient to choose the diet

ANS: B Providing oral hygiene is within the scope of UAP. Assessments and assisting patients to choose therapeutic diets are nursing actions that require higher-level nursing education and scope of practice and would be delegated to licensed practical/vocational nurses (LPNs/LVNs) or RNs

A 58-year-old man with blunt abdominal trauma from a motor vehicle crash undergoes peritoneal lavage. If the lavage returns brown fecal drainage, which action will the nurse plan to take next? a. Auscultate the bowel sounds. b. Prepare the patient for surgery. c. Check the patient's oral temperature. d. Obtain information about the accident.

ANS: B Return of brown drainage and fecal material suggests perforation of the bowel and the need for immediate surgery. Auscultation of bowel sounds, checking the temperature, and obtaining information about the accident are appropriate actions, but the priority is to prepare to send the patient for emergency surgery

A serum potassium level of 3.2 mEq/L (3.2 mmol/L) is reported for a patient with cirrhosis who has scheduled doses of spironolactone (Aldactone) and furosemide (Lasix). due. Which action should the nurse take? a. Administer both drugs. b. Administer the spironolactone. c. Withhold the spironolactone and administer the furosemide. d. Withhold both drugs until discussed with the health care provider.

ANS: B Spironolactone is a potassium-sparing diuretic and will help increase the patient's potassium level. The nurse does not need to talk with the doctor before giving the spironolactone, although the health care provider should be notified about the low potassium value. The furosemide will further decrease the patient's potassium level and should be held until the nurse talks with the health care provider.

Which patient statement indicates that the nurse's teaching about sulfasalazine (Azulfidine) for ulcerative colitis has been effective? a. "The medication will be tapered if I need surgery." b. "I will need to use a sunscreen when I am outdoors." c. "I will need to avoid contact with people who are sick." d. "The medication will prevent infections that cause the diarrhea."

ANS: B Sulfasalazine may cause photosensitivity in some patients. It is not used to treat infections. Sulfasalazine does not reduce immune function. Unlike corticosteroids, tapering of sulfasalazine is not needed.

Which information about dietary management should the nurse include when teaching a patient with peptic ulcer disease (PUD)? a. "You will need to remain on a bland diet." b. "Avoid foods that cause pain after you eat them." c. "High-protein foods are least likely to cause you pain." d. "You should avoid eating any raw fruits and vegetables."

ANS: B The best information is that each individual should choose foods that are not associated with postprandial discomfort. Raw fruits and vegetables may irritate the gastric mucosa, but chewing well seems to decrease this problem and some patients may tolerate these foods well. High-protein foods help neutralize acid, but they also stimulate hydrochloric (HCl) acid secretion and may increase discomfort for some patients. Bland diets may be recommended during an acute exacerbation of PUD, but there is little scientific evidence to support their use. DIF: Cognitive Level: Apply (application) REF: 947 TOP: Nursing Process: Implementation MSC: NCLEX: Physiological Integrity

Which topic is most important to include in patient teaching for a 41-year-old patient diagnosed with early alcoholic cirrhosis? a. Maintaining good nutrition b. Avoiding alcohol ingestion c. Taking lactulose (Cephulac) d. Using vitamin B supplements

ANS: B The disease progression can be stopped or reversed by alcohol abstinence. The other interventions may be used when cirrhosis becomes more severe to decrease symptoms or complications, but the priority for this patient is to stop the progression of the disease

Which medications will the nurse teach the patient about whose peptic ulcer disease is associated with Helicobacter pylori? a. Sucralfate (Carafate), nystatin (Mycostatin), and bismuth (Pepto-Bismol) b. Amoxicillin (Amoxil), clarithromycin (Biaxin), and omeprazole (Prilosec) c. Famotidine (Pepcid), magnesium hydroxide (Mylanta), and pantoprazole (Protonix) d. Metoclopramide (Reglan), bethanechol (Urecholine), and promethazine (Phenergan)

ANS: B The drugs used in triple drug therapy include a proton pump inhibitor such as omeprazole and the antibiotics amoxicillin and clarithromycin. The other combinations listed are not included in the protocol for H. pylori infection

A patient with peptic ulcer disease associated with the presence of Helicobacter pylori is treated with triple drug therapy. The nurse will plan to teach the patient about a. sucralfate (Carafate), nystatin (Mycostatin), and bismuth (Pepto-Bismol). b. amoxicillin (Amoxil), clarithromycin (Biaxin), and omeprazole (Prilosec). c. famotidine (Pepcid), magnesium hydroxide (Mylanta), and pantoprazole (Protonix). d. metoclopramide (Reglan), bethanechol (Urecholine), and promethazine (Phenergan).

ANS: B The drugs used in triple drug therapy include a proton pump inhibitor such as omeprazole and the antibiotics amoxicillin and clarithromycin. The other combinations listed are not included in the protocol for H. pylori infection.

When preparing to teach an 82-year-old Hispanic patient who lives with an adult daughter about ways to improve nutrition, which action should the nurse take first? a. Ask the daughter about the patient's food preferences. b. Determine who shops for groceries and prepares the meals. c. Question the patient about how many meals per day are eaten. d. Assure the patient that culturally appropriate foods will be included.

ANS: B The family member who shops for groceries and cooks will be in control of the patient's diet, so the nurse will need to ensure that this family member is involved in any teaching or discussion about the patient's nutritional needs. The other information also will be assessed and used but will not be useful in meeting the patient's nutritional needs unless nutritionally appropriate foods are purchased and prepared.

A 62- year-old man reports chronic constipation. To promote bowel evacuation, the nurse will suggest that the patient attempt defecation a. in the mid-afternoon. b. after eating breakfast. c. right after getting up in the morning. d. immediately before the first daily meal.

ANS: B The gastrocolic reflex is most active after the first daily meal. Arising in the morning, the anticipation of eating, and physical exercise do not stimulate these reflexes.

Which goal has the highest priority in the plan of care for a 26-year-old homeless patient admitted with viral hepatitis who has severe anorexia and fatigue? a. Increase activity level. b. Maintain adequate nutrition. c. Establish a stable environment. d. Identify sources of hepatitis exposure.

ANS: B The highest priority outcome is to maintain nutrition because adequate nutrition is needed for hepatocyte regeneration. Finding a home for the patient and identifying the source of the infection would be appropriate activities, but they do not have as high a priority as ensuring adequate nutrition. Although the patient's activity level will be gradually increased, rest is indicated during the acute phase of hepatitis

The nurse will determine that teaching a 67-year-old man to irrigate his new colostomy has been effective if the patient a. inserts the irrigation tubing 4 to 6 inches into the stoma. b. hangs the irrigating container 18 inches above the stoma. c. stops the irrigation and removes the irrigating cone if cramping occurs. d. fills the irrigating container with 1000 to 2000 mL of lukewarm tap water.

ANS: B The irrigating container should be hung 18 to 24 inches above the stoma. If cramping occurs, the irrigation should be temporarily stopped and the cone left in place. Five hundred to 1000 mL of water should be used for irrigation. An irrigation cone, rather than tubing, should be inserted into the stoma; 4 to 6 inches would be too far for safe insertion.

The nurse completing a physical assessment for a newly admitted male patient is unable to feel either kidney on palpation. Which action should the nurse take next? a. Obtain a urine specimen to check for hematuria. b. Document the information on the assessment form. c. Ask the patient about any history of recent sore throat. d. Ask the health care provider about scheduling a renal ultrasound.

ANS: B The kidneys are protected by the abdominal organs, ribs, and muscles of the back, and may not be palpable under normal circumstances, so no action except to document the assessment information is needed. Asking about a recent sore throat, checking for hematuria, or obtaining a renal ultrasound may be appropriate when assessing for renal problems for some patients, but there is nothing in the question stem to indicate that they are appropriate for this patient

A 53-year-old patient is being treated for bleeding esophageal varices with balloon tamponade. Which nursing action will be included in the plan of care? a. Instruct the patient to cough every hour. b. Monitor the patient for shortness of breath. c. Verify the position of the balloon every 4 hours. d. Deflate the gastric balloon if the patient reports nausea.

ANS: B The most common complication of balloon tamponade is aspiration pneumonia. In addition, if the gastric balloon ruptures, the esophageal balloon may slip upward and occlude the airway. Coughing increases the pressure on the varices and increases the risk for bleeding. Balloon position is verified after insertion and does not require further verification. The esophageal balloon is deflated every 8 to 12 hours to avoid necrosis, but if the gastric balloon is deflated, the esophageal balloon may occlude the airway

A 71-year-old male patient tells the nurse that growing old causes constipation so he has been using a suppository for constipation every morning. Which action should the nurse take first? a. Encourage the patient to increase oral fluid intake. b. Assess the patient about risk factors for constipation. c. Suggest that the patient increase intake of high-fiber foods. d. Teach the patient that a daily bowel movement is unnecessary.

ANS: B The nurse's initial action should be further assessment of the patient for risk factors for constipation and for his usual bowel pattern. The other actions may be appropriate but will be based on the assessment.

A 79-year-old man has been admitted with benign prostatic hyperplasia. What is most appropriate to include in the nursing plan of care? a. Limit fluid intake to no more than 1000 mL/day. b. Leave a light on in the bathroom during the night. c. Ask the patient to use a urinal so that urine can be measured. d. Pad the patient's bed to accommodate overflow incontinence.

ANS: B The patient's age and diagnosis indicate a likelihood of nocturia, so leaving the light on in the bathroom is appropriate. Fluids should be encouraged because dehydration is more common in older patients. The information in the question does not indicate that measurement of the patient's output is necessary or that the patient has overflow incontinence.

A 22-year-old who is hospitalized with anorexia nervosa is 5 ft 5 in (163 cm) tall and weighs 90 pounds (41 kg). Laboratory tests reveal hypokalemia and iron-deficiency anemia. Which nursing diagnosis has the highest priority for the patient? a. Risk for activity intolerance related to anemia and weakness b. Risk for electrolyte imbalance related to poor eating patterns c. Ineffective health maintenance related to obsession with body image d. Imbalanced nutrition: less than body requirements related to refusal to eat

ANS: B The patient's hypokalemia may lead to life-threatening cardiac dysrhythmias. The other diagnoses also are appropriate for this patient but are not associated with immediate risk for fatal complications.

A 44-year-old man admitted with a peptic ulcer has a nasogastric (NG) tube in place. When the patient develops sudden, severe upper abdominal pain, diaphoresis, and a firm abdomen, which action should the nurse take? a. Irrigate the NG tube. b. Check the vital signs. c. Give the ordered antacid. d. Elevate the foot of the bed.

ANS: B The patient's symptoms suggest acute perforation, and the nurse should assess for signs of hypovolemic shock. Irrigation of the NG tube, administration of antacids, or both would be contraindicated because any material in the stomach will increase the spillage into the peritoneal cavity. Elevating the foot of the bed may increase abdominal pressure and discomfort, as well as making it more difficult for the patient to breathe

A female patient with a suspected urinary tract infection (UTI) is to provide a clean-catch urine specimen for culture and sensitivity testing. To obtain the specimen, the nurse will a. have the patient empty the bladder completely, then obtain the next urine specimen that the patient is able to void. b. teach the patient to clean the urethral area, void a small amount into the toilet, and then void into a sterile specimen cup. c. insert a short sterile "mini" catheter attached to a collecting container into the urethra and bladder to obtain the specimen. d. clean the area around the meatus with a povidone-iodine (Betadine) swab, and then have the patient void into a sterile container.

ANS: B This answer describes the technique for obtaining a clean-catch specimen. The answer beginning, "insert a short, small, 'mini' catheter attached to a collecting container" describes a technique that would result in a sterile specimen, but a health care provider's order for a catheterized specimen would be required. Using Betadine before obtaining the specimen is not necessary, and might result in suppressing the growth of some bacteria. The technique described in the answer beginning "have the patient empty the bladder completely" would not result in a sterile specimen

After the nurse teaches a patient about the recommended amounts of foods from animal and plant sources, which menu selections indicate that the initial instructions about diet have been understood? a. 3 oz of lean beef, 2 oz of low-fat cheese, and a tomato slice b. 3 oz of roasted pork, a cup of corn, and a cup of carrot sticks c. Cup of tossed salad and nonfat dressing topped with a chicken breast d. Half cup of tuna mixed with nonfat mayonnaise and a half cup of celery

ANS: B This selection is most consistent with the recommendation of the American Institute for Cancer Research that one third of the diet should be from animal sources and two thirds from plant source foods. The other choices all have higher ratios of animal origin foods to plant source foods than would be recommended

Which information about an 80-year-old man at the senior center is of most concern to the nurse? a. Decreased appetite b. Unintended weight loss c. Difficulty chewing food d. Complaints of indigestion

ANS: B Unintentional weight loss is not a normal finding and may indicate a problem such as cancer or depression. Poor appetite, difficulty in chewing, and complaints of indigestion are common in older patients. These will need to be addressed but are not of as much concern as the weight loss

The client is taking a medication for an endocrine problem that inhibits aldosterone secretion and release. To what complications of this therapy should the nurse be alert? A. Dehydration, hypokalemia B. Dehydration, hyperkalemia C. Overhydration, hyponatremia D. Overhydration, hypernatremia

ANS: B Aldosterone is a mineralocorticoid that increases the reabsorption of water and sodium in the kidney at the same time that it promotes excretion of potassium. Any drug or condition that disrupts aldosterone secretion or release increases the client's risk for excessive water loss and potassium reabsorption.

Which condition would trigger the release of antidiuretic hormone (ADH)? A. Plasma osmolarity decreased secondary to overhydration. B. Plasma osmolarity increased secondary to dehydration. C. Plasma volume decreased secondary to hemorrhage. D. Plasma volume increased with edema formation.

ANS: B Antidiuretic hormone is triggered by a rising ECF osmolarity, especially hypernatremia.

The female client's urinalysis shows all the following characteristics. Which should the nurse document as abnormal? A. pH 5.6 B. Ketone bodies present C. Specific gravity is 1.030 D. Two white blood cells per high-power field

ANS: B Ketone bodies are byproducts of incomplete metabolism of fatty acids. Normally, there are no ketones in urine. Ketone bodies are produced when fat sources are used instead of glucose to provide cellular energy.

Which of the following conditions are associated with oversecretion of rennin? A. Alzheimer's disease B. Hypertension C. Diabetes mellitus D. Diabetes insipidus

ANS: B Renin is secreted when special cells in the DCT, called the macula densa, sense changes in blood volume and pressure. When the macula densa cells sense that blood volume is low, blood pressure is low, or blood sodium levels are low, renin is secreted. Renin then converts angiotensinogen into angiotensin I. This leads to a series of reactions that cause the secretion of the hormone aldosterone. This hormone increases kidney reabsorption of sodium and water, increasing blood pressure, blood volume, and blood sodium levels. Inappropriate or excessive renin secretion is a major cause of persistent hypertension.

The client's urine specific gravity is 1.018. What is the nurse's best action? A. Ask the client for a 24-hour recall of liquid intake. B. Document the finding as the only action. C. Obtain a specimen for culture. D. Notify the physician.

ANS: B This specific gravity is within the normal range for urine.

Which of the following muscle actions results in voluntary urination? A. Detrusor contraction, external sphincter contraction B. Detrusor contraction, external sphincter relaxation C. Detrusor relaxation, external sphincter contraction D. Detrusor relaxation, external sphincter relaxation

ANS: B Voiding becomes a voluntary act as a result of learned responses controlled by the cerebral cortex that cause contraction of the bladder detrusor muscle and simultaneous relaxation of the external urethral sphincter muscle.

The nurse is providing preoperative teaching for a 61-year-old man scheduled for an abdominal-perineal resection. Which information will the nurse include? a. Another surgery in 8 to 12 weeks will be used to create an ileal-anal reservoir. b. The patient will begin sitting in a chair at the bedside on the first postoperative day. c. The patient will drink polyethylene glycol lavage solution (GoLYTELY) preoperatively. d. IV antibiotics will be started at least 24 hours before surgery to reduce the bowel bacteria.

ANS: C A bowel-cleansing agent is used to empty the bowel before surgery to reduce the risk for infection. A permanent colostomy is created with this surgery. Sitting is contraindicated after an abdominal-perineal resection. Oral antibiotics (rather than IV antibiotics) are given to reduce colonic and rectal bacteria

A 26-year-old woman is being evaluated for vomiting and abdominal pain. Which question from the nurse will be most useful in determining the cause of the patient's symptoms? a. "What type of foods do you eat?" b. "Is it possible that you are pregnant?" c. "Can you tell me more about the pain?" d. "What is your usual elimination pattern?"

ANS: C A complete description of the pain provides clues about the cause of the problem. Although the nurse should ask whether the patient is pregnant to determine whether the patient might have an ectopic pregnancy and before any radiology studies are done, this information is not the most useful in determining the cause of the pain. The usual diet and elimination patterns are less helpful in determining the reason for the patient's symptoms

A 58-year-old patient has just been admitted to the emergency department with nausea and vomiting. Which information requires the most rapid intervention by the nurse? a. The patient has been vomiting for 4 days. b. The patient takes antacids 8 to 10 times a day. c. The patient is lethargic and difficult to arouse. d. The patient has undergone a small intestinal resection.

ANS: C A lethargic patient is at risk for aspiration, and the nurse will need to position the patient to decrease aspiration risk. The other information is also important to collect, but it does not require as quick action as the risk for aspiration.

When the nurse is assessing the mouth of a patient who uses smokeless tobacco for signs of oral cancer, which finding will be of most concern? a. Bleeding during tooth brushing b. Painful blisters at the border of the lips c. Red, velvety patches on the buccal mucosa d. White, curdlike plaques on the posterior tongue

ANS: C A red, velvety patch suggests erythroplasia, which has a high incidence (greater than 50%) of progression to squamous cell carcinoma. The other lesions are suggestive of acute processes (gingivitis, oral candidiasis, and herpes simplex).

A 54-year-old man has just arrived in the recovery area after an upper endoscopy. Which information collected by the nurse is most important to communicate to the health care provider? a. The patient is very drowsy. b. The patient reports a sore throat. c. The oral temperature is 101.6° F. d. The apical pulse is 104 beats/minute.

ANS: C A temperature elevation may indicate that a perforation has occurred. The other assessment data are normal immediately after the procedure.

Which action will the nurse include in the plan of care for a 42-year-old patient who is being admitted with Clostridium difficile? a. Educate the patient about proper food storage. b. Order a diet with no dairy products for the patient. c. Place the patient in a private room on contact isolation. d. Teach the patient about why antibiotics will not be used.

ANS: C Because C. difficile is highly contagious, the patient should be placed in a private room and contact precautions should be used. There is no need to restrict dairy products for this type of diarrhea. Metronidazole (Flagyl) is frequently used to treat C. difficile. Improper food handling and storage do not cause C. difficile.

A 62-year-old patient has had a hemorrhoidectomy at an outpatient surgical center. Which instructions will the nurse include in discharge teaching? a. Maintain a low-residue diet until the surgical area is healed. b. Use ice packs on the perianal area to relieve pain and swelling. c. Take prescribed pain medications before a bowel movement is expected. d. Delay having a bowel movement for several days until healing has occurred.

ANS: C Bowel movements may be very painful, and patients may avoid defecation unless pain medication is taken before the bowel movement. A high-residue diet will increase stool bulk and prevent constipation. Delay of bowel movements is likely to lead to constipation. Warm sitz baths rather than ice packs are used to relieve pain and keep the surgical area clean

A 38-year old woman receiving chemotherapy for breast cancer develops a Candida albicans oral infection. The nurse will anticipate the need for a. hydrogen peroxide rinses. b. the use of antiviral agents. c. administration of nystatin (Mycostatin) tablets. d. referral to a dentist for professional tooth cleaning.

ANS: C Candida albicans is treated with an antifungal such as nystatin. Oral saltwater rinses may be used but will not cure the infection. Antiviral agents are used for viral infections such as herpes simplex. Referral to a dentist is indicated for gingivitis but not for Candida infection

Which statement to the nurse from a patient with jaundice indicates a need for teaching? a. "I used cough syrup several times a day last week." b. "I take a baby aspirin every day to prevent strokes." c. "I use acetaminophen (Tylenol) every 4 hours for back pain." d. "I need to take an antacid for indigestion several times a week"

ANS: C Chronic use of high doses of acetaminophen can be hepatotoxic and may have caused the patient's jaundice. The other patient statements require further assessment by the nurse, but do not indicate a need for patient education

The nurse implements discharge teaching for a patient following a gastroduodenostomy for treatment of a peptic ulcer. Which patient statement indicates that the teaching has been effective? a. "Persistent heartburn is expected after surgery." b. "I will try to drink liquids along with my meals." c. "Vitamin supplements may be needed to prevent problems with anemia." d. "I will need to choose foods that are low in fat and high in carbohydrate."

ANS: C Cobalamin deficiency may occur after partial gastrectomy, and the patient may need to receive cobalamin via injections or nasal spray. Foods that have moderate fat and low carbohydrate should be chosen to prevent dumping syndrome. Ingestion of liquids with meals is avoided to prevent dumping syndrome. Although peptic ulcer disease may recur, persistent heartburn is not expected after surgery and the patient should call the health care provider if this occurs.

To evaluate an obese patient for adverse effects of lorcaserin (Belviq), which action will the nurse take? a. Take the apical pulse rate. b. Check sclera for jaundice. c. Ask about bowel movements. d. Assess for agitation or restlessness.

ANS: C Constipation is a common side effect of lorcaserin. The other assessments would be appropriate for other weight-loss medications

A patient born in 1955 had hepatitis A infection 1 year ago. According to Centers for Disease Control and Prevention (CDC) guidelines, which action should the nurse include in care when the patient is seen for a routine annual physical exam? a. Start the hepatitis B immunization series. b. Teach the patient about hepatitis A immune globulin. c. Ask whether the patient has been screened for hepatitis C. d. Test for anti-hepatitis-A virus immune globulin M (anti-HAV-IgM).

ANS: C Current CDC guidelines indicate that all patients who were born between 1945 and 1965 should be screened for hepatitis C because many individuals who are positive have not been diagnosed. Although routine hepatitis B immunization is recommended for infants, children, and adolescents, vaccination for hepatitis B is recommended only for adults at risk for blood-borne infections. Because the patient has already had hepatitis A, immunization and anti-HAV IgM levels will not be needed.

All of the following nursing actions are included in the plan of care for a patient who is malnourished. Which action is appropriate for the nurse to delegate to nursing assistive personnel (NAP)? a. Assist the patient to choose high nutrition items from the menu. b. Monitor the patient for skin breakdown over the bony prominences. c. Offer the patient the prescribed nutritional supplement between meals. d. Assess the patient's strength while ambulating the patient in the room.

ANS: C Feeding the patient and assisting with oral intake are included in NAP education and scope of practice. Assessing the patient and assisting the patient in choosing high nutrition foods require LPN/LVN- or RN-level education and scope of practice.

A 46-year-old female with gastroesophageal reflux disease (GERD) is experiencing increasing discomfort. Which patient statement indicates that additional teaching about GERD is needed? a. "I take antacids between meals and at bedtime each night." b. "I sleep with the head of the bed elevated on 4-inch blocks." c. "I eat small meals during the day and have a bedtime snack." d. "I quit smoking several years ago, but I still chew a lot of gum."

ANS: C GERD is exacerbated by eating late at night, and the nurse should plan to teach the patient to avoid eating at bedtime. The other patient actions are appropriate to control symptoms of GERD

A patient passing bloody urine is scheduled for a cystoscopy with cystogram. Which description of the procedure by the nurse is accurate? a. "Your doctor will place a catheter into an artery in your groin and inject a dye that will visualize the blood supply to the kidneys." b. "Your doctor will insert a lighted tube into the bladder, and little catheters will be inserted through the tube into your kidney." c. "Your doctor will insert a lighted tube into the bladder through your urethra, inspect the bladder, and instill a dye that will outline your bladder on x-ray." d. "Your doctor will inject a radioactive solution into a vein in your arm and the distribution of the isotope in your kidneys and bladder will be checked."

ANS: C In a cystoscope and cystogram procedure, a cystoscope is inserted into the bladder for direct visualization, and then contrast solution is injected through the scope so that x-rays can be taken. The response beginning, "Your doctor will place a catheter" describes a renal arteriogram procedure. The response beginning, "Your doctor will inject a radioactive solution" describes a nuclear scan. The response beginning, "Your doctor will insert a lighted tube into the bladder, and little catheters will be inserted" describes a retrograde pyelogram

An 80-year-old who is hospitalized with peptic ulcer disease develops new-onset auditory hallucinations. Which prescribed medication will the nurse discuss with the health care provider before administration? a. Sucralfate (Carafate) b. Omeprazole (Prilosec) c. Metoclopramide (Reglan) d. Aluminum hydroxide (Amphojel)

ANS: C Metoclopramide can cause central nervous system (CNS) side effects ranging from anxiety to hallucinations. Hallucinations are not a side effect of proton-pump inhibitors, mucosal protectants, or antacids

After 6 hours of parenteral nutrition (PN) infusion, the nurse checks a patient's capillary blood glucose level and finds it to be 120 mg/dL. The most appropriate action by the nurse is to a. obtain a venous blood glucose specimen. b. slow the infusion rate of the PN infusion. c. recheck the capillary blood glucose in 4 hours. d. notify the health care provider of the glucose level.

ANS: C Mild hyperglycemia is expected during the first few days after PN is started and requires ongoing monitoring. Because the glucose elevation is small and expected, notification of the health care provider is not necessary. There is no need to obtain a venous specimen for comparison. Slowing the rate of the infusion is beyond the nurse's scope of practice and will decrease the patient's nutritional intake.

Which statement by the nurse is most likely to help a morbidly obese 22-year-old man in losing weight on a 1000-calorie diet? a. "It will be necessary to change lifestyle habits permanently to maintain weight loss." b. "You will decrease your risk for future health problems such as diabetes by losing weight now." c. "You are likely to notice changes in how you feel with just a few weeks of diet and exercise." d. "Most of the weight that you lose during the first weeks of dieting is water weight rather than fat."

ANS: C Motivation is a key factor in successful weight loss and a short-term outcome provides a higher motivation. A 22-year-old patient is unlikely to be motivated by future health problems. Telling a patient that the initial weight loss is water will be discouraging, although this may be correct. Changing lifestyle habits is necessary, but this process occurs over time and discussing this is not likely to motivate the patient

The nurse will teach a patient with chronic pancreatitis to take the prescribed pancrelipase (Viokase) a. at bedtime. b. in the morning. c. with each meal. d. for abdominal pain.

ANS: C Pancreatic enzymes are used to help with digestion of nutrients and should be taken with every meal

Four hours after a bowel resection, a 74-year-old male patient with a nasogastric tube to suction complains of nausea and abdominal distention. The first action by the nurse should be to a. auscultate for hypotonic bowel sounds. b. notify the patient's health care provider. c. reposition the tube and check for placement. d. remove the tube and replace it with a new one.

ANS: C Repositioning the tube will frequently facilitate drainage. Because this is a common occurrence, it is not appropriate to notify the health care provider unless other interventions do not resolve the problem. Information about the presence or absence of bowel sounds will not be helpful in improving drainage. Removing the tube and replacing it are unnecessarily traumatic to the patient, so that would only be done if the tube was completely occluded

A patient is receiving tube feedings through a percutaneous endoscopic gastrostomy (PEG). Which action will the nurse include in the plan of care? a. Keep the patient positioned on the left side. b. Obtain a daily x-ray to verify tube placement. c. Check the gastric residual volume every 4 to 6 hours. d. Avoid giving bolus tube feedings through the PEG tube.

ANS: C The gastric residual volume is assessed every 4 to 6 hours to decrease the risk for aspiration. The patient does not need to be positioned on the left side. An x-ray is obtained immediately after placement of the PEG tube to check position, but daily x-rays are not needed. Bolus feedings can be administered through a PEG tube.

Which assessment of a 62-year-old patient who has just had an intravenous pyelogram (IVP) requires immediate action by the nurse? a. The heart rate is 58 beats/minute. b. The patient complains of a dry mouth. c. The respiratory rate is 38 breaths/minute. d. The urine output is 400 mL after 2 hours.

ANS: C The increased respiratory rate indicates that the patient may be experiencing an allergic reaction to the contrast medium used during the procedure. The nurse should immediately assess the patient's oxygen saturation and breath sounds. The other data are not unusual findings following an IVP.

A 76-year-old patient with obstipation has a fecal impaction and is incontinent of liquid stool. Which action should the nurse take first? a. Administer bulk-forming laxatives. b. Assist the patient to sit on the toilet. c. Manually remove the impacted stool. d. Increase the patient's oral fluid intake.

ANS: C The initial action with a fecal impaction is manual disimpaction. The other actions will be used to prevent future constipation and impactions

A 19-year-old female is brought to the emergency department with a knife handle protruding from the abdomen. During the initial assessment of the patient, the nurse should a. remove the knife and assess the wound. b. determine the presence of Rovsing sign. c. check for circulation and tissue perfusion. d. insert a urinary catheter and assess for hematuria.

ANS: C The initial assessment is focused on determining whether the patient has hypovolemic shock. The knife should not be removed until the patient is in surgery, where bleeding can be controlled. Rovsing sign is assessed in the patient with suspected appendicitis. A patient with a knife in place will be taken to surgery and assessed for bladder trauma there

After successfully losing 1 lb weekly for several months, a patient at the clinic has not lost any weight for the last month. The nurse should first a. review the diet and exercise guidelines with the patient. b. instruct the patient to weigh and record weights weekly. c. ask the patient whether there have been any changes in exercise or diet patterns. d. discuss the possibility that the patient has reached a temporary weight loss plateau.

ANS: C The initial nursing action should be assessment of any reason for the change in weight loss. The other actions may be needed, but further assessment is required before any interventions are planned or implemented

Which action should the nurse in the emergency department take first for a new patient who is vomiting blood? a. Insert a large-gauge IV catheter. b. Draw blood for coagulation studies. c. Check blood pressure (BP), heart rate, and respirations. d. Place the patient in the supine position.

ANS: C The nurse's first action should be to determine the patient's hemodynamic status by assessing vital signs. Drawing blood for coagulation studies and inserting an IV catheter are also appropriate. However, the vital signs may indicate the need for more urgent actions. Because aspiration is a concern for this patient, the nurse will need to assess the patient's vital signs and neurologic status before placing the patient in a supine position

To prepare a 56-year-old male patient with ascites for paracentesis, the nurse a. places the patient on NPO status. b. assists the patient to lie flat in bed. c. asks the patient to empty the bladder. d. positions the patient on the right side.

ANS: C The patient should empty the bladder to decrease the risk of bladder perforation during the procedure. The patient would be positioned in Fowler's position and would not be able to lie flat without compromising breathing. Because no sedation is required for paracentesis, the patient does not need to be NPO.

A 22-year-old patient with Escherichia coli O157:H7 food poisoning is admitted to the hospital with bloody diarrhea and dehydration. All of the following orders are received. Which order will the nurse question? a. Infuse lactated Ringer's solution at 250 mL/hr. b. Monitor blood urea nitrogen and creatinine daily. c. Administer loperamide (Imodium) after each stool. d. Provide a clear liquid diet and progress diet as tolerated.

ANS: C Use of antidiarrheal agents is avoided with this type of food poisoning. The other orders are appropriate.

A 38-year-old patient with cirrhosis has ascites and 4+ edema of the feet and legs. Which nursing action will be included in the plan of care? a. Restrict daily dietary protein intake. b. Reposition the patient every 4 hours. c. Place the patient on a pressure-relieving mattress. d. Perform passive range of motion daily.

ANS: C The pressure-relieving mattress will decrease the risk for skin breakdown for this patient. Adequate dietary protein intake is necessary in patients with ascites to improve oncotic pressure. Repositioning the patient every 4 hours will not be adequate to maintain skin integrity. Passive range of motion will not take the pressure off areas such as the sacrum that are vulnerable to breakdown

What information will the nurse include for an overweight 35-year-old woman who is starting a weight-loss plan? a. Weigh yourself at the same time every morning and evening. b. Stick to a 600- to 800-calorie diet for the most rapid weight loss. c. Low carbohydrate diets lead to rapid weight loss but are difficult to maintain. d. Weighing all foods on a scale is necessary to choose appropriate portion sizes.

ANS: C The restrictive nature of fad diets makes the weight loss achieved by the patient more difficult to maintain. Portion size can be estimated in other ways besides weighing. Severely calorie-restricted diets are not necessary for patients in the overweight category of obesity and need to be closely supervised. Patients should weigh weekly rather than daily

Which finding is most important for the nurse to communicate to the health care provider about a patient who received a liver transplant 1 week ago? a. Dry palpebral and oral mucosa b. Crackles at bilateral lung bases c. Temperature 100.8° F (38.2° C) d. No bowel movement for 4 days

ANS: C The risk of infection is high in the first few months after liver transplant and fever is frequently the only sign of infection. The other patient data indicate the need for further assessment or nursing actions and might be communicated to the health care provider, but they do not indicate a need for urgent action

During change-of-shift report, the nurse learns about the following four patients. Which patient requires assessment first? a. 40-year-old with chronic pancreatitis who has gnawing abdominal pain b. 58-year-old who has compensated cirrhosis and is complaining of anorexia c. 55-year-old with cirrhosis and ascites who has an oral temperature of 102° F (38.8° C) d. 36-year-old recovering from a laparoscopic cholecystectomy who has severe shoulder pain

ANS: C This patient's history and fever suggest possible spontaneous bacterial peritonitis, which would require rapid assessment and interventions such as antibiotic therapy. The clinical manifestations for the other patients are consistent with their diagnoses and do not indicate complications are occurring

The nurse notes that the peripheral parenteral nutrition (PN) bag has only 20 mL left and a new PN bag has not yet arrived from the pharmacy. Which intervention is the priority? a. Monitor the patient's capillary blood glucose until a new PN bag is hung b. Flush the peripheral line with saline and wait until the new PN bag is available c. Infuse 5% dextrose in water until the new PN bag is delivered from the pharmacy d. Decrease the rate of the current PN infusion to 10 mL/hr until the new bag arrives

ANS: C To prevent hypoglycemia, the nurse should infuse a 5% dextrose solution until the next PN bag can be started. Decreasing the rate of the ordered PN infusion is beyond the nurse's scope of practice. Flushing the line and then waiting for the next bag may lead to hypoglycemia. Monitoring the capillary blood glucose is appropriate but is not the priority.

Which care activity for a patient with a paralytic ileus is appropriate for the registered nurse (RN) to delegate to unlicensed assistive personnel (UAP)? a. Auscultation for bowel sounds b. Nasogastric (NG) tube irrigation c. Applying petroleum jelly to the lips d. Assessment of the nares for irritation

ANS: C UAP education and scope of practice include patient hygiene such as oral care. The other actions require education and scope of practice appropriate to the RN.

A 57-year-old man with Escherichia coli O157:H7 food poisoning is admitted to the hospital with bloody diarrhea and dehydration. Which order will the nurse question? a. Infuse lactated Ringer's solution at 250 mL/hr. b. Monitor blood urea nitrogen and creatinine daily. c. Administer loperamide (Imodium) after each stool. d. Provide a clear liquid diet and progress diet as tolerated.

ANS: C Use of antidiarrheal agents is avoided with this type of food poisoning. The other orders are appropriate

A 22-year-old female patient with an exacerbation of ulcerative colitis is having 15 to 20 stools daily and has excoriated perianal skin. Which patient behavior indicates that teaching regarding maintenance of skin integrity has been effective? a. The patient uses incontinence briefs to contain loose stools. b. The patient asks for antidiarrheal medication after each stool. c. The patient uses witch hazel compresses to decrease irritation. d. The patient cleans the perianal area with soap after each stool.

ANS: C Witch hazel compresses are suggested to reduce anal irritation and discomfort. Incontinence briefs may trap diarrhea and increase the incidence of skin breakdown. Antidiarrheal medications are not given 15 to 20 times a day. The perianal area should be washed with plain water after each stool.

With a renal threshold for glucose of 220 mg/dL, what is the expected response when a client has a blood glucose level of 400 mg/dL? A. 400 mg/dL of excreted glucose in the urine B. 220 mg/dL of excreted glucose in the urine C. 180 mg/dL of glucose is excreted in the urine D. No excreted glucose in the urine

ANS: C Blood glucose is freely filtered at the glomerulus. Therefore, if a client has a blood sugar level of 400 mg/dl, the filtrate in the proximal convoluted tubule will have a glucose concentration of 400 mg/dL. With a renal threshold of 220 mg/dl, a total of 220 mg/dL of the 400 mg/dL will be reabsorbed back into the systemic circulation, and the final urine will have a glucose concentration of 180 mg/dL.

The client is going home after urography. Which instruction or precaution should the nurse teach this client? A. "Avoid direct contact with the urine for 24 hours until the radioisotope clears." B. "You are likely to experience some dribbling of urine for several weeks after this procedure." C. "Be sure to drink at least 3 L of fluids today to help eliminate the dye faster." D. "Your skin may become slightly yellow-tinged from the dye used in this procedure."

ANS: C Dyes used in urography are potentially nephrotoxic.

What is the result of stimulation of erythropoietin production in the kidney tissue? A. Increased blood flow to the kidney B. Inhibition of vitamin D and loss of bone density C. Increased bone marrow production of red blood cells D. Inhibition of the active transport of sodium, leading to hyponatremi

ANS: C Erythropoietin is produced in the kidney and released in response to decreased oxygen tension in the renal blood supply. Erythropoietin stimulates red blood cell (RBC) production in the bone marrow.

Confirmed by palpation and x-ray study, the client's right kidney is lower than the left kidney. What is the nurse's interpretation of this finding? A. The client has a problem involving the right kidney. B. The client has a problem involving the left kidney. C. The client has both kidneys in the normal position. D. The client is at increased risk for kidney impairment.

ANS: C Normally, the right kidney is positioned somewhat lower than the left kidney. This anatomic difference in otherwise symmetric organs is caused by liver displacement. The significance of this difference is that the right kidney is easier to palpate in an adult than is the left kidney.

A 64-year-old woman who has chronic constipation asks the nurse about the use of psyllium (Metamucil). Which information will the nurse include in the response? a. Absorption of fat-soluble vitamins may be reduced by fiber-containing laxatives. b. Dietary sources of fiber should be eliminated to prevent excessive gas formation. c. Use of this type of laxative to prevent constipation does not cause adverse effects. d. Large amounts of fluid should be taken to prevent impaction or bowel obstruction.

ANS: D A high fluid intake is needed when patients are using bulk-forming laxatives to avoid worsening constipation. Although bulk-forming laxatives are generally safe, the nurse should emphasize the possibility of constipation or obstipation if inadequate fluid intake occurs. Although increased gas formation is likely to occur with increased dietary fiber, the patient should gradually increase dietary fiber and eventually may not need the psyllium. Fat-soluble vitamin absorption is blocked by stool softeners and lubricants, not by bulk-forming laxatives

Which assessment finding is of most concern for a 46-year-old woman with acute pancreatitis? a. Absent bowel sounds b. Abdominal tenderness c. Left upper quadrant pain d. Palpable abdominal mass

ANS: D A palpable abdominal mass may indicate the presence of a pancreatic abscess, which will require rapid surgical drainage to prevent sepsis. Absent bowel sounds, abdominal tenderness, and left upper quadrant pain are common in acute pancreatitis and do not require rapid action to prevent further complications

Which nursing action is essential for a patient immediately after a renal biopsy? a. Check blood glucose to assess for hyperglycemia or hypoglycemia. b. Insert a urinary catheter and test urine for gross or microscopic hematuria. c. Monitor the blood urea nitrogen (BUN) and creatinine to assess renal function. d. Apply a pressure dressing and keep the patient on the affected side for 30 minutes.

ANS: D A pressure dressing is applied and the patient is kept on the affected side for 30 to 60 minutes to put pressure on the biopsy side and decrease the risk for bleeding. The blood glucose and BUN/creatinine will not be affected by the biopsy. Although monitoring for hematuria is needed, there is no need for catheterization

A 42-year-old male patient has had a herniorrhaphy to repair an incarcerated inguinal hernia. Which patient teaching will the nurse provide before discharge? a. Soak in sitz baths several times each day. b. Cough 5 times each hour for the next 48 hours. c. Avoid use of acetaminophen (Tylenol) for pain. d. Apply a scrotal support and ice to reduce swelling.

ANS: D A scrotal support and ice are used to reduce edema and pain. Coughing will increase pressure on the incision. Sitz baths will not relieve pain and would not be of use after this surgery. Acetaminophen can be used for postoperative pain.

The nurse will ask a 64-year-old patient being admitted with acute pancreatitis specifically about a history of a. diabetes mellitus. b. high-protein diet. c. cigarette smoking. d. alcohol consumption.

ANS: D Alcohol use is one of the most common risk factors for pancreatitis in the United States. Cigarette smoking, diabetes, and high-protein diets are not risk factors

Which breakfast choice indicates a patient's good understanding of information about a diet for celiac disease? a. Oatmeal with nonfat milk b. Whole wheat toast with butter c. Bagel with low-fat cream cheese d. Corn tortilla with scrambled eggs

ANS: D Avoidance of gluten-containing foods is the only treatment for celiac disease. Corn does not contain gluten, while oatmeal and wheat do

A patient who recently has been experiencing frequent heartburn is seen in the clinic. The nurse will anticipate teaching the patient about a. barium swallow. b. radionuclide tests. c. endoscopy procedures. d. proton pump inhibitors.

ANS: D Because diagnostic testing for heartburn that is probably caused by gastroesophageal reflux disease (GERD) is expensive and uncomfortable, proton pump inhibitors are frequently used for a short period as the first step in the diagnosis of GERD. The other tests may be used but are not usually the first step in diagnosis.

A patient with protein calorie malnutrition who has had abdominal surgery is receiving parenteral nutrition (PN). Which assessment information obtained by the nurse is the best indicator that the patient is receiving adequate nutrition? a. Blood glucose is 110 mg/dL. b. Serum albumin level is 3.5 mg/dL. c. Fluid intake and output are balanced. d. Surgical incision is healing normally.

ANS: D Because poor wound healing is a possible complication of malnutrition for this patient, normal healing of the incision is an indicator of the effectiveness of the PN in providing adequate nutrition. Blood glucose is monitored to prevent the complications of hyperglycemia and hypoglycemia, but it does not indicate that the patient's nutrition is adequate. The intake and output will be monitored but do not indicate that the PN is effective. The albumin level is in the low-normal range but does not reflect adequate caloric intake, which is also important for the patient.

A patient who is receiving chemotherapy develops a Candida albicans oral infection. The nurse will anticipate the need for a. hydrogen peroxide rinses. b. the use of antiviral agents. c. referral to a dentist for professional tooth cleaning. d. administration of nystatin (Mycostatin) oral tablets.

ANS: D Candida albicans is treated with an antifungal such as nystatin. Oral saltwater rinses may be used but will not cure the infection. Antiviral agents are used for viral infections such as herpes simplex. Referral to a dentist is indicated for gingivitis but not for Candida infection.

The nurse will plan to teach a patient with Crohn's disease who has megaloblastic anemia about the need for a. oral ferrous sulfate tablets. b. regular blood transfusions. c. iron dextran (Imferon) infusions. d. cobalamin (B12) spray or injections.

ANS: D Crohn's disease frequently affects the ileum, where absorption of cobalamin occurs. Cobalamin must be administered regularly by nasal spray or IM to correct the anemia. Iron deficiency does not cause megaloblastic anemia. The patient may need occasional transfusions but not regularly scheduled transfusions

To assess whether there is any improvement in a patient's dysuria, which question will the nurse ask? a. "Do you have to urinate at night?" b. "Do you have blood in your urine?" c. "Do you have to urinate frequently?" d. "Do you have pain when you urinate?"

ANS: D Dysuria is painful urination. The alternate responses are used to assess other urinary tract symptoms: hematuria, nocturia, and frequency

When a 72-year-old patient is diagnosed with achalasia, the nurse will teach the patient that a. lying down after meals is recommended. b. a liquid or blenderized diet will be necessary. c. drinking fluids with meals should be avoided. d. treatment may include endoscopic procedures.

ANS: D Endoscopic and laparoscopic procedures are the most effective therapy for improving symptoms caused by achalasia. Keeping the head elevated after eating will improve esophageal emptying. A semisoft diet is recommended to improve esophageal emptying. Patients are advised to drink fluid with meals. DIF: Cognitive Level: Apply (application) REF: 940 TOP: Nursing Process: Planning MSC: NCLEX: Physiological Integrity

A 32-year-old patient who is employed as a hairdresser and has a 15 pack-year history of cigarette smoking is scheduled for an annual physical examination. The nurse will plan to teach the patient about the increased risk for a. renal failure. b. kidney stones. c. pyelonephritis. d. bladder cancer.

ANS: D Exposure to the chemicals involved with working as a hairdresser and in smoking both increase the risk of bladder cancer, and the nurse should assess whether the patient understands this risk. The patient is not at increased risk for renal failure, pyelonephritis, or kidney stones

A 42-year-old woman is admitted to the outpatient testing area for an ultrasound of the gallbladder. Which information obtained by the nurse indicates that the ultrasound may need to be rescheduled? a. The patient took a laxative the previous evening. b. The patient had a high-fat meal the previous evening. c. The patient has a permanent gastrostomy tube in place. d. The patient ate a low-fat bagel 4 hours ago for breakfast.

ANS: D Food intake can cause the gallbladder to contract and result in a suboptimal study. The patient should be NPO for 8 to 12 hours before the test. A high-fat meal the previous evening, laxative use, or a gastrostomy tube will not affect the results of the study.

The nurse receives the following information about a 51-year-old woman who is scheduled for a colonoscopy. Which information should be communicated to the health care provider before sending the patient for the procedure? a. The patient has a permanent pacemaker to prevent bradycardia. b. The patient is worried about discomfort during the examination. c. The patient has had an allergic reaction to shellfish and iodine in the past. d. The patient refused to drink the ordered polyethylene glycol (GoLYTELY).

ANS: D If the patient has had inadequate bowel preparation, the colon cannot be visualized and the procedure should be rescheduled. Because contrast solution is not used during colonoscopy, the iodine allergy is not pertinent. A pacemaker is a contraindication to magnetic resonance imaging (MRI), but not to colonoscopy. The nurse should instruct the patient about the sedation used during the examination to decrease the patient's anxiety about discomfort

When working in the urology/nephrology clinic, which patient could the nurse delegate to an experienced licensed practical/vocational nurse (LPN/LVN)? a. Patient who is scheduled for a renal biopsy after a recent kidney transplant b. Patient who will need monitoring for several hours after a renal arteriogram c. Patient who requires teaching about possible post-cystoscopy complications d. Patient who will have catheterization to check for residual urine after voiding

ANS: D LPN/LVN education includes common procedures such as catheterization of stable patients. The other patients require more complex assessments and/or patient teaching that are included in registered nurse (RN) education and scope of practice.

A 67-year-old male patient with acute pancreatitis has a nasogastric (NG) tube to suction and is NPO. Which information obtained by the nurse indicates that these therapies have been effective? a. Bowel sounds are present. b. Grey Turner sign resolves. c. Electrolyte levels are normal. d. Abdominal pain is decreased.

ANS: D NG suction and NPO status will decrease the release of pancreatic enzymes into the pancreas and decrease pain. Although bowel sounds may be hypotonic with acute pancreatitis, the presence of bowel sounds does not indicate that treatment with NG suction and NPO status has been effective. Electrolyte levels may be abnormal with NG suction and must be replaced by appropriate IV infusion. Although Grey Turner sign will eventually resolve, it would not be appropriate to wait for this to occur to determine whether treatment was effective

Which question from the nurse would help determine if a patient's abdominal pain might indicate irritable bowel syndrome? a. "Have you been passing a lot of gas?" b. "What foods affect your bowel patterns?" c. "Do you have any abdominal distention?" d. "How long have you had abdominal pain?"

ANS: D One criterion for the diagnosis of irritable bowel syndrome (IBS) is the presence of abdominal discomfort or pain for at least 3 months. Abdominal distention, flatulence, and food intolerance are also associated with IBS, but are not diagnostic criteria.

A 51-year-old woman had an incisional cholecystectomy 6 hours ago. The nurse will place the highest priority on assisting the patient to a. choose low-fat foods from the menu. b. perform leg exercises hourly while awake. c. ambulate the evening of the operative day. d. turn, cough, and deep breathe every 2 hours.

ANS: D Postoperative nursing care after a cholecystectomy focuses on prevention of respiratory complications because the surgical incision is high in the abdomen and impairs coughing and deep breathing. The other nursing actions are also important to implement but are not as high a priority as ensuring adequate ventilation

Which action will be included in the care for a patient who has recently been diagnosed with asymptomatic nonalcoholic fatty liver disease (NAFLD)? a. Teach symptoms of variceal bleeding. b. Draw blood for hepatitis serology testing. c. Discuss the need to increase caloric intake. d. Review the patient's current medication list.

ANS: D Some medications can increase the risk for NAFLD, and they should be eliminated. NAFLD is not associated with hepatitis, weight loss is usually indicated, and variceal bleeding would not be a concern in a patient with asymptomatic NAFLD

The health care provider prescribes antacids and sucralfate (Carafate) for treatment of a patient's peptic ulcer. The nurse will teach the patient to take a. sucralfate at bedtime and antacids before each meal. b. sucralfate and antacids together 30 minutes before meals. c. antacids 30 minutes before each dose of sucralfate is taken. d. antacids after meals and sucralfate 30 minutes before meals.

ANS: D Sucralfate is most effective when the pH is low and should not be given with or soon after antacids. Antacids are most effective when taken after eating. Administration of sucralfate 30 minutes before eating and antacids just after eating will ensure that both drugs can be most effective. The other regimens will decrease the effectiveness of the medications.

Which finding indicates to the nurse that a patient's transjugular intrahepatic portosystemic shunt (TIPS) placed 3 months ago has been effective? a. Increased serum albumin level b. Decreased indirect bilirubin level c. Improved alertness and orientation d. Fewer episodes of bleeding varices

ANS: D TIPS is used to lower pressure in the portal venous system and decrease the risk of bleeding from esophageal varices. Indirect bilirubin level and serum albumin levels are not affected by shunting procedures. TIPS will increase the risk for hepatic encephalopathy

Which statement by a patient who had a cystoscopy the previous day should be reported immediately to the health care provider? a. "My urine looks pink." b. "My IV site is bruised." c. "My sleep was restless." d. "My temperature is 101."

ANS: D The patient's elevated temperature may indicate a bladder infection, a possible complication of cystoscopy. The health care provider should be notified so that antibiotic therapy can be started. Pink-tinged urine is expected after a cystoscopy. The insomnia and bruising should be discussed further with the patient but do not indicate a need to notify the health care provider.

After vertical banded gastroplasty, a 42-year-old male patient returns to the surgical nursing unit with a nasogastric tube to low, intermittent suction and a patient-controlled analgesia (PCA) machine for pain control. Which nursing action should be included in the postoperative plan of care? a. Offer sips of fruit juices at frequent intervals. b. Irrigate the nasogastric (NG) tube frequently. c. Remind the patient that PCA use may slow the return of bowel function. d. Support the surgical incision during patient coughing and turning in bed.

ANS: D The incision should be protected from strain to decrease the risk for wound dehiscence. The patient should be encouraged to use the PCA because pain control will improve the cough effort and patient mobility. NG irrigation may damage the suture line or overfill the stomach pouch. Sugar-free clear liquids are offered during the immediate postoperative time to decrease the risk for dumping syndrome

Which data will the nurse monitor in relation to the 4+ pitting edema assessed in a patient with cirrhosis? a. Hemoglobin b. Temperature c. Activity level d. Albumin level

ANS: D The low oncotic pressure caused by hypoalbuminemia is a major pathophysiologic factor in the development of edema. The other parameters should also be monitored, but they are not directly associated with the patient's current symptoms

The nurse is performing an admission assessment on a 20-year-old college student who is being admitted for electrolyte disorders of unknown etiology. Which assessment finding is most important to report to the health care provider? a. The patient's knuckles are macerated. b. The patient uses laxatives on a daily basis. c. The patient has a history of weight fluctuations. d. The patient's serum potassium level is 2.9 mEq/L.

ANS: D The low serum potassium level may cause life-threatening cardiac dysrhythmias and potassium supplementation is needed rapidly. The other information also will be reported because it suggests that bulimia may be the etiology of the patient's electrolyte disturbances, but it does not suggest imminent life-threatening complications.

A patient with a recent 20-pound unintended weight loss is diagnosed with stomach cancer. Which nursing action will be included in the plan of care? a. Refer the patient for hospice services. b. Infuse IV fluids through a central line. c. Teach the patient about antiemetic therapy. d. Offer supplemental feedings between meals.

ANS: D The patient data indicate a poor nutritional state and improvement in nutrition will be helpful in improving response to therapies such as surgery, chemotherapy, or radiation. Nausea and vomiting are not common clinical manifestations of stomach cancer. There is no indication that the patient requires hospice or IV fluid infusions.

The nurse receives change-of-shift report about the following four patients. Which patient will the nurse assess first? a. A patient who has malnutrition associated with 4+ generalized pitting edema b. A patient whose parenteral nutrition has 10 mL of solution left in the infusion bag c. A patient whose gastrostomy tube is plugged after crushed medications were given through the tube d. A patient who is receiving continuous enteral feedings and has new-onset crackles throughout the lungs

ANS: D The patient data suggest aspiration has occurred and rapid assessment and intervention are needed. The other patients also should be assessed as quickly as possible, but the data about them do not suggest any immediately life-threatening complications.

When caring for a patient with a history of a total gastrectomy, the nurse will monitor for a. constipation. b. dehydration. c. elevated total serum cholesterol. d. cobalamin (vitamin B12) deficiency.

ANS: D The patient with a total gastrectomy does not secrete intrinsic factor, which is needed for cobalamin (vitamin B12) absorption. Because the stomach absorbs only small amounts of water and nutrients, the patient is not at higher risk for dehydration, elevated cholesterol, or constipation.

A nurse is considering which patient to admit to the same room as a patient who had a liver transplant 3 weeks ago and is now hospitalized with acute rejection. Which patient would be the best choice? a. Patient who is receiving chemotherapy for liver cancer b. Patient who is receiving treatment for acute hepatitis C c. Patient who has a wound infection after cholecystectomy d. Patient who requires pain management for chronic pancreatitis

ANS: D The patient with chronic pancreatitis does not present an infection risk to the immunosuppressed patient who had a liver transplant. The other patients either are at risk for infection or currently have an infection, which will place the immunosuppressed patient at risk for infection.

After several days of antibiotic therapy, an older hospitalized patient develops watery diarrhea. Which action should the nurse take first? a. Notify the health care provider. b. Obtain a stool specimen for analysis. c. Teach the patient about handwashing. d. Place the patient on contact precautions.

ANS: D The patient's history and new onset diarrhea suggest a C. difficile infection, which requires implementation of contact precautions to prevent spread of the infection to other patients. The other actions are also appropriate but can be accomplished after contact precautions are implemented

A patient who has a wound infection after major surgery has only been taking in about 50% to 75% of the ordered meals and states, "Nothing on the menu really appeals to me." Which action by the nurse will be most effective in improving the patient's oral intake? a. Make a referral to the dietician. b. Order at least six small meals daily. c. Teach the patient about high-calorie, high-protein foods. d. Have family members bring in favorite foods from home.

ANS: D The patient's statement that the hospital foods are unappealing indicates that favorite home-cooked foods might improve intake. The other interventions also may help improve the patient's intake, but the most effective action will be to offer the patient more appealing foods.

A 45-year-old patient is admitted to the emergency department with severe abdominal pain and rebound tenderness. Vital signs include temperature 102° F (38.3° C), pulse 120, respirations 32, and blood pressure (BP) 82/54. Which prescribed intervention should the nurse implement first? a. Administer IV ketorolac (Toradol) 15 mg. b. Draw blood for a complete blood count (CBC). c. Obtain a computed tomography (CT) scan of the abdomen. d. Infuse 1 liter of lactated Ringer's solution over 30 minutes.

ANS: D The priority for this patient is to treat the patient's hypovolemic shock with fluid infusion. The other actions should be implemented after starting the fluid infusion

The nurse explaining esomeprazole (Nexium) to a patient with recurring heartburn describes that the medication a. reduces gastroesophageal reflux by increasing the rate of gastric emptying. b. neutralizes stomach acid and provides relief of symptoms in a few minutes. c. coats and protects the lining of the stomach and esophagus from gastric acid. d. treats gastroesophageal reflux disease by decreasing stomach acid production.

ANS: D The proton pump inhibitors decrease the rate of gastric acid secretion. Promotility drugs such as metoclopramide (Reglan) increase the rate of gastric emptying. Cryoprotective medications such as sucralfate (Carafate) protect the stomach. Antacids neutralize stomach acid and work rapidly

A patient who is nauseated and vomiting up blood-streaked fluid is admitted to the hospital with acute gastritis. To determine possible risk factors for gastritis, the nurse will ask the patient about a. the amount of fat in the diet. b. history of recent weight gain or loss. c. any family history of gastric problems. d. use of nonsteroidal anti-inflammatory drugs (NSAIDs).

ANS: D Use of an NSAID is associated with damage to the gastric mucosa, which can result in acute gastritis. Family history, recent weight gain or loss, and fatty foods are not risk factors for acute gastritis.

A nurse observes that the client's left flank region is larger than the right flank region. What is the nurse's best action? A. Ask the client if he or she participates in contact sports and has been recently injured. B. Document the finding as the only action on the appropriate flowsheet. C. Apply a heating pad to the left flank after inspecting the site for signs of infection. D. Anticipate further diagnostic testing after sharing informing the physician of this finding

ANS: D Asymmetry of the flank or a unilateral protrusion may indicate an enlargement of a kidney. The enlargement may be benign or may be associated with a hydronephrosis or mass on the kidney.

The client reports the regular use of all the following medications. Which one alerts the nurse to the possibility of renal impairment when used consistently? A. Antacids B. Penicillin C. Antihistamine nasal sprays D. Nonsteroidal anti-inflammatory drug

ANS: D NSAIDs inhibit prostaglandin production and decrease blood flow to the nephrons. They can cause an interstitial nephritis and renal impairment.

The client is scheduled to have a renogram (kidney scan). She is concerned about discomfort during the procedure. What is the nurse's best response? A. "Before the test you will be given a sedative to reduce any pain." B. "A local anesthetic agent will be used, so you might feel a little pressure but no pain." C. "Although this test is very sensitive, there is no more discomfort than you would have with an ordinary x-ray." D. "The only pain associated with this procedure is a small needle stick when you are given the radioisotope

ANS: D The test involves an intravenous injection of the radioisotope and the subsequent recording of the emission by a scintillator.

A patient who is given a bisacodyl (Dulcolax) suppository asks the nurse how long it will take to work. The nurse replies that the patient will probably need to use the bedpan or commode within which time frame after administration? a. 2-5 minutes b. 15-60 minutes c. 2-4 hours d. 6-8 hours

b. 15-60 minutes Bisacodyl suppositories usually are effective within 15 to 60 minutes of administration, so the nurse should plan accordingly to assist the patient to use the bedpan or commode.

A 58-year-old woman is being discharged home today after ostomy surgery for colon cancer. The nurse should assign the patient to which staff member? A. A nursing assistant on the unit who also has hospice experience B. A licensed practical nurse who has worked on the unit for 10 years C. A registered nurse with 6 months of experience on the surgical unit D. A registered nurse who has floated to the surgical unit from pediatrics

C. A registered nurse with 6 months of experience on the surgical unit The patient needs ostomy care directions/reinforcement at discharge and should be assigned to a registered nurse with experience in providing discharge teaching for ostomy care. Teaching should not be delegated to a licensed practical/vocational nurse or unlicensed assistive personnel.

What is the main underlying risk factor for metabolic syndrome? a. Age b. Heart disease c. Insulin resistance d. High cholesterol levels

Correct answer: c Rationale: Insulin resistance is the main underlying risk factor for metabolic syndrome. Aging is associated with metabolic syndrome. High cholesterol, hypertension, and increased clotting risk are characteristics of metabolic syndrome.

After an abdominal hysterectomy, a 45-year-old woman complains of severe gas pains. Her abdomen is distended. It is most appropriate for the nurse to administer which prescribed medication? A. Morphine sulfate B. Ondansetron (Zofran) C. Acetaminophen (Tylenol) D. Metoclopramide (Reglan)

D. Metoclopramide (Reglan) Swallowed air and reduced peristalsis after surgery can result in abdominal distention and gas pains. Early ambulation helps restore peristalsis and eliminate flatus and gas pain. Medications used to reduce gas pain include metoclopramide (Reglan) or alvimopan (Entereg) to stimulate peristalsis.

The condition of a patient who has cirrhosis of the liver has deteriorated. Which diagnostic study would help determine if the patient has developed liver cancer? Serum α-fetoprotein level Ventilation/perfusion scan Hepatic structure ultrasound Abdominal girth measurement

Hepatic structure ultrasound Hepatic structure ultrasound, CT, and MRI are used to screen and diagnose liver cancer. Serum α-fetoprotein level may be elevated with liver cancer or other liver problems. Ventilation/perfusion scans do not diagnose liver cancer. Abdominal girth measurement would not differentiate between cirrhosis and liver cancer.

A patient who has hepatitis B surface antigen (HBsAg) in the serum is being discharged with pain medication after knee surgery. Which medication order should the nurse question because it is most likely to cause hepatic complications? Tramadol (Ultram) Hydromorphone (Dilaudid) Oxycodone with aspirin (Percodan) Hydrocodone with acetaminophen (Vicodin)

Hydrocodone with acetaminophen (Vicodin) The analgesic with acetaminophen should be questioned because this patient is a chronic carrier of hepatitis B and is likely to have impaired liver function. Acetaminophen is not suitable for this patient because it is converted to a toxic metabolite in the liver after absorption, increasing the risk of hepatocellular damage.

A nurse is caring for a client who has had diarrhea for the past 4 days. When assessing the client, the nurse should expect which of the following findings? (Select all that apply.) ___Bradycardia ___Hypotension ___Fever ___Poor skin turgor ___Peripheral edema ___Abdominal cramping

Hypotension Fever Poor skin turgor Abdominal cramping Prolonged diarrhea leads to dehydration, which is characterized by tachycardia, hypotension, fever, lethargy, poor skin turgor, and abdominal cramping. Peripheral edema is more likely to be caused by a fluid overload rather than a fluid deficit. Chapter 45; Page 1096, Box 46-3

When planning care for a patient with cirrhosis, the nurse will give highest priority to which nursing diagnosis? Impaired skin integrity related to edema, ascites, and pruritus Imbalanced nutrition: less than body requirements related to anorexia Excess fluid volume related to portal hypertension and hyperaldosteronism Ineffective breathing pattern related to pressure on diaphragm and reduced lung volume

Ineffective breathing pattern related to pressure on diaphragm and reduced lung volume Although all of these nursing diagnoses are appropriate and important in the care of a patient with cirrhosis, airway and breathing are always the highest priorities.

The patient with sudden pain in the left upper quadrant radiating to the back and vomiting was diagnosed with acute pancreatitis. What intervention(s) should the nurse expect to include in the patient's plan of care? Immediately start enteral feeding to prevent malnutrition. Insert an NG and maintain NPO status to allow pancreas to rest. Initiate early prophylactic antibiotic therapy to prevent infection. Administer acetaminophen (Tylenol) every 4 hours for pain relief.

Insert an NG and maintain NPO status to allow pancreas to rest. Correct Initial treatment with acute pancreatitis will include an NG tube if there is vomiting and being NPO to decrease pancreatic enzyme stimulation and allow the pancreas to rest and heal. Fluid will be administered to treat or prevent shock. The pain will be treated with IV morphine because of the NPO status. Enteral feedings will only be used for the patient with severe acute pancreatitis in whom oral intake is not resumed. Antibiotic therapy is only needed with acute necrotizing pancreatitis and signs of infection.

The patient with a history of lung cancer and hepatitis C has developed liver failure and is considering liver transplantation. After the comprehensive evaluation, the nurse knows that which factor discovered may be a contraindication for liver transplantation? Has completed a college education Has been able to stop smoking cigarettes Has well-controlled type 1 diabetes mellitus The chest x-ray showed another lung cancer lesion.

The chest x-ray showed another lung cancer lesion. Correct Contraindications for liver transplant include severe extrahepatic disease, advanced hepatocellular carcinoma or other cancer, ongoing drug and/or alcohol abuse, and the inability to comprehend or comply with the rigorous post-transplant course.

When caring for a patient with liver disease, the nurse recognizes the need to prevent bleeding resulting from altered clotting factors and rupture of varices. Which nursing interventions would be appropriate to achieve this outcome (select all that apply)? Use smallest gauge needle possible when giving injections or drawing blood. Teach patient to avoid straining at stool, vigorous blowing of nose, and coughing. Advise patient to use soft-bristle toothbrush and avoid ingestion of irritating food. Apply gentle pressure for the shortest possible time period after performing venipuncture. Instruct patient to avoid aspirin and NSAIDs to prevent hemorrhage when varices are present.

Use smallest gauge needle possible when giving injections or drawing blood. Teach patient to avoid straining at stool, vigorous blowing of nose, and coughing. Advise patient to use soft-bristle toothbrush and avoid ingestion of irritating food. Instruct patient to avoid aspirin and NSAIDs to prevent hemorrhage when varices are present. Using the smallest gauge needle for injections will minimize the risk of bleeding into the tissues. Avoiding straining, nose blowing, and coughing will reduce the risk of hemorrhage at these sites. The use of a soft-bristle toothbrush and avoidance of irritating food will reduce injury to highly vascular mucous membranes. The nurse should apply gentle but prolonged pressure to venipuncture sites to minimize the risk of bleeding. Aspirin and NSAIDs should not be used in patients with liver disease because they interfere with platelet aggregation, thus increasing the risk for bleeding.

When caring for a patient with a biliary obstruction, the nurse will anticipate administering which vitamin supplements (select all that apply)? Vitamin A Vitamin D Vitamin E Vitamin K Vitamin B

Vitamin A Correct Vitamin D Correct Vitamin E Correct Vitamin K Correct Biliary obstruction prevents bile from entering the small intestine and thus prevents the absorption of fat-soluble vitamins. Vitamins A, D, E, and K are all fat-soluble and thus would need to be supplemented in a patient with biliary obstruction.

What causes a pilonidal cyst? A. Infection of a congenital tract under the skin between the buttocks B. Fistula formed from large intestine C. Cellulitis most often caused by methicillin-resistant Staphylococcus (MRSA) D. Complication of exudates from hemorrhoids

a Rationale A pilonidal sinus is a small tract under the skin between the buttocks in the sacrococcygeal area, and it is thought to be congenital. It is lined with epithelium and hair, and with moisture collection and movement of the hair, it becomes infected, forming an abscess. Reference: 1054

You determine that the goals of dietary teaching have been met when the patient with celiac disease selects from the menu A. scrambled eggs and sausage. B. buckwheat pancakes with syrup. C. oatmeal, skim milk, and orange juice. D. yogurt, strawberries, and rye toast with butter.

a Rationale Celiac disease is treated with lifelong avoidance of dietary gluten. Wheat, barley, oats, and rye products must be avoided. Although pure oats do not contain gluten, oat products can become contaminated with wheat, rye, and barley during the milling process. Gluten is also found in some medications and in many food additives, preservatives, and stabilizers. Reference: 1050

The patient with Crohn's disease has had multiple intestinal resections. Which symptom indicates that short bowel syndrome has developed? A. Steatorrhea B. Constipation C. Hypercholesteremia D. Hypercalcemia

a Rationale The predominant manifestation is diarrhea or steatorrhea. Diarrhea, not constipation, is a concern because there is decreased intestinal surface to absorb fluid and nutrients. Decreased absorption of bile salts is the issue; increased cholesterol is not related to short bowel syndrome. The risk is deficiencies of cobalamin, zinc, and calcium. Reference: 1051

Which is the best explanation of the pathophysiology that occurs in celiac or gluten-sensitive enteropathy? A. Intestinal inflammation, especially in the duodenum B. Full-thickness ulcers throughout the small and large intestines C. Lack of enzyme necessary to digest gluten D. Irregular peristalsis with heightened sensitivity

a Rationale Tissue destruction occurs as a result of chronic inflammation, activated by gluten. Damage is most severe in the duodenum, probably because it is the site of the highest concentration of gluten. Intermittent, full-thickness ulcers are characteristic of IBD. Lack of the necessary enzyme for digestion of milk is characteristic of lactase deficiency. Irregular peristalsis with enhanced sensitivity is characteristic of irritable bowel syndrome (IBS). Reference: 1050

In planning care for a patient with metastatic liver cancer, the nurse should include interventions that a. focus primarily on symptomatic and comfort measures. b. reassure the patient that chemotherapy offers a good prognosis. c. promote the patient's confidence that surgical excision of the tumor will be successful. d. provide information necessary for the patient to make decisions regarding liver transplantation. (Lewis 1042)

a Rationale: Nursing intervention for a patient with liver cancer focuses on keeping the patient as comfortable as possible. The prognosis for patients with liver cancer is poor. The cancer grows rapidly, and death may occur within 4 to 7 months as a result of hepatic encephalopathy or massive blood loss from gastrointestinal (GI) bleeding.

The nurse performs a detailed assessment of the abdomen of a patient with a possible bowel obstruction, knowing that manifestations of a obstruction in the large intestine are (select all that apply): a. persistent abdominal pain b. marked abdominal distention c. diarrhea that is loose or liquid d. colicky, severe, intermittent pain e. profuse committing that relieves abdominal pain

a & b Rationale: With lower intestinal obstructions, abdominal distention is markedly increased and pain is persistent. Onset of a large intestine obstruction is gradual, vomiting is rare, and there is usually absolute constipation, not diarrhea.

For the patient hospitalized with inflammatory bowel disease (IBD), which treatments would be used to rest the bowel (select all that apply)? a. NPO d. Sedatives b. IV fluids e. Nasogastric suction c. Bed rest f. Parenteral nutrition

a, b, e, f. With an acute exacerbation of inflammatory bowel disease (IBD), to rest the bowel the patient will be NPO, receive IV fluids and parenteral nutrition, and have nasogastric suction. Sedatives would be used to alleviate stress. Enteral nutrition will be used as soon as possible

A 20-year old patient with a history of Crohn's disease comes to the clinic with persistent diarrhea. What are characteristics of Crohn's disease (select all that apply)? a. Weight loss d. Toxic megacolon b. Rectal bleeding e. Has segmented distribution c. Abdominal pain f. Involves the entire thickness of the bowel wall

a, c, e, f. Crohn's disease may have severe weight loss, segmented distribution through the entire wall of the bowel, and crampy abdominal pain. Rectal bleeding and toxic megacolon are more often seen with ulcerative colitis.

Which is correct information regarding gastrointestinal stromal tumors (GISTs)? A. It is cancer of the villa cells that affect absorption of nutrients. B. It is cancer of cells controlling movement of food through stomach. C. They are usually detected by occult blood stool screening. D. Screening looks for elevation of the carcinogenic embryonic antigen (CEA) level.

b Rationale GISTs are a rare form of cancer that originates in cells found in the gastrointestinal tract wall. These cells (interstitial cells of Cajal) are part of the autonomic nervous system. They help control the movement of food and liquid through the stomach and intestines. About 65% are found in the stomach. Because initial symptoms are usually vague and similar to many other gastrointestinal problems, it is difficult to detect early. Colon cancer is usually detected by occult blood stool screening or by screening for elevation of the level of CEA. Reference: 1052

The patient with lactase deficiency is at risk for which nutritional deficiency? A. Vitamin K B. Calcium C. Folic acid D. Cobalamin (vitamin B12)

b Rationale Milk products are the main source of calcium in the American diet, and a lactose-intolerant person can acquire a calcium deficiency. The other nutrients are not a concern. Reference: 1051

The patient has cystic fibrosis. Which finding indicates malabsorption? A. Small stature and thin B. Greasy, foul-smelling, mushy stool C. Urine specific gravity of 1.031 D. Current jelly-like stool

b Rationale People with cystic fibrosis have malabsorption because of pancreatic dysfunction. Steatorrhea is a symptom of failure to properly digest fats. Children with cystic fibrosis typically have growth issues, but small stature and being thin does not necessarily indicate malabsorption. The urine specific gravity is slightly high and indicates dehydration, not malabsorption. Current jelly-like stool is a classic symptom of intussusception. Reference: 1049

A patient with pancreatic cancer is admitted to the hospital for evaluation of possible treatment options. The patient asks the nurse to explain the Whipple procedure that the surgeon has described. The explanation includes the information that a Whipple procedure involves a. creating a bypass around the obstruction caused by the tumor by joining the gallbladder to the jejunum. b. resection of the entire pancreas and the distal portion of the stomach, with anastomosis of the common bile duct and the stomach into the duodenum. c. removal of part of the pancreas, part of the stomach, the duodenum, and the gallbladder, with joining of the pancreatic duct, the common bile duct, and the stomach into the jejunum. d. radical removal of the pancreas, the duodenum, and the spleen, and attachment of the stomach to the jejunum, which requires oral supplementation of pancreatic digestive enzymes and insulin replacement therapy. (Lewis 1042)

c Rationale: The classic operation for pancreatic cancer is a radical pancreaticoduodenectomy, or Whipple procedure. This entails resection of the proximal pancreas (i.e., proximal pancreatectomy), the adjoining duodenum (i.e., duodenectomy), the distal portion of the stomach (i.e., partial gastrectomy), and the distal segment of the common bile duct. The pancreatic duct, common bile duct, and stomach are anastomosed to the jejunum.

The RN coordinating the care for a patient who is 2 days postoperative following an anterior- posterior resection with colostomy may delegate which interventions to the licensed practical nurse (LPN) (select all that apply)? a. Irrigate the colostomy. b. Teach ostomy and skin care. c. Assess and document stoma appearance. d. Monitor and record the volume, color, and odor of the drainage. e. Empty the ostomy bag and measure and record the amount of drainage.

d, e. The licensed practical nurse (LPN) can monitor and record observations related to the drainage and can measure and record the amount. The LPN could also monitor the skin around the stoma for breakdown. LPNs can irrigate a colostomy in a stable patient but this patient is only 2 days postoperative. The other actions are responsibilities of the RN (teaching, assessing stoma, and developing a care plan).

The nurse is caring for a postoperative patient with a colostomy. The nurse is preparing to administer a dose of famotidine (Pepcid) when the patient asks why the medication was ordered since the patient does not have a history of heartburn or gastroesophageal reflux disease (GERD). What response by the nurse would be the most appropriate? a. "This will prevent air from accumulating in the stomach, causing gas pains." b. "This will prevent the heartburn that occurs as a side effect of general anesthesia." c. "The stress of surgery is likely to cause stomach bleeding if you do not receive it." d. "This will reduce the amount of HCl in the stomach until the nasogastric tube is removed and you can eat a regular diet again."

d. "This will reduce the amount of HCl in the stomach until the nasogastric tube is removed and you can eat a regular diet again." Famotidine is an H2-receptor antagonist that inhibits gastric HCl secretion and thus minimizes damage to gastric mucosa while the patient is not eating a regular diet after surgery. Famotidine does not prevent air from accumulating in the stomach or stop the stomach from bleeding. Heartburn is not a side effect of general anesthesia.

A nursing intervention that is most appropriate to decrease post-operative edema and pain after an inguinal herniorrhaphy is: a. applying a truss to the hernia site b. allowing the patient to stand to void c. supporting the incision during coughing d. applying a scrotal support with ice bag

d. applying a scrotal support with ice bag Rationale: Scrotal edema is a painful complication after an inguinal hernia repair. Scrotal support with application of an ice bag may help relieve pain and edema.

The nurse provides discharge instructions for a 64-year-old woman with ascites and peripheral edema related to cirrhosis. Which statement, if made by the patient, indicates teaching was effective? "It is safe to take acetaminophen up to four times a day for pain." "Lactulose (Cephulac) should be taken every day to prevent constipation." "Herbs and other spices should be used to season my foods instead of salt." "I will eat foods high in potassium while taking spironolactone (Aldactone)."

"Herbs and other spices should be used to season my foods instead of salt." A low-sodium diet is indicated for the patient with ascites and edema related to cirrhosis. Table salt is a well-known source of sodium and should be avoided. Alternatives to salt to season foods include the use of seasonings such as garlic, parsley, onion, lemon juice, and spices. Pain medications such as acetaminophen, aspirin, and ibuprofen should be avoided as these medications may be toxic to the liver. The patient should avoid potentially hepatotoxic over-the-counter drugs (e.g., acetaminophen) because the diseased liver is unable to metabolize these drugs. Spironolactone is a potassium-sparing diuretic. Lactulose results in the acidification of feces in bowel and trapping of ammonia, causing its elimination in feces.

The patient with cirrhosis is being taught self-care. Which statement indicates the patient needs more teaching? "If I notice a fast heart rate or irregular beats, this is normal for cirrhosis." "I need to take good care of my belly and ankle skin where it is swollen." "A scrotal support may be more comfortable when I have scrotal edema." "I can use pillows to support my head to help me breathe when I am in bed."

"If I notice a fast heart rate or irregular beats, this is normal for cirrhosis." Correct If the patient with cirrhosis experiences a fast or irregular heart rate, it may be indicative of hypokalemia and should be reported to the health care provider, as this is not normal for cirrhosis. Edematous tissue is subject to breakdown and needs meticulous skin care. Pillows and a semi-Fowler's or Fowler's position will increase respiratory efficiency. A scrotal support may improve comfort if there is scrotal edema.

The nurse instructs a 50-year-old woman about cholestyramine to reduce pruritis caused by gallbladder disease. Which statement by the patient to the nurse indicates she understands the instructions? "This medication will help me digest fats and fat-soluble vitamins." "I will apply the medicated lotion sparingly to the areas where I itch." "The medication is a powder and needs to be mixed with milk or juice." "I should take this medication on an empty stomach at the same time each day."

"The medication is a powder and needs to be mixed with milk or juice." For treatment of pruritus, cholestyramine may provide relief. This is a resin that binds bile salts in the intestine, increasing their excretion in the feces. Cholestyramine is in powder form and should be mixed with milk or juice before oral administration.

After a transurethral prostatectomy a patient returns to his room with a triple-lumen indwelling catheter and continuous bladder irrigation. The irrigation is normal saline at 150 mL/hr. The nurse empties the drainage bag for a total of 2520 mL after an 8-hour period. How much of the total is urine output?

1320

A patient with a history of peptic ulcer disease has presented to the emergency department reporting severe abdominal pain and has a rigid, boardlike abdomen that prompts the health care team to suspect a perforated ulcer. What intervention should the nurse anticipate? a) Providing IV fluids and inserting a nasogastric (NG) tube b) Administering oral bicarbonate and testing the patient's gastric pH level c) Performing a fecal occult blood test and administering IV calcium gluconate d) Starting parenteral nutrition and placing the patient in a high-Fowler's position

A A perforated peptic ulcer requires IV replacement of fluid losses and continued gastric aspiration by NG tube. Nothing is given by mouth, and gastric pH testing is not a priority. Calcium gluconate is not a medication directly relevant to the patient's suspected diagnosis, and parenteral nutrition is not a priority in the short term.

The appropriate collaborative therapy for the patient with acute diarrhea caused by a viral infection is to A. increase fluid intake. B. administer an antibiotic. C. administer antimotility drugs. D. quarantine the patient to prevent spread of the virus.

A Rationale Acute diarrhea from infectious causes (e.g., a virus) is usually self-limiting. The major concerns are preventing transmission, fluid and electrolyte replacement, and resolution of the diarrhea. Antidiarrheal agents are contraindicated in the treatment of infectious diarrhea because they potentially prolong exposure to the infectious organism. Antibiotics are rarely used to treat acute diarrhea. To prevent transmission of diarrhea caused by a virus, hand washing should be performed before and after contact with the patient and when body fluids of any kind are handled. Flush vomitus and stool in the toilet, and wash contaminated clothing immediately with soap and hot water. Reference: 1008-1009

Which factor in a patient's history would make the patient at risk for constipation? A. Takes codeine daily for chronic back problem B. Runs a marathon yearly C. Takes an angiotensin-converting enzyme (ACE) inhibitor for hypertension D. Follows a vegan diet

A Rationale Common causes of constipation include insufficient dietary fiber, inadequate fluid intake, decreased physical activity, ignoring the defecation urge, and medications such as opioids. Constipation from opioids is a side effect that does not decrease with time. Training for and running a marathon would help regularity. An ACE inhibitor does not cause constipation. A vegan diet tends to have increased bulk, but it is not a factor as long as the patient has adequate fluids and fiber. Reference: 1012

The patient has a history of chronic diarrhea of three or more liquid stools for the past 4 weeks. What potential imbalance is of greatest concern? A. Hypokalemia B. Hyponatremia C. Metabolic alkalosis D. Thrombocytopenia

A Rationale Diarrhea can cause electrolyte imbalances, particularly hypokalemia. Because dehydration can result, the patient is at risk for hypernatremia. Metabolic acidosis can develop with severe diarrhea. Low platelet levels are not necessarily associated with diarrhea. Reference: 1008

You are caring for a patient in the emergency department with complaints of acute abdominal pain, nausea, and vomiting. When you palpate the patient's left lower abdominal quadrant, the patient complains of pain in the right lower quadrant. You will document this as which diagnostic sign of appendicitis? A. Rovsing sign B. Referred pain C. Chvostek's sign D. Rebound tenderness

A Rationale In patients with suspected appendicitis, the Rovsing sign may be elicited by palpation of the left lower quadrant, causing pain to be felt in the right lower quadrant. Reference: 1020

You are conducting discharge teaching for a patient with metastatic lung cancer who was admitted with a bowel impaction. Which instruction would be most helpful to prevent further episodes of constipation? A. Maintain a high intake of fluid and fiber in the diet. B. Reduce intake of medications causing constipation. C. Eat several small meals per day to maintain bowel motility. D. Sit upright during meals to increase bowel motility by gravity.

A Rationale Increased fluid intake and a high-fiber diet reduce the incidence of constipation caused by immobility, medications, and other factors. Fluid and fiber provide bulk that in turn increases peristalsis and bowel motility. Reference: 1013

The elderly patient was informed that outpouches were found in the descending colon during the screening colonoscopy. The patient asks you what this finding means. What is the best explanation? A. Most people get these outpouchings as they age. B. These findings respond well to treatment with sulfa antibiotics. C. It is a precursor to colon cancer, and routine screening is essential. D. They contribute to malabsorption of cobalamin (vitamin B12) and fat.

A Rationale It is believed that 65% of people have the saccular dilations or outpouchings of the mucosa by the time they are 85 years old. It is believed to be from high intraluminal pressure on weakened areas of the bowel wall from inadequate dietary fiber. It is typically asymptomatic and not a concern unless inflamed or diverticulitis develops. Antibiotics are not required unless they are inflamed (diverticulitis). Polyps are a precursor to colon cancer; diverticula are not precancerous. Malabsorption of cobalamin (vitamin B12) and fat is seen with disease of the terminal ileum; the sigmoid colon mainly absorbs water. Reference: 1046

The patient is admitted to the hospital with a severe exacerbation of ulcerative colitis. What finding is most important for you to act on? A. Blood urea nitrogen (BUN): 50 mg/dL B. Hemoglobin (Hb): 12 g/dL C. White blood cells (WBC): 11,000/μL D. Sodium (Na+): 148 mEq/L

A Rationale Patients with severe ulcerative colitis frequently have bloody diarrhea. Dehydration is present as evidenced by the high BUN. This must be treated first before the mild anemia and mild inflammation are addressed. Hypernatremia is usually not treated until the sodium level is above 150 mEq/L, and it usually responds to fluid replacement. Reference: 1024

A 60-year-old man reports to the clinic nurse that he feels tired. The CBC count shows a hemoglobin level of 9 g/dL. What action should you advocate? A. Fecal occult blood test (FOBT) B. Consuming foods high in iron, such as red meat C. Assessing the patient's stress and sleep schedule D. Encouraging over-the-counter iron supplement

A Rationale Symptoms of colorectal cancer are nonspecific or absent until the disease is advanced. Anemia can be a symptom in an older adult. The cause of the anemia, including screening for colorectal cancer, needs to be done before advocating other treatment. Cancer assessment takes priority over lifestyle assessment. Reference: 1035-1036

You explain to the patient undergoing ostomy surgery that the procedure that maintains the most normal functioning of the bowel is A. A sigmoid colostomy B. A transverse colostomy C. A descending colostomy D. An ascending colostomy

A Rationale The more distal the ostomy, the more the intestinal contents resemble feces that are eliminated from an intact colon and rectum. Output from a sigmoid colostomy resembles normally formed stool, and some patients are able to regulate emptying time so they do not need to wear a collection bag. Reference: 1039

The nurse is conducting discharge teaching for a patient with metastatic lung cancer who was admitted with a bowel impaction. Which instructions would be most helpful to prevent further episodes of constipation? A. Maintain a high intake of fluid and fiber in the diet. B. Reduce intake of medications causing constipation. C. Eat several small meals per day to maintain bowel motility. D. Sit upright during meals to increase bowel motility by gravity.

A Increased fluid intake and a high-fiber diet reduce the incidence of constipation caused by immobility, medications, and other factors. Fluid and fiber provide bulk that in turn increases peristalsis and bowel motility. Analgesics taken for lung cancer probably cannot be reduced. Other medications may decrease constipation, but it is best to avoid laxatives. Eating several small meals per day and position do not facilitate bowel motility. Defecation is easiest when the person sits on the commode with the knees higher than the hips.

A stroke patient who primarily uses a wheelchair for mobility has diarrhea with fecal incontinence. What should the nurse assess first? A. Fecal impaction B. Perineal hygiene C. Dietary fiber intake D. Antidiarrheal agent use

A Patients with limited mobility are at risk for fecal impactions due to constipation that may lead to liquid stool leaking around the hardened impacted feces, so assessing for fecal impaction is the priority. Perineal hygiene can be assessed at the same time. Assessing the dietary fiber and fluid intake and antidiarrheal agent use will be assessed and considered next.

The nurse is preparing to insert a nasogastric (NG) tube into a 68-year-old female patient who is nauseated and vomiting. She has an abdominal mass and suspected small intestinal obstruction. The patient asks the nurse why this procedure is necessary. What response by the nurse is most appropriate? A. "The tube will help to drain the stomach contents and prevent further vomiting." B. "The tube will push past the area that is blocked and thus help to stop the vomiting." C. "The tube is just a standard procedure before many types of surgery to the abdomen." D. "The tube will let us measure your stomach contents so that we can plan what type of IV fluid replacement would be best."

A The NG tube is used to decompress the stomach by draining stomach contents and thereby prevent further vomiting. The NG tube will not push past the blocked area. Potential surgery is not currently indicated. The location of the obstruction will determine the type of fluid to use, not measure the amount of stomach contents

A patient with type 2 diabetes and cirrhosis asks the nurse if it would be okay to take silymarin (milk thistle) to help minimize liver damage. The nurse responds based on knowledge that: A. Milk thistle may affect liver enzymes and thus alter drug metabolism. B. Milk thistle is generally safe in recommended doses for up to 10 years. C. is unclear scientific evidence for the use of milk thistle in treating cirrhosis. D. Milk thistle may elevate the serum glucose levels and is thus contraindicated in diabetes.

A) There is good scientific evidence that there is no real benefit from using milk thistle to protect the liver cells from toxic damage in the treatment of cirrhosis. Milk thistle does affect liver enzymes and thus could alter drug metabolism. Therefore patients will need to be monitored for drug interactions. It is noted to be safe for up to 6 years, not 10 years, and it may lower, not elevate, blood glucose levels.

When caring for a patient with liver disease, the nurse recognizes the need to prevent bleeding resulting from altered clotting factors and rupture of varices. Which of the following nursing interventions would be appropriate to achieve this outcome (select all that apply)? A. Use smallest gauge possible when giving injections or drawing blood. B. Teach patient to avoid straining at stool, vigorous blowing of nose, and coughing. C. Advise patient to use soft-bristle toothbrush and avoid ingestion of irritating food. D. Apply gentle pressure for the shortest possible time period after performing venipuncture. E. Instruct patient to avoid aspirin and NSAIDs to prevent hemorrhage when varices are present.

A,B,C,E) Using the smallest gauge needle for injections will minimize the risk of bleeding into the tissues. Avoiding straining, nose blowing, and coughing will reduce the risk of hemorrhage at these sites. The use of a soft-bristle toothbrush and avoidance of irritating food will reduce injury to highly vascular mucous membranes. The nurse should apply gentle but prolonged pressure to venipuncture sites to minimize the risk of bleeding. Aspirin and NSAIDs should not be used in patients with liver disease because they interfere with platelet aggregation, thus increasing the risk for bleeding.

The patient with cirrhosis has an increased abdominal girth from ascites. The nurse should know that this fluid gathers in the abdomen for which reasons (select all that apply)? A) There is decreased colloid oncotic pressure from the liver's inability to synthesize albumin. B) Hyperaldosteronism related to damaged hepatocytes increases sodium and fluid retention. C) Portal hypertension pushes proteins from the blood vessels, causing leaking into the peritoneal cavity. D) Osmoreceptors in the hypothalamus stimulate thirst, which causes the stimulation to take in fluids orally. E) Overactivity of the enlarged spleen results in increased removal of blood cells from the circulation, which decreases the vascular pressure.

A,B.C) The ascites related to cirrhosis are caused by decreased colloid oncotic pressure from the lack of albumin from liver inability to synthesize it and the portal hypertension that shifts the protein from the blood vessels to the peritoneal cavity, and hyperaldosteronism which increases sodium and fluid retention. The intake of fluids orally and the removal of blood cells by the spleen do not directly contribute to ascites.

The nurse admitting a patient with acute diverticulitis explains that the initial plan of care is to a. administer IV fluids. b. give stool softeners and enemas. c. order a diet high in fiber and fluids. d. prepare the patient for colonoscopy.

ANS: A A patient with acute diverticulitis will be NPO and given parenteral fluids. A diet high in fiber and fluids will be implemented before discharge. Bulk-forming laxatives, rather than stool softeners, are usually given, and these will be implemented later in the hospitalization. The patient with acute diverticulitis will not have enemas or a colonoscopy because of the risk for perforation and peritonitis.

A female patient being admitted with pneumonia has a history of neurogenic bladder as a result of a spinal cord injury. Which action will the nurse plan to take first? a. Ask about the usual urinary pattern and any measures used for bladder control. b. Assist the patient to the toilet at scheduled times to help ensure bladder emptying. c. Check the patient for urinary incontinence every 2 hours to maintain skin integrity. d. Use intermittent catheterization on a regular schedule to avoid the risk of infection.

ANS: A Before planning any interventions, the nurse should complete the assessment and determine the patient's normal bladder pattern and the usual measures used by the patient at home. All the other responses may be appropriate, but until the assessment is complete, an individualized plan for the patient cannot be developed

A female patient is awaiting surgery for acute peritonitis. Which action will the nurse include in the plan of care? a. Position patient with the knees flexed. b. Avoid use of opioids or sedative drugs. c. Offer frequent small sips of clear liquids. d. Assist patient to breathe deeply and cough.

ANS: A There is less peritoneal irritation with the knees flexed, which will help decrease pain. Opioids and sedatives are typically given to control pain and anxiety. Preoperative patients with peritonitis are given IV fluids for hydration. Deep breathing and coughing will increase the patient's discomfort

Which finding for a patient who has been taking orlistat (Xenical) is most important to report to the health care provider? a. The patient frequently has liquid stools. b. The patient is pale and has many bruises. c. The patient complains of bloating after meals. d. The patient is experiencing a weight loss plateau.

ANS: B Because orlistat blocks the absorption of fat-soluble vitamins, the patient may not be receiving an adequate amount of vitamin K, resulting in a decrease in clotting factors. Abdominal bloating and liquid stools are common side effects of orlistat and indicate that the nurse should remind the patient that fat in the diet may increase these side effects. Weight loss plateaus are normal during weight reduction

A patient in the emergency department has just been diagnosed with peritonitis caused by a ruptured diverticulum. Which prescribed intervention will the nurse implement first? a. Insert a urinary catheter to drainage. b. Infuse metronidazole (Flagyl) 500 mg IV. c. Send the patient for a computerized tomography scan. d. Place a nasogastric (NG) tube to intermittent low suction.

ANS: B Because peritonitis can be fatal if treatment is delayed, the initial action should be to start antibiotic therapy (after any ordered cultures are obtained). The other actions can be done after antibiotic therapy is initiated

A 36-year-old female patient is receiving treatment for chronic hepatitis C with pegylated interferon (PEG-Intron, Pegasys), ribavirin (Rebetol), and telaprevir (Incivek). Which finding is most important to communicate to the health care provider? a. Weight loss of 2 lb (1 kg) b. Positive urine pregnancy test c. Hemoglobin level of 10.4 g/dL d. Complaints of nausea and anorexia

ANS: B Because ribavirin is teratogenic, the medication will need to be discontinued immediately. Anemia, weight loss, and nausea are common adverse effects of the prescribed regimen and may require actions such as patient teaching, but they would not require immediate cessation of the therapy.

A 49-year-old man has been admitted with hypotension and dehydration after 3 days of nausea and vomiting. Which order from the health care provider will the nurse implement first? a. Insert a nasogastric (NG) tube. b. Infuse normal saline at 250 mL/hr. c. Administer IV ondansetron (Zofran). d. Provide oral care with moistened swabs.

ANS: B Because the patient has severe dehydration, rehydration with IV fluids is the priority. The other orders should be accomplished as quickly as possible after the IV fluids are initiated

Which nursing action is appropriate when coaching obese adults enrolled in a behavior modification program? a. Having the adults write down the caloric intake of each meal b. Asking the adults about situations that tend to increase appetite c. Suggesting that the adults plan rewards, such as sugarless candy, for achieving their goals d. Encouraging the adults to eat small amounts frequently rather than having scheduled meals

ANS: B Behavior modification programs focus on how and when the person eats and de-emphasize aspects such as calorie counting. Nonfood rewards are recommended for achievement of weight-loss goals. Patients are often taught to restrict eating to designated meals when using behavior modification

Which action will the nurse include in the plan of care for a patient who has been diagnosed with chronic hepatitis B? a. Advise limiting alcohol intake to 1 drink daily. b. Schedule for liver cancer screening every 6 months. c. Initiate administration of the hepatitis C vaccine series. d. Monitor anti-hepatitis B surface antigen (anti-HBs) levels annually.

ANS: B Patients with chronic hepatitis are at higher risk for development of liver cancer, and should be screened for liver cancer every 6 to 12 months. Patients with chronic hepatitis are advised to completely avoid alcohol. There is no hepatitis C vaccine. Because anti-HBs is present whenever there has been a past hepatitis B infection or vaccination, there is no need to regularly monitor for this antibody.

A 25-year-old male patient calls the clinic complaining of diarrhea for 24 hours. Which action should the nurse take first? a. Inform the patient that laboratory testing of blood and stools will be necessary. b. Ask the patient to describe the character of the stools and any associated symptoms. c. Suggest that the patient drink clear liquid fluids with electrolytes, such as Gatorade or Pedialyte. d. Advise the patient to use over-the-counter loperamide (Imodium) to slow gastrointestinal (GI) motility.

ANS: B The initial response by the nurse should be further assessment of the patient. The other responses may be appropriate, depending on what is learned in the assessment

The nurse is administering IV fluid boluses and nasogastric irrigation to a patient with acute gastrointestinal (GI) bleeding. Which assessment finding is most important for the nurse to communicate to the health care provider? a. The bowel sounds are hyperactive in all four quadrants. b. The patient's lungs have crackles audible to the midchest. c. The nasogastric (NG) suction is returning coffee-ground material. d. The patient's blood pressure (BP) has increased to 142/84 mm Hg.

ANS: B The patient's lung sounds indicate that pulmonary edema may be developing as a result of the rapid infusion of IV fluid and that the fluid infusion rate should be slowed. The return of coffee-ground material in an NG tube is expected for a patient with upper GI bleeding. The BP is slightly elevated but would not be an indication to contact the health care provider immediately. Hyperactive bowel sounds are common when a patient has GI bleeding

A patient with a peptic ulcer who has a nasogastric (NG) tube develops sudden, severe upper abdominal pain, diaphoresis, and a very firm abdomen. Which action should the nurse take next? a. Irrigate the NG tube. b. Obtain the vital signs. c. Listen for bowel sounds. d. Give the ordered antacid.

ANS: B The patient's symptoms suggest acute perforation, and the nurse should assess for signs of hypovolemic shock. Irrigation of the NG tube, administration of antacids, or both would be contraindicated because any material in the stomach will increase the spillage into the peritoneal cavity. The nurse should assess the bowel sounds, but this is not the first action that should be taken.

A 55-year-old patient admitted with an abrupt onset of jaundice and nausea has abnormal liver function studies but serologic testing is negative for viral causes of hepatitis. Which question by the nurse is most appropriate? a. "Is there any history of IV drug use?" b. "Do you use any over-the-counter drugs?" c. "Are you taking corticosteroids for any reason?" d. "Have you recently traveled to a foreign country?"

ANS: B The patient's symptoms, lack of antibodies for hepatitis, and the abrupt onset of symptoms suggest toxic hepatitis, which can be caused by commonly used over-the-counter drugs such as acetaminophen (Tylenol). Travel to a foreign country and a history of IV drug use are risk factors for viral hepatitis. Corticosteroid use does not cause the symptoms listed

Administration of hepatitis B vaccine to a healthy 18-year-old patient has been effective when a specimen of the patient's blood reveals a. HBsAg. b. anti-HBs. c. anti-HBc IgG. d. anti-HBc IgM.

ANS: B The presence of surface antibody to HBV (anti-HBs) is a marker of a positive response to the vaccine. The other laboratory values indicate current infection with HBV

To detect possible complications in a patient with severe cirrhosis who has bleeding esophageal varices, it is most important for the nurse to monitor a. bilirubin levels. b. ammonia levels. c. potassium levels. d. prothrombin time.

ANS: B The protein in the blood in the gastrointestinal (GI) tract will be absorbed and may result in an increase in the ammonia level because the liver cannot metabolize protein very well. The prothrombin time, bilirubin, and potassium levels should also be monitored, but they will not be affected by the bleeding episode

A 68-year-old patient with a bleeding duodenal ulcer has a nasogastric (NG) tube in place, and the health care provider orders 30 mL of aluminum hydroxide/magnesium hydroxide (Maalox) to be instilled through the tube every hour. To evaluate the effectiveness of this treatment, the nurse a. monitors arterial blood gas values daily. b. periodically aspirates and tests gastric pH. c. checks each stool for the presence of occult blood. d. measures the volume of residual stomach contents.

ANS: B The purpose for antacids is to increase gastric pH. Checking gastric pH is the most direct way of evaluating the effectiveness of the medication. Arterial blood gases may change slightly, but this does not directly reflect the effect of antacids on gastric pH. Because the patient has upper gastrointestinal (GI) bleeding, occult blood in the stools will appear even after the acute bleeding has stopped. The amount of residual stomach contents is not a reflection of resolution of bleeding or of gastric pH

How many grams of protein will the nurse recommend to meet the minimum daily requirement for a patient who weighs 145 pounds (66 kg)? a. 36 b. 53 c. 75 d. 98

ANS: B The recommended daily protein intake is 0.8 to 1 g/kg of body weight, which for this patient is 66 kg × 0.8 g = 52.8 or 53 g/day.

When using a soft, silicone nasogastric tube for enteral feedings, the nurse will need to a. avoid giving medications through the feeding tube. b. flush the tubing after checking for residual volumes. c. administer continuous feedings using an infusion pump. d. replace the tube every 3 to 5 days to avoid mucosal damage.

ANS: B The soft silicone feeding tubes are small in diameter and can easily become clogged unless they are flushed after the nurse checks the residual volume. Either intermittent or continuous feedings can be given. The tubes are less likely to cause mucosal damage than the stiffer polyvinyl chloride tubes used for nasogastric suction and do not need to be replaced at certain intervals. Medications can be given through these tubes, but flushing after medication administration is important to avoid clogging

Which adult will the nurse plan to teach about risks associated with obesity? a. Man who has a BMI of 18 kg/m2 b. Man with a 42 in waist and 44 in hips c. Woman who has a body mass index (BMI) of 24 kg/m2 d. Woman with a waist circumference of 34 inches (86 cm)

ANS: B The waist-to-hip ratio for this patient is 0.95, which exceeds the recommended level of <0.80. A patient with a BMI of 18 kg/m2 is considered underweight. A BMI of 24 kg/m2 is normal. Health risks associated with obesity increase in women with a waist circumference larger than 35 in (89 cm) and men with a waist circumference larger than 40 in (102 cm).

A male patient in the clinic provides a urine sample that is red-orange in color. Which action should the nurse take first? a. Notify the patient's health care provider. b. Teach correct midstream urine collection. c. Ask the patient about current medications. d. Question the patient about urinary tract infection (UTI) risk factors.

ANS: C A red-orange color in the urine is normal with some over-the-counter (OTC) medications such as phenazopyridine (Pyridium). The color would not be expected with urinary tract infection, is not a sign that poor technique was used in obtaining the specimen, and does not need to be communicated to the health care provider until further assessment is done

A 68-year-old male patient with a stroke is unconscious and unresponsive to stimuli. After learning that the patient has a history of gastroesophageal reflux disease (GERD), the nurse will plan to do frequent assessments of the patient's a. apical pulse. b. bowel sounds. c. breath sounds. d. abdominal girth.

ANS: C Because GERD may cause aspiration, the unconscious patient is at risk for developing aspiration pneumonia. Bowel sounds, abdominal girth, and apical pulse will not be affected by the patient's stroke or GERD and do not require more frequent monitoring than the routine. DIF: Cognitive Level: Apply (application) REF: 932 TOP: Nursing Process: Assessment MSC: NCLEX: Physiological Integrity

A 53-year-old male patient with deep partial-thickness burns from a chemical spill in the workplace experiences severe pain followed by nausea during dressing changes. Which action will be most useful in decreasing the patient's nausea? a. Keep the patient NPO for 2 hours before and after dressing changes. b. Avoid performing dressing changes close to the patient's mealtimes. c. Administer the prescribed morphine sulfate before dressing changes. d. Give the ordered prochlorperazine (Compazine) before dressing changes.

ANS: C Because the patient's nausea is associated with severe pain, it is likely that it is precipitated by stress and pain. The best treatment will be to provide adequate pain medication before dressing changes. The nurse should avoid doing painful procedures close to mealtimes, but nausea/vomiting that occur at other times also should be addressed. Keeping the patient NPO does not address the reason for the nausea and vomiting and will have an adverse effect on the patient's nutrition. Administration of antiemetics is not the best choice for a patient with nausea caused by pain

A 49-year-old female patient with cirrhosis and esophageal varices has a new prescription for propranolol (Inderal). Which finding is the best indicator that the medication has been effective? a. The patient reports no chest pain. b. Blood pressure is 140/90 mm Hg. c. Stools test negative for occult blood. d. The apical pulse rate is 68 beats/minute.

ANS: C Because the purpose of b-blocker therapy for patients with esophageal varices is to decrease the risk for bleeding from esophageal varices, the best indicator of the effectiveness for propranolol is the lack of blood in the stools. Although propranolol is used to treat hypertension, angina, and tachycardia, the purpose for use in this patient is to decrease the risk for bleeding from esophageal varices

A patient returned from a laparoscopic Nissen fundoplication for hiatal hernia 4 hours ago. Which assessment finding is most important for the nurse to address immediately? a. The patient is experiencing intermittent waves of nausea. b. The patient complains of 7/10 (0 to 10 scale) abdominal pain. c. The patient has absent breath sounds in the left anterior chest. d. The patient has hypoactive bowel sounds in all four quadrants.

ANS: C Decreased breath sounds on one side may indicate a pneumothorax, which requires rapid diagnosis and treatment. The nausea and abdominal pain should also be addressed but they are not as high priority as the patient's respiratory status. The patient's decreased bowel sounds are expected after surgery and require ongoing monitoring but no other action. DIF: Cognitive Level: Apply (application) REF: 935 OBJ: Special Questions: Prioritization TOP: Nursing Process: Assessment MSC: NCLEX: Physiological Integrity

Which activity in the care of a 48-year-old female patient with a new colostomy could the nurse delegate to unlicensed assistive personnel (UAP)? a. Document the appearance of the stoma. b. Place a pouching system over the ostomy. c. Drain and measure the output from the ostomy. d. Check the skin around the stoma for breakdown.

ANS: C Draining and measuring the output from the ostomy is included in UAP education and scope of practice. The other actions should be implemented by LPNs or RNs.

Which diet choice by the patient with an acute exacerbation of inflammatory bowel disease (IBD) indicates a need for more teaching? a. Scrambled eggs b. White toast and jam c. Oatmeal with cream d. Pancakes with syrup

ANS: C During acute exacerbations of IBD, the patient should avoid high-fiber foods such as whole grains. High-fat foods also may cause diarrhea in some patients. The other choices are low residue and would be appropriate for this patient.

A patient with a body mass index (BMI) of 17 kg/m2 and a low albumin level is being admitted by the nurse. Which assessment finding will the nurse expect to find? a. Restlessness b. Hypertension c. Pitting edema d. Food allergies

ANS: C Edema occurs when serum albumin levels and plasma oncotic pressure decrease. The blood pressure and level of consciousness are not directly affected by malnutrition. Food allergies are not an indicator of nutritional status.

The nurse is caring for a 36-year-old patient with pancreatic cancer. Which nursing action is the highest priority? a. Offer psychologic support for depression. b. Offer high-calorie, high-protein dietary choices. c. Administer prescribed opioids to relieve pain as needed. d. Teach about the need to avoid scratching any pruritic areas.

ANS: C Effective pain management will be necessary in order for the patient to improve nutrition, be receptive to teaching, or manage anxiety or depression

The nurse will plan to teach the patient with newly diagnosed achalasia that a. a liquid or blenderized diet will be necessary. b. drinking fluids with meals should be avoided. c. endoscopic procedures may be used for treatment. d. lying down and resting after meals is recommended.

ANS: C Endoscopic and laparoscopic procedures are the most effective therapy for improving symptoms caused by achalasia. Patients are advised to drink fluid with meals. Keeping the head elevated after eating will improve esophageal emptying. A semisoft diet is recommended to improve esophageal emptying.

After the nurse has completed teaching a patient with newly diagnosed eosinophilic esophagitis about the management of the disease, which patient action indicates that the teaching has been effective? a. Patient orders nonfat milk for each meal. b. Patient uses the prescribed corticosteroid inhaler. c. Patient schedules an appointment for allergy testing. d. Patient takes ibuprofen (Advil) to control throat pain.

ANS: C Eosinophilic esophagitis is frequently associated with environmental allergens, so allergy testing is used to determine possible triggers. Corticosteroid therapy may be prescribed, but the medication will be swallowed, not inhaled. Milk is a frequent trigger for attacks. NSAIDs are not used for eosinophilic esophagitis

Which response by the nurse best explains the purpose of ranitidine (Zantac) for a patient admitted with bleeding esophageal varices? a. The medication will reduce the risk for aspiration. b. The medication will inhibit development of gastric ulcers. c. The medication will prevent irritation of the enlarged veins. d. The medication will decrease nausea and improve the appetite.

ANS: C Esophageal varices are dilated submucosal veins. The therapeutic action of H2-receptor blockers in patients with esophageal varices is to prevent irritation and bleeding from the varices caused by reflux of acid gastric contents. Although ranitidine does decrease the risk for peptic ulcers, reduce nausea, and help prevent aspiration pneumonia, these are not the primary purposes for H2-receptor blockade in this patient

While interviewing a 30-year-old man, the nurse learns that the patient has a family history of familial adenomatous polyposis (FAP). The nurse will plan to assess the patient's knowledge about a. preventing noninfectious hepatitis. b. treating inflammatory bowel disease. c. risk for developing colorectal cancer. d. using antacids and proton pump inhibitors.

ANS: C Familial adenomatous polyposis is a genetic condition that greatly increases the risk for colorectal cancer. Noninfectious hepatitis, use of medications that treat increased gastric pH, and inflammatory bowel disease are not related to FAP.

When taking the blood pressure (BP) on the right arm of a patient with severe acute pancreatitis, the nurse notices carpal spasms of the patient's right hand. Which action should the nurse take next? a. Ask the patient about any arm pain. b. Retake the patient's blood pressure. c. Check the calcium level in the chart. d. Notify the health care provider immediately.

ANS: C The patient with acute pancreatitis is at risk for hypocalcemia, and the assessment data indicate a positive Trousseau's sign. The health care provider should be notified after the nurse checks the patient's calcium level. There is no indication that the patient needs to have the BP rechecked or that there is any arm pain

Which patient should the nurse assess first after receiving change-of-shift report? a. A patient with nausea who has a dose of metoclopramide (Reglan) due b. A patient who is crying after receiving a diagnosis of esophageal cancer c. A patient with esophageal varices who has a blood pressure of 92/58 mm Hg d. A patient admitted yesterday with gastrointestinal (GI) bleeding who has melena

ANS: C The patient's history and blood pressure indicate possible hemodynamic instability caused by GI bleeding. The data about the other patients do not indicate acutely life-threatening complications

After receiving change-of-shift report, which patient should the nurse assess first? a. A patient who was admitted yesterday with gastrointestinal (GI) bleeding and has melena b. A patient who is crying after receiving a diagnosis of esophageal cancer c. A patient with esophageal varices who has a blood pressure of 96/54 mm Hg d. A patient with nausea who has a dose of metoclopramide (Reglan) scheduled

ANS: C The patient's history and blood pressure indicate possible hemodynamic instability caused by GI bleeding. The data about the other patients do not indicate acutely life-threatening complications.

A patient with acute gastrointestinal (GI) bleeding is receiving normal saline IV at a rate of 500 mL/hr. Which assessment finding obtained by the nurse is most important to communicate immediately to the health care provider? a. The patient's blood pressure (BP) has increased to 142/94 mm Hg. b. The nasogastric (NG) suction is returning coffee-ground material. c. The patient's lungs have crackles audible to the midline. d. The bowel sounds are very hyperactive in all four quadrants.

ANS: C The patient's lung sounds indicate that pulmonary edema may be developing as a result of the rapid infusion of IV fluid and that the fluid infusion rate should be slowed. The return of coffee-ground material in an NG tube is expected for a patient with upper GI bleeding. The BP is slightly elevated but would not be an indication to contact the health care provider immediately. Hyperactive bowel sounds are common when a patient has GI bleeding.

You would increase the comfort of the patient with appendicitis by A. having the patient lie prone. B. flexing the patient's right knee. C. sitting the patient upright in a chair. D. turning the patient onto his or her left side.

B Rationale The patient with appendicitis usually prefers to lie still, often with the right leg flexed to decrease pain. Reference: 1020

The client has an elevated blood urea nitrogen (BUN) level and an increased ratio of blood urea nitrogen to creatinine. What is the nurse's interpretation of these laboratory results? A. The client probably has a urinary tract infection. B. The client may be overhydrated. C. The kidney may be hypoperfused. D. The kidney may be damaged.

ANS: C When dehydration or renal hypoperfusion exist, the BUN level rises more rapidly than the serum creatinine level, causing the ratio to be increased, even when no renal dysfunction is present.

The nurse should recognize that the liver performs which functions (select all that apply) a. Bile storage b. Detoxification c. Protein metabolism d. Steroid metabolism e. Red blood cell (RBC) destruction

Correct answer: b, c, d Rationale: The liver performs multiple major functions that aid in the maintenance of homeostasis. These include metabolism of proteins and steroids as well as detoxification of drugs and metabolic waste products. The Kupffer cells of the liver participate in the breakdown of old RBCs. The liver produces bile, but storage occurs in the gall bladder.

In preparing to care for the obese patient with cancer, what physiologic problems is this patient at a greater risk for having (select all that apply)? a. Tinnitus b. Fractures c. Sleep apnea d. Trousseau's sign e. Type 2 diabetes mellitus f. Gastroesophageal reflux disease (GERD)

Correct answers: c, e, f Rationale: Obese patients are at a higher risk for cancer, sleep apnea and sleep deprivation, type 2 diabetes mellitus, gastroesophageal reflux disease (GERD), nonalcoholic steatohepatits, osteoarthritis, and cardiovascular problems. The other options are not related to obesity.

After the nurse teaches a patient with gastroesophageal reflux disease (GERD) about recommended dietary modifications, which diet choice for a snack 2 hours before bedtime indicates that the teaching has been effective? a. Chocolate pudding b. Glass of low-fat milk c. Peanut butter sandwich d. Cherry gelatin and fruit

ANS: D Gelatin and fruit are low fat and will not decrease lower esophageal sphincter (LES) pressure. Foods like chocolate are avoided because they lower LES pressure. Milk products increase gastric acid secretion. High-fat foods such as peanut butter decrease both gastric emptying and LES pressure.

A patient who requires daily use of a nonsteroidal anti-inflammatory drug (NSAID) for management of severe rheumatoid arthritis has recently developed melena. The nurse will anticipate teaching the patient about a. substitution of acetaminophen (Tylenol) for the NSAID. b. use of enteric-coated NSAIDs to reduce gastric irritation. c. reasons for using corticosteroids to treat the rheumatoid arthritis. d. the benefits of misoprostol (Cytotec) in protecting the gastrointestinal (GI) mucosa.

ANS: D Misoprostol, a prostaglandin analog, reduces acid secretion and incidence of upper GI bleeding associated with NSAID use. Enteric coating of NSAIDs does not reduce the risk for GI bleeding. Corticosteroids increase the risk for ulcer development and will not be substituted for NSAIDs for this patient. Acetaminophen will not be effective in treating the patient's rheumatoid arthritis.

A 62-year-old man patient who requires daily use of a nonsteroidal antiinflammatory drug (NSAID) for the management of severe rheumatoid arthritis has recently developed melena. The nurse will anticipate teaching the patient about a. substitution of acetaminophen (Tylenol) for the NSAID. b. use of enteric-coated NSAIDs to reduce gastric irritation. c. reasons for using corticosteroids to treat the rheumatoid arthritis. d. misoprostol (Cytotec) to protect the gastrointestinal (GI) mucosa.

ANS: D Misoprostol, a prostaglandin analog, reduces acid secretion and the incidence of upper GI bleeding associated with NSAID use. Enteric coating of NSAIDs does not reduce the risk for GI bleeding. Corticosteroids increase the risk for ulcer development, and will not be substituted for NSAIDs for this patient. Acetaminophen will not be effective in treating the patient's rheumatoid arthritis

You are conducting a community education session. Which is the best information to provide? A. Women are at highest risk for inguinal hernia. B. Hernias are a result of a low-fiber diet. C. Hernia is not an emergency unless it cannot be reduced. D. Pulsation is commonly seen in abdominal hernias.

C Rationale A hernia is not a concern unless it cannot be reduced and an obstruction occurs in the intestine. Men are at highest risk for an inguinal hernia. Hernias are a result of an abnormal opening or a weakened area in the wall of the cavity. A pulsating abdominal mass is a symptom of an abdominal aortic aneurysm. Reference: 1048

A 47-year-old female patient is transferred from the recovery room to a surgical unit after a transverse colostomy. The nurse observes the stoma to be deep pink with edema and a small amount of sanguineous drainage. The nurse should a. place ice packs around the stoma. b. notify the surgeon about the stoma. c. monitor the stoma every 30 minutes. d. document stoma assessment findings.

ANS: D The stoma appearance indicates good circulation to the stoma. There is no indication that surgical intervention is needed or that frequent stoma monitoring is required. Swelling of the stoma is normal for 2 to 3 weeks after surgery, and an ice pack is not needed

Which assessment should the nurse perform first for a patient who just vomited bright red blood? a. Measuring the quantity of emesis b. Palpating the abdomen for distention c. Auscultating the chest for breath sounds d. Taking the blood pressure (BP) and pulse

ANS: D The nurse is concerned about blood loss and possible hypovolemic shock in a patient with acute gastrointestinal (GI) bleeding. BP and pulse are the best indicators of these complications. The other information is important to obtain, but BP and pulse rate are the best indicators for assessing intravascular volume. DIF: Cognitive Level: Apply (application) REF: 925 | 956 OBJ: Special Questions: Prioritization TOP: Nursing Process: Assessment MSC: NCLEX: Physiological Integrity

Which patient should the nurse assess first after receiving change-of-shift report? a. 60-year-old patient whose new ileostomy has drained 800 mL over the previous 8 hours b. 50-year-old patient with familial adenomatous polyposis who has occult blood in the stool c. 40-year-old patient with ulcerative colitis who has had six liquid stools in the previous 4 hours d. 30-year-old patient who has abdominal distention and an apical heart rate of 136 beats/minute

ANS: D The patient's abdominal distention and tachycardia suggest hypovolemic shock caused by problems such as peritonitis or intestinal obstruction, which will require rapid intervention. The other patients should also be assessed as quickly as possible, but the data do not indicate any life-threatening complications associated with their diagnoses

A 27-year-old female patient is admitted to the hospital for evaluation of right lower quadrant abdominal pain with nausea and vomiting. Which action should the nurse take? a. Encourage the patient to sip clear liquids. b. Assess the abdomen for rebound tenderness. c. Assist the patient to cough and deep breathe. d. Apply an ice pack to the right lower quadrant.

ANS: D The patient's clinical manifestations are consistent with appendicitis, and application of an ice pack will decrease inflammation at the area. Checking for rebound tenderness frequently is unnecessary and uncomfortable for the patient. The patient should be NPO in case immediate surgery is needed. The patient will need to know how to cough and deep breathe postoperatively, but coughing will increase pain at this time

After change-of-shift report, which patient should the nurse assess first? a. 42-year-old who has acute gastritis and ongoing epigastric pain b. 70-year-old with a hiatal hernia who experiences frequent heartburn c. 53-year-old who has dumping syndrome after a recent partial gastrectomy d. 60-year-old with nausea and vomiting who has dry oral mucosa and lethargy

ANS: D This older patient is at high risk for problems such as aspiration, dehydration, and fluid and electrolyte disturbances. The other patients will also need to be assessed, but the information about them indicates symptoms that are typical for their diagnoses and are not life threatening. DIF: Cognitive Level: Apply (application) REF: 928 OBJ: Special Questions: Prioritization

A 50-year-old man vomiting blood-streaked fluid is admitted to the hospital with acute gastritis. To determine possible risk factors for gastritis, the nurse will ask the patient about a. the amount of saturated fat in the diet. b. any family history of gastric or colon cancer. c. a history of a large recent weight gain or loss. d. use of nonsteroidal antiinflammatory drugs (NSAIDs).

ANS: D Use of an NSAID is associated with damage to the gastric mucosa, which can result in acute gastritis. Family history, recent weight gain or loss, and fatty foods are not risk factors for acute gastritis. DIF: Cognitive Level: Understand (comprehension) REF: 941 TOP: Nursing Process: Assessment MSC: NCLEX: Physiological Integrity

The young adult female patient reports unilateral left lower abdominal pain. The last menstrual period was 4 weeks ago. What diagnostic test is essential? A. Orthostatic vital signs B. Hemoglobin and hematocrit C. Pregnancy test D. Appendix ultrasound

C Rationale A pregnancy test is performed in women of childbearing age with acute abdominal pain to rule out ectopic pregnancy, and it should be performed before the other tests. The appendix is on the right side. If the pregnancy test is negative, additional testing for other causes should be done. Reference: 1015

What is cause of abdominal compartment syndrome? A. Uncontrolled systemic hypertension B. Wearing restrictive clothing, decreasing venous flow C. Internal bleeding that causes pressure on organs D. Deficiency of the clotting factors

C Rationale Abdominal compartment syndrome is organ dysfunction caused by intraabdominal hypertension. Abdominal or retroperitoneal bleeding places pressure on abdominal organs located within the abdominal cavity. The other options are not causes of acute compartment syndrome. Reference: 1018

What is the best indication that the intravenous (IV) fluid replacement is adequate during the treatment of a patient with intestinal obstruction? A. Serum sodium: 155 mEq/L B. Urine specific gravity: 1.050 C. Urine output: 0.5 ml/kg/ hour D. Bowel sounds: 4 times/minute

C Rationale Adequate fluid replacement results in urine output of 0.5 mL/kg/ hour. The first two options indicate dehydration. Bowel sounds (peristalsis) are not used to determine rehydration. Reference: 1033

For a patient with Crohn's disease which assessment finding is most important for you to follow-up? A. Bloody diarrheal stool: 4 times/day B. Abdominal cramping C. Temperature: 100.4° F (38° C) D. Positive rebound tenderness

D Rationale Positive rebound tenderness is a classic sign of peritonitis and requires emergency follow-up. The other options are expected signs or symptoms with ulcerative colitis, which has intermittent exacerbations. Reference: 1021, 1023

The nurse would increase the comfort of a patient with appendicitis by: A. having the patient lie prone B. flexing the patient's right knee C. sitting the patient upright in a chair D. turning the patient onto his left side

B

The nurse cares for a postoperative patient who has just vomited yellow green liquid and reports nausea. Which action would be an appropriate nursing intervention? a) Offer the patient a herbal supplement such as ginseng. b) Apply a cool washcloth to the forehead and provide mouth care. c) Take the patient for a walk in the hallway to promote peristalsis. d) Discontinue any medications that may cause nausea or vomiting.

B Cleansing the face and hands with a cool washcloth and providing mouth care are appropriate comfort interventions for nausea and vomiting. Ginseng is not used to treat postoperative nausea and vomiting. Unnecessary activity should be avoided. The patient should rest in a quiet environment. Medications may be temporarily withheld until the acute phase is over, but the *medications should not be discontinued without consultation with the health care provider.*

The patient is diagnosed with infectious diarrhea caused by E. coli. Which treatment do you anticipate? A. Antidiarrheal agent B. Pedialyte C. Antibiotic D. Stool transplantation

B Rationale Acute diarrhea from infectious causes is usually self-limiting. The major concerns are preventing transmission, fluid and electrolyte replacement, and resolution of the diarrhea. Oral solutions such as Gatorade or Pedialyte may be sufficient. Antidiarrheal agents are contraindicated in the treatment of infectious diarrhea because they potentially prolong exposure to the infectious organisms. Antibiotics are rarely used to treat acute diarrhea. Exceptions are certain infections (C. difficile) or patients who are immunosuppressed. Stool transplantation is used as a last resort for recurrent C. difficile infections. Reference: 1008

Which is correct information about the treatment of Crohn's disease? A. Surgery is the preferred treatment. B. Aminosalicylates are frequently used first. C. Corticosteroids are given for long-term therapy. D. High-fiber foods are encouraged to add bulk to diarrheal stool.

B Rationale Aminosalicylates (5-ASAs) are used first because they are less toxic, although there is a movement to using biologic and targeted therapy as first-line therapy. Drugs with 5-ASA suppress the proinflammatory cytokines and inflammatory mediators. Because there is a high rate of recurrence after surgical treatment of Crohn's disease, medication is the preferred treatment, although up to 75% of patients eventually must have surgery. Corticosteroids are given for the shortest time possible due to the complications of long-term therapy. Patients are put on a low-residue diet. Reference: 1025

The patient is brought in with a piece of wood impaled in his lower abdomen after being rescued from a collapsing building. What action should you take? A. Assess for positive rebound pressure. B. Stabilize the wood until examined by a physician. C. Remove the wood and apply direct pressure. D. Note the presence of bowel sounds.

B Rationale An impaled object should never be removed until skilled care is available. Removal may cause further injury and bleeding. Reference: 1020

The patient presents with abdominal pain. Which assessment finding should you follow-up? A. 130/80 mm Hg, 100 beats/minute, 16 breaths/minute B. Fetal posture C. Vomit × 1 green liquid D. Negative rebound tenderness

B Rationale Fetal posture is common with peritoneal irritation. It lessens the tension on the inflamed perineal area. The elevated pulse and blood pressure may be caused by pain and anxiety. Signs of shock include elevated pulse and respirations with normal or decreased blood pressure. Green emesis is likely a sign of bile, and one episode is not of major concern. Negative rebound tenderness is a normal finding. Reference: 1015

The patient with fecal incontinence has not responded to the bowel-retraining program and a medication will be used. Which is most appropriate? A. Oil retention enema B. Bisacodyl (Dulcolax) C. Diphenoxylate and atropine (Lomotil) D. Loperamide (Imodium)

B Rationale If nonpharmacologic treatments are ineffective in reestablishing bowel regularity, Dulcolax or a small phosphate enema can be administered 15 to 30 minutes before the usual evacuation time to stimulate the anorectal reflex. After a regular pattern is reestablished, the mediation is discontinued. An oil retention enema is used for impaction or constipation. Lomotil and Imodium are used for diarrhea and are not part of an incontinent program. Reference: 1012

The patient with ulcerative colitis has a total proctocolectomy with permanent ileostomy. Which instruction is most important to teach the patient? A. Contact the primary provider if there is more than 1500 mL of drainage per day. B. Monitor skin integrity. C. Irrigate to gain control of stool elimination. D. Assess for signs of cobalamin (vitamin B12) deficiency.

B Rationale If the stoma retracts, stool can contact the skin, causing loss of the epidermal layer. An ileostomy can initially drain 1500 to 2000 mL per 24 hours. It is not possible to obtain continence with an ileostomy. Cobalamin deficiency is a concern if the terminal ileus is removed because that is where this vitamin is absorbed. It is not a problem with large intestine removal. Reference: 1028

What is the main treatment for a patient with acute diverticulitis? A. Colon resection and ostomy B. Nasogastric tube and intravenous (IV) fluids C. Long-term course of oral corticosteroids D. Mechanical soft diet

B Rationale In acute diverticulitis, the goal of treatment is to allow the colon to rest and inflammation to subside. Bowel rest can be accomplished with the use of a nasogastric tube and IV fluids. Surgery is reserved for patients with complications, such as an abscess or obstruction that does not respond to medical management. The colon is reanastomosed after it has healed. Corticosteroids are not routinely used with diverticulitis; antibiotics are. During an acute attack, bowel rest is required. A mechanical soft diet is used for patients who lack the ability to adequately chew. Reference: 1047

The patient with Crohn's disease has an ileostomy, with the terminal ileum removed. Absorption of what nutrient is a key concern? A. Carbohydrate B. Cobalamin C. Gluten D. Lactose

B Rationale Patients who had the terminal ileum removed have reduced absorption of cobalamin (vitamin B12). Instrinsic factor is secreted in the stomach but absorbed in the small intestine. Fat absorption is decreased because bile salts are absorbed in the terminal ileum, but not carbohydrates. Lack of absorption of gluten occurs in celiac disease. Lactose intolerance occurs because the enzyme lactase is deficient. Reference: 1045

The immunosuppressant azathioprine (Imuran) is given to maintain remission after corticosteroid induction therapy for an exacerbation of ulcerative colitis. What monitoring is required? A. Carcinogenic embryonic antigen (CEA) B. Complete blood cell count (CBC) C. Prostate-specific antigen (PSA) D. Potassium

B Rationale Regular CBC monitoring is required because the drug can suppress the bone marrow and lead to inflammation of the pancreas or gallbladder. CEA is used to monitor for recurrence of colorectal cancer. PSA is used to monitor for prostate cancer. It is not necessary to specifically monitor for potassium with this drug. Reference: 1026

How does the drug sulfasalazine (Azulfidine) work in the treatment of IBD? A. Destroys bacteria B. Suppresses inflammatory mediators C. Slows gastric motility D. Promotes electrolyte exchange across intestinal membrane

B Rationale Sulfasalazine contains sulfapyridine and 5-aminosalicylic acid (5-ASA). Although the exact action is unknown, it works by suppressing inflammatory mediators. IBD is an autoimmune inflammatory disease; no specific infectious agent has been identified, although antimicrobials (Flagyl, Cipro) occasionally are used. The last two options are not related to this drug. Reference: 1025

Compared with a colostomy, which complication is a patient with an ileostomy at an increased risk for? A. Constipation B. Obstruction C. Flatus D. Polyps

B Rationale The ileostomy patient is susceptible to obstruction because the lumen is less than an inch in diameter and may narrow further at the point where the bowel passes through the fascia-muscle layer of the abdomen. Ileostomies have loose drainage because fluid is not absorbed in the large colon. Flatus can occur in patients with either an ileostomy or a colostomy. Polyps are most often found in the distal colon and rectum, which has been removed if the patient has an ileostomy. Reference: 1045

What is a key distinction between ulcerative colitis and Crohn's disease? A. Presence of intermittent constipation B. Pattern of inflammation C. Age of onset D. Prescribed diet

B Rationale The pattern of inflammation differs between Crohn's disease and ulcerative colitis. Crohn's disease involves all layers of the bowel wall anywhere in the gastrointestinal tract. Ulcerative colitis usually starts in the rectum and moves progressively toward the cecum, staying mainly located in the colon and rectum. Inflammation occurs in the mucosal layer. Intermittent diarrhea and constipation is a symptom of irritable bowel syndrome; diarrhea can occur in both diseases. Both commonly occur during the teenage years and early adulthood, with a second peak in the sixth decade. Both are treated similarly in terms of bowel rest, diet, and drugs. Reference: 1023

Which is the best understanding of colon irrigation? A. It is taught to patients with ascending colostomies. B. The tip should be inside a cone to prevent perforation. C. Use cold water to promote peristalsis. D. Administer 2000 mL of sterile saline.

B Rationale The tip is inside a cone to control the depth of insertion, prevent water from leaking out, and prevent perforation. Irrigation is used only in the distal colon or rectum because the stool is solid there. The water should be warm; cold water can cause cramping. Approximately 500 to 1000 mL of tap water is administered. Reference: 1045

What screening test should you recommend to a white person with an average risk of colorectal cancer? A. Flexible sigmoidoscopy at age 21 B. Initial colonoscopy starting at age 50 C. Stool DNA every 10 years D. Carcinoembryonic antigen (CEA) yearly

B Rationale Whites with an average risk should have a colonoscopy every 10 years starting at age 50 (African Americans should have the first one at age 45). The use of a flexible sigmoidoscopy is decreasing because it evaluates only about 50% of the colon. When used, it is begun at age 50. Stool DNA is less favorable but acceptable. Stool tests must be done frequently since DNA shedding occur at intervals and may be easily missed. CEA is used to monitor disease recurrence after surgery or chemotherapy. Reference: 1036

A 61-year-old patient with suspected bowel obstruction had a nasogastric tube inserted at 4:00 AM. The nurse shares in the morning report that the day shift staff should check the tube for patency at what times? A. 7:00 AM, 10:00 AM, and 1:00 PM B. 8:00 AM, 12:00 PM, and 4:00 PM C. 9:00 AM and 3:00 PM D. 9:00 AM, 12:00 PM, and 3:00 PM

B A nasogastric tube should be checked for patency routinely at 4-hour intervals. Thus if the tube were inserted at 4:00 AM, it would be due to be checked at 8:00 AM, 12:00 PM, and 4:00 PM.

What information would have the highest priority to be included in preoperative teaching for a 68-year-old patient scheduled for a colectomy? A. How to care for the wound B. How to deep breathe and cough C. The location and care of drains after surgery D. Which medications will be used during surgery

B Because anesthesia, an abdominal incision, and pain can impair the patient's respiratory status in the postoperative period, it is of high priority to teach the patient to cough and deep breathe. Otherwise, the patient could develop atelectasis and pneumonia, which would delay early recovery from surgery and hospital discharge. Care for the wound and location and care of the drains will be briefly discussed preoperatively, but done again with higher priority after surgery. Knowing which drugs will be used during surgery may not be meaningful to the patient and should be reviewed with the patient by the anesthesiologist.

A colectomy is scheduled for a 38-year-old woman with ulcerative colitis. The nurse should plan to include what prescribed measure in the preoperative preparation of this patient? A. Instruction on irrigating a colostomy B. Administration of a cleansing enema C. A high-fiber diet the day before surgery D. Administration of IV antibiotics for bowel preparation

B Preoperative preparation for bowel surgery typically includes bowel cleansing with antibiotics, such as oral neomycin and cleansing enemas, including Fleet enemas. Instructions to irrigate the colostomy will be done postoperatively. Oral antibiotics are given preoperatively, and an IV antibiotic may be used in the OR. A clear liquid diet will be used the day before surgery with the bowel cleansing.

Which patient would be at highest risk for developing oral candidiasis? a. A 74-yr-old patient who has vitamin B and C deficiencies b. A 22-yr-old patient who smokes 2 packs of cigarettes per day c. A 32-yr-old patient who is receiving ciprofloxacin for 3 weeks d. A 58-yr-old patient who is receiving amphotericin B for 2 days

Correct answer: C Oral candidiasis is caused by prolonged antibiotic treatment (e.g., ciprofloxacin) or high doses of corticosteroids. Amphotericin B is used to treat candidiasis. Vitamin B and C deficiencies may lead to Vincent's infection. Use of tobacco products leads to stomatitis, not candidiasis.

The patient with sudden pain in the left upper quadrant radiating to the back and vomiting was diagnosed with acute pancreatitis. What intervention(s) should the nurse expect to include in the patient's plan of care? A) Immediately start enteral feeding to prevent malnutrition. B) Insert an NG and maintain NPO status to allow pancreas to rest. C) Initiate early prophylactic antibiotic therapy to prevent infection. D) Administer acetaminophen (Tylenol) every 4 hours for pain relief.

B) Initial treatment with acute pancreatitis will include an NG tube if there is vomiting and being NPO to decrease pancreatic enzyme stimulation and allow the pancreas to rest and heal. Fluid will be administered to treat or prevent shock. The pain will be treated with IV morphine because of the NPO status. Enteral feedings will only be used for the patient with severe acute pancreatitis in whom oral intake is not resumed. Antibiotic therapy is only needed with acute necrotizing pancreatitis and signs of infection.

Assessment findings suggestive of peritonitis include a. rebound abdominal pain. b. a soft, distended abdomen. c. dull, continuous abdominal pain. d. observing that the patient is restless.

Correct answer: a Rationale: With peritoneal irritation, the abdomen is hard, like a board, and the patient has severe abdominal pain that is worse with any sudden movement. The patient lies very still. Palpating the abdomen and releasing the hands suddenly causes sudden movement within the abdomen and severe pain. This is called rebound tenderness.

Following administration of a dose of metoclopramide (Reglan) to the patient, the nurse determines that the medication has been effective when what is noted? a) Decreased blood pressure b) Absence of muscle tremors c) Relief of nausea and vomiting d) No further episodes of diarrhea

C Metoclopramide is classified as a prokinetic and antiemetic medication. If it is effective, the patient's nausea and vomiting should resolve. Metoclopramide does not affect blood pressure, muscle tremors, or diarrhea.

Which action will assist in caring for the patient with fecal incontinence? A. Assess stool consistency using the Braden scale. B. Encourage the use of a rectal tube to prevent skin breakdown. C. Assist the patient to the bathroom at a regular time daily. D. Encourage morning coffee to promote stool evacuation.

C Rationale Assisting with bowel training can help regulate evacuation. This can include assisting the patient to the bathroom at a regular time daily. Thirty minutes after breakfast is the time often recommended. The Braden Scale measures skin breakdown; the Bristol Stool Scale assesses stool consistency. Rectal tubes should be avoided because they can decrease responsiveness of the rectal sphincter and cause ulceration of the rectal mucosa. Patients with stool incontinence are taught to avoid caffeine, which worsens symptoms. Reference: 1011-1012

An elderly hospitalized patient, who was recently on broad-spectrum antibiotics, develops foul-smelling diarrhea. What is the most likely cause? A. Giardia lamblia B. Salmonella C. Clostridium difficile D. Lactose intolerance

C Rationale C. difficile is often a health care-acquired infection. Spores can survive up to 70 days. People at risk include elderly hospitalized patients who had antibiotics in the past 2 months. G. lamblia is a parasite found in contaminated lakes or pools. Salmonella is found in undercooked eggs or chicken. It is unusual for the patient to suddenly develop lactose intolerance and symptoms. Reference: 1007-1008

A patient with metastatic colorectal cancer is scheduled for chemotherapy and radiation therapy. Patient teaching regarding these therapies should include which explanation? A. Chemotherapy can be used to cure colorectal cancer. B. Irradiation is routinely used as adjuvant therapy after surgery. C. Both chemotherapy and irradiation can be used as palliative treatments. D. The patient should expect few or no side effects from chemotherapeutic agents.

C Rationale Chemotherapy can be used to shrink the tumor before surgery, as an adjuvant therapy after colon resection, and as palliative therapy for nonresectable colorectal cancer. Radiation therapy may be used postoperatively as an adjuvant to surgery and chemotherapy or as a palliative measure for patients with metastatic cancer. Reference: 1038

Which is the best category of food to encourage prevention of diverticulosis? A. High iron, such as organ meats B. Low gluten C. High fiber, such as raw vegetables D. No nuts or popcorn

C Rationale Foods high in fiber (bulk), such as fresh fruits and vegetables (along with decreased fat and red meat), help to prevent diverticulosis. Iron and gluten are not specific for diverticulosis. There is no evidence to support avoiding nuts and seeds to prevent diverticulitis, and they may have a protective effect. Reference: 1047

The patient has nonresectable colorectal cancer. The primary provider has recommended chemotherapy. What is the best explanation of this treatment? A. It gives the patient a sense of hope that something is being done. B. It shrinks the tumor before surgery. C. It provides palliative treatment. D. It prevents metastasis to the liver.

C Rationale Palliative treatment is done for nonresectable colorectal cancer to shrink the tumor and prevent obstruction. Telling the patient that chemotherapy may cure the cancer is not realistic and provides false information. This patient's tumor is classified as nonresectable, indicating that surgical treatment is not an option. In this situation it probably means that the tumor has already metastasized. Reference: 1038

What should you teach a patient with colorectal cancer who is receiving capecitabine (Xeloda) for chemotherapy? A. Obtain weekly CBC count. B. Monitor the stool for occult blood. C. Do not get any immunizations without physician's approval. D. Take your temperature daily.

C Rationale Patients should not receive immunizations during chemotherapy because resistance is low, and they are unable to build up immunity. The other options are not specific for this drug, although the general instruction to monitor for bleeding or infection can be done. Reference: 1038

A female college student goes to the university health clinic complaining of pain that started at the umbilicus and moved to the right lower quadrant over the last 12 hours. You notice muscle guarding on examination. What action should you take? A. Administer a PRN laxative per standing orders. B. Ask about the last menstrual period. C. Make the student NPO. D. Assess bowel sounds.

C Rationale This is a classic description of appendicitis. At the very least, it is an acute abdomen, and the student should be kept NPO until a need for surgery is ruled out. The student should be referred to an emergency department. A laxative should not be given because it can increase peristalsis and cause perforation. Asking about her last menstrual cycle (possibility of a ruptured ectopic pregnancy) is important but the symptoms suggest appendicitis. Bowel sounds should be assessed, but the NPO status is a priority. Reference: 1021

The patient had an ileostomy 4 days earlier and has a daily drainage of 1800 mL. What action should you take? A. Notify the primary provider. B. Send a specimen to the laboratory. C. Document the findings. D. Test the stool for occult blood.

C Rationale With an ileostomy, the volume of drainage is high (1000 to 1800 mL/day) after peristalsis returns because the adsorptive functions provided by the colon and the delay provided by the ileocecal valve have been altered. Eventually, the amount of drainage is reduced to 500 mL daily as the proximal small bowel adapts. If the small bowel has been shortened from resections, the drainage can be greater. There is no need to do the other options. Reference: 1045

A 74-year-old female patient with gastroesophageal reflux disease (GERD) takes over-the-counter medications. For which medication, if taken long-term, should the nurse teach about an increased risk of fractures? a) Sucralfate (Carafate) b) Cimetidine (Tagamet) c) Omeprazole (Prilosec) d) Metoclopramide (Reglan)

C There is a potential link between proton pump inhibitors (PPIs) (e.g., omeprazole) use and bone metabolism. Long-term use or high doses of PPIs may increase the risk of fractures of the hip, wrist, and spine. Lower doses or shorter duration of therapy should be considered. REGLAN--> TARDIVE DYSKINESIA

The patient with chronic gastritis is being put on a combination of medications to eradicate H. pylori. Which drugs does the nurse know will probably be used for this patient? a) Antibiotic(s), antacid, and corticosteroid b) Antibiotic(s), aspirin, and antiulcer/protectant c) Antibiotic(s), proton pump inhibitor, and bismuth d) Antibiotic(s) and nonsteroidal antiinflammatory drugs (NSAIDs)

C To eradicate H. pylori, a combination of antibiotics, a proton pump inhibitor, and possibly bismuth (for quadruple therapy) will be used. Corticosteroids, aspirin, and NSAIDs are drugs that can cause gastritis and do not affect H. pylori.

The nurse receives an order for a parenteral dose of promethazine (Phenergan) and prepares to administer the medication to a 38-year-old male patient with nausea and repeated vomiting. Which action is most important for the nurse to take? a) Administer the medication subcutaneously for fast absorption. b) Administer the medication into an arterial line to prevent extravasation. c) Administer the medication deep into the muscle to prevent tissue damage. d) Administer the medication with 0.5 mL of lidocaine to decrease injection pain.

C Promethazine (Phenergan) is an antihistamine administered to relieve nausea and vomiting. Deep muscle injection is the preferred route of injection administration. This medication should not be administered into an artery or under the skin because of the risk of severe tissue injury, including gangrene. When administered IV, a risk factor is that it can leach out from the vein and cause serious damage to surrounding tissue.

The nurse teaches a 50-year-old woman who has a body mass index (BMI) of 39 kg/m2 about weight loss. Which dietary change would be appropriate for the nurse to recommend to this patient? a. Decrease fat intake and control portion size b. Increase vegetables and decrease fluid intake c. Increase protein intake and avoid carbohydrates d. Decrease complex carbohydrates and limit fiber

Correct answer: a Rationale: The safest dietary guideline for weight loss is to decrease caloric intake by maintaining a balance of nutrients and adequate hydration while controlling portion size and decreasing fat intake.

A patient with cholelithiasis needs to have the gallbladder removed. Which patient assessment is a contraindication for a cholecystectomy? A) Low-grade fever of 100° F and dehydration B) Abscess in the right upper quadrant of the abdomen C) Activated partial thromboplastin time (aPTT) of 54 seconds D) Multiple obstructions in the cystic and common bile duct

C) An aPTT of 54 seconds is above normal and indicates insufficient clotting ability. If the patient had surgery, significant bleeding complications postoperatively are very likely. Fluids can be given to eliminate the dehydration; the abscess can be assessed, and the obstructions in the cystic and common bile duct would be relieved with the cholecystectomy.

A 54-year-old patient admitted with diabetes mellitus, malnutrition, osteomyelitis, and alcohol abuse has a serum amylase level of 280 U/L and a serum lipase level of 310 U/L. To which of the following diagnoses does the nurse attribute these findings? A. Malnutrition B. Osteomyelitis C. Alcohol abuse D. Diabetes mellitus

C) The patient with alcohol abuse could develop pancreatitis as a complication, which would increase the serum amylase (normal 30-122 U/L) and serum lipase (normal 31-186 U/L) levels as shown.

A 20-year-old man is admitted to the emergency department after a motor vehicle crash with suspected abdominal trauma. What assessment finding by the nurse is of highest priority? A. Nausea and vomiting B. Hyperactive bowel sounds C. Firmly distended abdomen D. Abrasions on all extremities

C. Firmly distended abdomen Clinical manifestations of abdominal trauma are guarding and splinting of the abdominal wall; a hard, distended abdomen (indicating possible intraabdominal bleeding); decreased or absent bowel sounds; contusions, abrasions, or bruising over the abdomen; abdominal pain; pain over the scapula; hematemesis or hematuria; and signs of hypovolemic shock (tachycardia and decreased blood pressure).

Since removal of the patient's Foley catheter, the patient has voided 50 to 100 mL every 2 to 3 hours. Which action should the nurse take first? A) Check for bladder distention B) Encourage fluid intake C) Obtain an order to recatheterize the patient D) Document the amount of each voiding for 24 hours

Check for bladder distention

The nurse notes that the patient's Foley catheter bag has been empty for 4 hours. The priority action would be to: A) Irrigate the Foley. B) Check for kinks in the tubing. C) Notify the health care provider. D) Assess the patient's intake.

Check for kinks in the tubing.

The patient with right upper quadrant abdominal pain has an abdominal ultrasound that reveals cholelithiasis. What should the nurse expect to do for this patient? Prevent all oral intake. Control abdominal pain. Provide enteral feedings. Avoid dietary cholesterol.

Control abdominal pain. Patients with cholelithiasis can have severe pain, so controlling pain is important until the problem can be treated. NPO status may be needed if the patient will have surgery but will not be used for all patients with cholelithiasis. Enteral feedings should not be needed, and avoiding dietary cholesterol is not used to treat cholelithiasis.

A 90-year-old healthy man is suffering from dysphagia. The nurse explains what age-related change of the GI tract is the most likely cause of his difficulty? a. Xerostomia b. Esophageal cancer c. Decreased taste buds d. Thinner abdominal wall

Correct answer: a Rationale: Xerostomia, decreased volume of saliva, leads to dry oral mucosa and dysphagia. Esophageal cancer is not an age-related change. Decreased taste buds and a thinner abdominal wall do not contribute to difficulty swallowing.

The nurse is caring for a patient who complains of abdominal pain and hematemesis. Which new assessment finding(s) would indicate the patient is experiencing a decline in condition? a. Pallor and diaphoresis b. Ecchymotic peripheral IV site c. Guaiac-positive diarrhea stools d. Heart rate 90, respiratory rate 20, BP 110/60

Correct Answer: a A patient with hematemesis has some degree of bleeding from an unknown source. Guaiac-positive diarrhea stools would be an expected finding. When monitoring the patient for stability, the nurse observes for signs of hypovolemic shock such as tachycardia, tachypnea, hypotension, altered level of consciousness, pallor, and cool and clammy skin. An ecchymotic peripheral IV site will require assessment to determine the need for reinsertion. Access would be critical in the immediate treatment of shock, but the ecchymotic site does not represent a decline in condition.

The nurse teaches senior citizens at a community center how to prevent food poisoning at social events. Which community member statement reflects accurate understanding? a. "Pasteurized juices and milk are safe to drink." b. "Alfalfa sprouts are safe if rinsed before eating." c. "Fresh fruits do not need to be washed before eating." d. "Ground beef is safe to eat if cooked until it is brown."

Correct answer: a Drink only pasteurized milk, juice, or cider. Ground beef should be cooked thoroughly. Browned meat can still harbor live bacteria. Cook ground beef until a thermometer reads at least 160° F. If a thermometer is unavailable, decrease the risk of illness by cooking the ground beef until there is no pink color in the middle. Fruits and vegetables should be washed thoroughly, especially those that will not be cooked. Persons who are immunocompromised or older should avoid eating alfalfa sprouts until the safety of the sprouts can be ensured.

The nurse instructs an obese 22-year-old man with a sedentary job about the health benefits of an exercise program. The nurse evaluates that teaching is effective when the patient makes which statement? a. "The goal is to walk at least 10,000 steps every day of the week." b. "Weekend aerobics for 2 hours is better than exercising every day." c. "Aerobic exercise will increase my appetite and result in weight gain." d. "Exercise causes weight loss by decreasing my resting metabolic rate."

Correct answer: a Rationale: A realistic activity goal is to walk 10,000 steps a day. Increased activity does not promote an increase in appetite or lead to weight gain. Exercise should be done daily, preferably 30 minutes to an hour a day. Exercise increases metabolic rate.

What problem should the nurse assess the patient for if the patient was on prolonged antibiotic therapy? a. Coagulation problems b. Elevated serum ammonia levels c. Impaired absorption of amino acids d. Increased mucus and bicarbonate secretion

Correct answer: a Rationale: Bacteria int he colon (1) synthesize vitamin K, which is needed for the production of prothrombin by the liver and (2) deaminate undigested or non absorbed proteins, producing ammonia, which is converted to urea by the liver. A reduction in normal flora bacteria by antibiotic therapy can lead to decreased vitamin K, resulting in decreased prothrombin and coagulation problems. Bowel bacteria do not influence protein absorption or the secretion of mucus.

A patient is jaundiced and her stools are clay colored (gray). This is most likely related to a. decreased bile flow into the intestine. b. increase production of urobilinogen. c. increased production of cholecystokinin. d. increased bile and bilirubin in the blood.

Correct answer: a Rationale: Bile is produced by the hepatocytes and is stored and concentrated in the gallbladder. When bile is released from the common bile duct, it enters the duodenum. In the intestines, bilirubin is reduced to stercobilinogen and urobilinogen by bacterial action. Stercobilinogen accounts for the brown color of stool. Stools may be clay-colored if bile is not released from the common bile duct into the duodenum. Jaundice may result if the bilirubin level in the blood is elevated.

What information should be included in the dietary teaching for the patient following a Roux-en-Y gastric bypass? a. Avoid sugary foods and limit fluids to prevent dumping syndrome. b. Gradually increase the amount of food ingested to preoperative levels. c. Maintain a long-term liquid diet to prevent damage to the surgical site. d. Consume foods high in complex carbohydrates, protein, and fiber to add bulk to contents.

Correct answer: a Rationale: Fluids and foods high in carbohydrates tend to promote diarrhea and symptoms of dumping syndrome in patients with gastric bypass surgery. The diet generally should be high in protein and low in carbohydrates, fat, and roughage and consists of six small feedings a day because of the small stomach size. Liquid diets are likely to be used longer for the patient with a gastroplasty.

A patient had a stomach resection for stomach cancer. The nurse should teach the patient about the loss of the hormone that stimulates gastric acid secretion and motility and maintains lower esophageal sphincter tone. Which hormone will be decreased with a gastric resection? a. Gastrin b. Secretin c. Cholecystokinin d. Gastric inhibitory peptide

Correct answer: a Rationale: Gastrin is the hormone activated in the stomach (and duodenal mucosa) by stomach distention that stimulates gastric acid secretion and motility and maintains lower esophageal sphincter tone. Secretin inhibits gastric motility and acid secretion and stimulates pancreatic bicarbonate secretion. Cholecystokinin allows increased flow of bile into the duodenum and release of pancreatic digestive enzymes. Gastric inhibitory peptide inhibits gastric acid secretion and motility.

The nurse is caring for a patient who is 5'6" tall and weighs 186 lb. The nurse has discussed reasonable weight loss goals and a low-calorie diet with the patient. Which statement made by the patient indicates a need for further teaching? a. "I will limit intake to 500 calories a day." b. "I will try to eat very slowly during mealtimes." c. "I'll try to pick foods from all of the basic food groups." d. "It's important for me to begin a regular exercise program."

Correct answer: a Rationale: Limiting intake to 500 calories per day is not indicated for this patient, and the severe calorie energy restriction would place this patient at risk for multiple nutrient deficiencies. Decreasing caloric intake at least 500 to 1000 calories a day is recommended for weight loss of one to two pounds per week. The other options show understanding of the teaching.

A 50-year-old African American woman has a BMI of 35 kg/m2, type 2 diabetes mellitus, hypercholesterolemia, and irritable bowel syndrome (IBS). She is seeking assistance in losing weight, because, "I have trouble stopping eating when I should, but I do not want to have bariatric surgery." Which drug therapy should the nurse question if it is prescribed for this patient? a. Orlistat (Xenical) b. Locaserin (Belviq) c. Phentermine (Adipex-P) d. Phentermine and topiramate (Qsymia)

Correct answer: a Rationale: Orlistat (Xenical), which blocks fat breakdown and absorption in the intestine, produces some unpleasant GI side effects. This drug would not be appropriate for someone with IBS. Locaserin (Belviq) suppresses the appetite and creates a sense of satiety that may be helpful for this patient. Phentermine (Adipex-P) needs to be used for a limited period of time (3 months or less). Qsymia is a combination of two drugs, phentermine and topiramate. Phentermine is a sympathomimetic agent that suppresses appetite and topiramate induces a sense of satiety.

A patient has been on a 1000-calorie diet with a daily exercise routine. In 2 months, the patient has lost 20 lb (9kg) toward a goal of 50 lb (23 kg) but is now discouraged that no weight has been lost in the last 2 weeks. What should the nurse tell the patient about this? a. Plateaus where no weight is lost normally occur during a weight-loss program. b. A weight considered by the body to most efficient for functioning has been reached. c. A return to former eating habits is the most common cause of not continuing to lose weight. d. A steady weight may be due to water gain from eating foods high in sodium.

Correct answer: a Rationale: Plateau periods during which no weight is lost are normal occurrences during weight reduction and may last for several days to several weeks but weight loss will resume if the prescribed weight reduction plan is continued. Weight loss may stop if former eating habits are resumed but this not the most common cause of plateaus.

A patient asks the nurse about taking phentermine and topiramate (Qsymia) for weight loss. To avoid side effects, it is important for the nurse to determine whether the patient has a history of a. glaucoma. b. hypertension. c. valvular heart disease. d. irritable bowel disease.

Correct answer: a Rationale: Qsymia is a combination of phentermine and topiramate. It must not be used in patients with glaucoma or hyperthyroidism.

The percentage of daily calories for a healthy individual consists of a. 50% carbohydrates, 25% protein, 25% fat, and <10% of fat from saturated fatty acids. b. 65% carbohydrates, 25% protein, 25% fat, and >10% of fat from saturated fatty acids. c. 50% carbohydrates, 40% protein, 10% fat, and <10% of fat from saturated fatty acids. d. 40% carbohydrates, 30% protein, 30% fat, and >10% of fat from saturated fatty acids.

Correct answer: a Rationale: The 2005 Dietary Guidelines for Americans recommend that 45% to 65% of total calories should come from carbohydrates. Ideally, 10% to 35% of daily caloric needs should come from protein. Individuals should limit their fat intake to 20% to 35% of total calories. Additional recommendations focus on the type of fat consumed because diets high in excess calories, usually in the form of fats, contribute to the development of obesity. Individuals should consume less than 10% of calories from saturated fatty acids, limit intake of fat and oils high in trans fatty acids, and should limit intake of dietary cholesterol to 300 mg/day.

A patient with hepatitis A is in the acute phase. The nurse plans care for the patient based on the knowledge that a. pruritus is a common problem with jaundice in this phase. b. the patient is most likely to transmit the disease during this phase. c. gastrointestinal symptoms are not as severe in hepatitis A as they are in hepatitis B. d. extrahepatic manifestations of glomerulonephritis and polyarteritis are common in this phase. (Lewis 1042)

Correct answer: a Rationale: The acute phase of jaundice may be icteric or anicteric. Jaundice results when bilirubin diffuses into the tissues. Pruritus sometimes accompanies jaundice. Pruritus is the result of an accumulation of bile salts beneath the skin.

During the initial postoperative period following bariatric surgery, the nurse recognizes the importance of monitoring obese patients for respiratory insufficiency based on what knowledge? a. The body stores anesthetics in adipose tissue. b. Postoperative pain may cause a decreased respiratory rate. c. Intubation may be difficult because of extra chin skinfolds. d. The patient's head must remain flat for a minimum of 2 hours postprocedure.

Correct answer: a Rationale: The body stores anesthetics in adipose tissue, placing patients with excess adipose tissue at risk for re-sedation. As adipose cells release anesthetics back into the bloodstream, the patient may become sedated after surgery, increasing the risk of hypoventilation and resultant respiratory insufficiency. Difficult intubation does not cause respiratory insufficiency. Pain usually increases respiratory rate. The patient's head should be elevated after bariatric surgery to decrease abdominal pressure and facilitate respirations.

The nurse cares for a 34-year-old woman after bariatric surgery. The nurse determines that discharge teaching related to diet is successful if the patient makes which statement? a. "A high protein diet that is low in carbohydrates and fat will prevent diarrhea." b. "Food should be high in fiber to prevent constipation from the pain medication." c. "Three meals a day with no snacks between meals will provide optimal nutrition." d. "Fluid intake should be at least 2000 mL per day with meals to avoid dehydration."

Correct answer: a Rationale: The diet generally prescribed is high in protein and low in carbohydrates, fat, and roughage and consists of six small feedings daily. Fluids should not be ingested with the meal, and in some cases, fluids should be restricted to less than 1000 mL per day. Fluids and foods high in carbohydrate tend to promote diarrhea and symptoms of the dumping syndrome. Generally, calorically dense foods (foods high in fat) should be avoided to permit more nutritionally sound food to be consumed.

Checking for the return of the gag reflex and monitoring for LUQ pain, nausea and vomiting are necessary nursing actions after which diagnostic procedure? a. ERCP b. Colonoscopy c. Barium swallow d. Esophagogastroduodenoscopy (EGD)

Correct answer: a Rationale: The left upper quadrant (LUQ) pain and nausea and vomiting could occur from perforation. The return of gag reflex is essential to prevent aspiration after an ERCP. The gag reflex is also assessed with an EGD. These are not relevant assessments for the colonoscopy and barium swallow.

The nurse explains to the patient undergoing ostomy surgery that the procedure that maintains the most normal functioning of the bowel is a. a sigmoid colostomy. b. a transverse colostomy. c. a descending colostomy. d. an ascending colostomy.

Correct answer: a Rationale: The more distal the ostomy is, the more the intestinal contents resemble feces eliminated from an intact colon and rectum. Output from a sigmoid colostomy resembles normally formed stool, and some patients are able to regulate emptying time so they do not need to wear a collection bag.

During assessment of the patient with protein-calorie malnutrition, what should the nurse expect to find (select all that apply)? a. Frequent cold symptoms b. Decreased bowel sounds c. Cool, rough, dry, scaly skin d. A flat or concave abdomen e. Prominent bony structures f. Decreased reflexes and lack of attention

Correct answer: a, b, c, e, f Rationale: In malnutrition, metabolic processes are slowed, leading to increased sensitivity to cold, decreased heart rate (HR) and cardiac output (CO), and decreased neurologic function. Because of slowed GI motility and absorption, the abdomen becomes distended and protruding and bowel sounds are decreased. Skin is rough, dry, and scaly whereas bone structures protrude because of muscle loss. Because the immune system is weakened, susceptibility to respiratory infections is increased.

The stable patient has a gastrostomy tube for enteral feeding. Which care could the RN delegate to the LPN (select all that apply)? a. Administer bolus or continuous feedings. b. Evaluate the nutritional status of the patient. c. Administer medications through the gastrostomy tube. d. Monitor for complications related to the tube and enteral feeding. e. Teach the caregiver about feeding via the gastrostomy tube at home.

Correct answer: a, c Rationale: For the stable patient, the LPN can administer bolus or continuous feedings and administer medications through the gastrostomy. The RN must evaluate the nutritional status of the patient, monitor for complications related to the tube and the enteral feeding, and teach the caregiver about feeding via the gastrostomy tube at home.

The patient has parenteral nutrition infusing with amino acids and dextrose. In report, the oncoming nurse is told that the tubing, the bag, and the dressing were changed 22 hours ago. What care should the nurse coming on be prepared to do (select all that apply)? a. Give the patient insulin. b. Check amount of feeding left in the bag. c. Check that the next bag has been ordered. d. Check the insertion site and change the tubing. e. Check the label to ensure ingredients and solution are as ordered.

Correct answer: a,b,c,e Rationale: The nurse should check the amount of feeding left in the bag, and that the next bag has been ordered to be sure the solution will not run out before the next bag is available. Parenteral nutrition solutions are only good for 24 hours and usually take some time for the pharmacy to mix for each patient. The label on the bag should be checked to ensure that the ingredients and solution are what was ordered. The patient would only receive insulin if the patient is experiencing hyperglycemia and was receiving sliding scale insulin or had diabetes mellitus. The insertion site should be checked, but the tubing is only changed every 72 hours unless lipids are being used.

The nurse should administer an as-needed dose of magnesium hydroxide (MOM) after noting what information while reviewing a patient's medical record? a. Abdominal pain and bloating b. No bowel movement for 3 days c. A decrease in appetite by 50% over 24 hours d. Muscle tremors and other signs of hypomagnesemia

Correct answer: b Rationale: MOM is an osmotic laxative that produces a soft, semisolid stool usually within 15 minutes to 3 hours. This medication would benefit the patient who has not had a bowel movement for 3 days. MOM would not be given for abdominal pain and bloating, decreased appetite, or signs of hypomagnesemia.

The nurse is caring for a postoperative patient who has just vomited yellow green liquid and reports nausea. Which action would be an appropriate nursing intervention? a. Offer the patient an herbal supplement such as ginseng. b. Apply a cool washcloth to the forehead and provide mouth care. c, Take the patient for a walk in the hallway to promote peristalsis. d. Discontinue any medications that may cause nausea or vomiting. TERM

Correct answer: b Cleansing the face and hands with a cool washcloth and providing mouth care are appropriate comfort interventions for nausea and vomiting. Ginseng is not used to treat postoperative nausea and vomiting. Unnecessary activity should be avoided. The patient should rest in a quiet environment. Medications may be temporarily withheld until the acute phase is over, but the medications should not be discontinued without consultation with the health care provider.

The patient receiving chemotherapy rings the call bell and reports the onset of nausea. The nurse should prepare an as-needed dose of which medication? a. Zolpidem b. Ondansetron c. Dexamethasone d. Morphine sulfate

Correct answer: b Ondansetron is a 5-HT3 receptor antagonist antiemetic that is especially effective in reducing cancer chemotherapy-induced nausea and vomiting. Morphine sulfate may cause nausea and vomiting. Zolpidem does not relieve nausea and vomiting. Dexamethasone is usually used in combination with ondansetron for acute and chemotherapy-induced emesis.

The nurse is caring for a 45-year-old woman with a herniated lumbar disc. The patient realizes that weight loss is necessary to lessen back strain. The patient is 5'6" tall and weighs 186 lb (84.5 kg) with a body mass index (BMI) of 28 kg/m2. The nurse explains to the patient that this measurement places her in which of the following weight categories? a. Normal weight b. Overweight c. Obese d. Severely obese

Correct answer: b Rationale: A normal BMI is 18.5 to 24.9 kg/m2, whereas a BMI of 25 to 29.9 kg/m2 is considered overweight. A BMI of 30.0-39.9 is considered obese, and a BMI of 40 or greater is severely obese.

A woman is 5 ft, 6 in (166 cm) tall and weighs 200 lb (90.9 kg) with a waist-to-hip ratio of 0.7. The nurse counsels the patient with the knowledge that the patient is at greatest risk for a. heart disease. b. osteoporosis. c. diabetes mellitus. d. endometrial cancer.

Correct answer: b Rationale: A patient who is obese (BMI of 32.2) but has a waist-to-hip ratio of less than 0.8, indicating gynoid obesity, has an increased risk for osteoporosis. The other conditions are risks associated with android obesity.

When teaching the older adult about nutritional needs during aging, what does the nurse emphasize? a. Need for all nutrients decreases as one ages. b. Fewer calories, but the same or slightly increased amount of protein, are required as one ages. c. Fats, carbohydrates, and protein should be decreased, but vitamin and mineral intake should be increased. d. High-calorie oral supplements should be taken between meals to ensure that recommended nutrient needs are met.

Correct answer: b Rationale: Although calorie intake should be decreased in the older adult because of decreased activity and basal metabolic rate, the need for specific nutrients, such as proteins and vitamins, does not change.

The patient with advanced cirrhosis asks why his abdomen is so swollen. The nurse's response is based on the knowledge that a. a lack of clotting factors promotes the collection of blood in the abdominal cavity. b. portal hypertension and hypoalbuminemia cause a fluid shift into the peritoneal space. c. decreased peristalsis in the GI tract contributes to gas formation and distention of the bowel. d. bile salts in the blood irritate the peritoneal membranes, causing edema and pocketing of fluid. (Lewis 1042)

Correct answer: b Rationale: Ascites is the accumulation of serous fluid in the peritoneal or abdominal cavity and is a common manifestation of cirrhosis. With portal hypertension, proteins shift from the blood vessels through the larger pores of the sinusoids (capillaries) into the lymph space. When the lymphatic system is unable to carry off the excess proteins and water, those substances leak through the liver capsule into the peritoneal cavity. Osmotic pressure of the proteins pulls additional fluid into the peritoneal cavity. A second mechanism of ascites formation is hypoalbuminemia, which results from the inability of the liver to synthesize albumin. Hypoalbuminemia results in decreased colloidal oncotic pressure. A third mechanism is hyperaldosteronism, which occurs when aldosterone is not metabolized by damaged hepatocytes. The increased level of aldosterone causes increases in sodium reabsorption by the renal tubules. Sodium retention and an increase in antidiuretic hormone levels cause additional water retention.

A patient has an elevated blood level of indirect (unconjugated) bilirubin. One cause of this finding is that a. the gallbladder is unable to contract to release stored bile. b. bilirubin is not being conjugated and excreted into the bile by the liver. c. the Kupffer cells in the liver are unable to remove bilirubin from the blood. d. there is an obstruction in the biliary tract preventing flow of bile into the small intestine.

Correct answer: b Rationale: Bilirubin is a pigment derived from the breakdown of hemoglobin and is insoluble in water. Bilirubin is bound to albumin for transport to the liver and is referred to as unconjugated. An indirect bilirubin determination is a measurement of unconjugated bilirubin, and the level may be elevated in hepatocellular and hemolytic conditions.

A patient who has suffered severe burns in a motor vehicle accident will soon be started on parenteral nutrition (PN). Which principle should guide the nurse's administration of the patient's nutrition? a. Administration of PN requires clean technique. b. Central PN requires rapid dilution in a large volume of blood. c. Peripheral PN delivery is preferred over the use of a central line. d. Only water-soluble medications may be added to the PN by the nurse.

Correct answer: b Rationale: Central PN is hypertonic and requires rapid dilution in a large volume of blood. Because PN is an excellent medium for microbial growth, aseptic technique is necessary during administration. Administration through a central line is preferred over the use of peripheral PN, and the nurse may not add any medications to PN.

A patient with acute hepatitis B is being discharged in 2 days. In the discharge teaching plan the nurse should include instructions to a. avoid alcohol for the first 3 weeks. b. use a condom during sexual intercourse. c. have family members get an injection of immunoglobulin. d. follow a low-protein, moderate-carbohydrate, moderate-fat diet. (Lewis 1042)

Correct answer: b Rationale: Hepatitis B virus may be transmitted by mucosal exposure to infected blood, blood products, or other body fluids (e.g., semen, vaginal secretions, saliva). Hepatitis B is a sexually transmitted disease that is acquired through unprotected sex with an infected person. Condom use should be taught to patients to prevent transmission of hepatitis B.

During starvation, the order in which the body obtains substrate for energy is a. visceral protein, skeletal protein, fat, glycogen b. glycogen, skeletal protein, fat stores, visceral protein c. visceral protein, fat stores, glycogen, skeletal protein d. fat stores, skeletal protein, visceral protein, glycogen

Correct answer: b Rationale: Initially, the body selectively uses carbohydrates (e.g., glycogen) rather than fat and protein to meet metabolic needs. These carbohydrate stores, found in the liver and muscles, are minimal and may be totally depleted within 18 hours. After carbohydrate stores are depleted, skeletal protein begins to be converted to glucose for energy. Within 5 to 9 days, body fat is fully mobilized to supply much of the needed energy. In prolonged starvation, up to 97% of calories are provided by fat, and protein is conserved. Depletion of fat stores depends on the amount available, but fat stores typically are used up in 4 to 6 weeks. After fat stores are used, body or visceral proteins, including those in internal organs and plasma, can no longer be spared and rapidly decrease because they are the only remaining body source of energy available.

When medications are used in the treatment of obesity, what is most important for the nurse to teach the patient? a. Over-the-counter (OTC) diet aids are safer than other agents and con be useful in controlling appetite. b. Drugs should be used only as adjuncts to a diet and exercise program as treatment for a chronic condition. c. All drugs used for weight control are capable of altering central nervous system (CNS) function and should be used with caution. d. The primary effect of the medications is psychologic, controlling the urge to eat in response to stress or feelings of rejection.

Correct answer: b Rationale: Medications are used only as adjuncts to diet and exercise programs in the treatment of obesity. Drugs do not cure obesity; without changes in food intake and physical activity, weight gain will occur when the medications are discontinued. The medications used work in a variety of ways to control appetite but over-the-counter drugs are probably the least effective and most abused of these drugs.

A patient has been told that she has elevated liver enzymes caused by nonalcoholic fatty liver disease (NAFLD). The nursing teaching plan should include a. having genetic testing done. b. recommending a heart-healthy diet. c. the necessity to reduce weight rapidly. d. avoiding alcohol until liver enzymes return to normal. (Lewis 1042)

Correct answer: b Rationale: Nonalcoholic fatty liver disease (NAFLD) can progress to liver cirrhosis. There is no definitive treatment, and therapy is directed at reduction of risk factors, which include treatment of diabetes, reduction in body weight, and elimination of harmful medications. For patients who are overweight, weight reduction is important. Weight loss improves insulin sensitivity and reduces liver enzyme levels. No specific dietary therapy is recommended. However, a heart-healthy diet as recommended by the American Heart Association is appropriate.

An 80-year-old man states that, although be adds a lot of salt to his food, it still does not have much taste. The nurse's response is based on the knowledge that the older adult a. should not experience changes in taste. b. has a loss of taste buds, especially for sweet and salty. c. has some loss of taste but no difficulty chewing food. d. loses the sense of taste because the ability to smell is decreased.

Correct answer: b Rationale: Older adults have decreased numbers of taste buds and a decreased sense of smell. These age-related changes diminish the sense of taste (especially of salty and sweet substances).

The nurse admitting a patient for bariatric surgery obtains the following information from the patient. Which finding should be brought to the surgeon's attention before proceeding with further patient preparation? a. History of hypertension b. History of untreated depression c. History of multiple attempts at weight loss d. History of sleep apnea treated with continuous positive airway pressure (CPAP)

Correct answer: b Rationale: Patients with histories of untreated depression or psychosis are not good candidates for surgery. All other historical information includes medical complications of severe obesity that would help to qualify the patient for the surgery.

An older patient was admitted with a fractured hip after being found on the floor of her home. She was extremely malnourished and started on parenteral nutrition (PN) 3 days ago. Which assessment finding would be of most concern to the nurse? a. Blood glucose level of 125 mg/dL b. Serum phosphate level of 1.9 mg/dL c. White blood cell count of 10,500/µL d. Serum potassium level of 4.6 mEq/L

Correct answer: b Rationale: Refeeding syndrome can occur if a malnourished patient is started on aggressive nutritional support. Hypophosphatemia (serum phosphate level less than 2.4 mg/dL) is the hallmark of refeeding syndrome and could result in cardiac dysrhythmias, respiratory arrest, and neurologic problems. An increase in the blood glucose level is expected during the first few days after PN is started. The goal is to maintain a glucose range of 110 to 150 mg/dL. An elevated white blood cell count (greater than 11,000/µL) could indicate an infection. Normal serum potassium levels are between 3.5 and 5.0 mEq/L.

Which patient is at highest risk for complications of obesity? a. A 30-year-old woman who is 5 ft (151 cm) tall, weighs 140 lb (63 kg), and carries weight in her thighs. b. A 56-year-old woman with a BMI of 38 kg/m2, a waist measurement of 38 in (96 cm), and a hip measurement of 36 in (91 cm) c. A 42-year-old man with a waist measurement of 36 in (91 cm) and a hip measurement of 36 in (91 cm) who is 5 ft, 6 in (166 cm) tall and weighs 150 lb (68.2 kg) d. A 68-year-old man with a waist measurement of 38 in (96 cm) and a hip measurement of 42 in (76 cm) who is 5 ft, 11 in (179 cm) tall and weighs 200 lb (90.9 kg)

Correct answer: b Rationale: The 56-year-old woman has a body mass index (BMI) of 38 kg/m2 (obese, Class II) with a waist-to-hip ratio of 1.1 with android obesity and is more at risk (very high) than the other patients. The 30-year-old woman has the least risk with a BMI of 27.3 kg/m2 (overweight) and gynoid shape. The 42-year-old man has a BMI of 24.2 kg/m2 (normal weight) with one risk factor in the waist-to-hip ratio of 1.0 and the 68-year-old man has a BMI of 27.9 kg/m2 (overweight) with a waist-to-hip ratio of 0.9.

A 40-year-old severely obese female patient with type 2 diabetes wants to lose weight. After learning about the surgical procedures, she thinks a combination of restrictive and malabsorptive surgery would be best. Which procedure should the nurse teach her about? a. Lipectomy b. Roux-en-Y gastric bypass c. Adjustable gastric banding d. Vertical sleeve gastrectomy

Correct answer: b Rationale: The Roux-en-Y gastric bypass is a common combination of restrictive (limiting the size of the stomach) and malabsorptive (less food is absorbed) surgery. Lipectomy is used to remove unsightly flabby folds of adipose tissue. Adjustable gastric banding is the most common restrictive procedure. Vertical sleeve gastrectomy is a restrictive procedure that preserves stomach function.

The patient had a car accident and was "scared to death." The patient is now reporting constipation. What affecting the gastrointestinal (GI) tract does the nurse know could be contributing to the constipation? a. The patient is too nervous to eat or drink, so there is no stool. b. The sympathetic nervous system was activated, so the GI tract was slowed. c. The parasympathetic nervous system is now functioning to slow the GI tract. d. The circulation in the GI system has been increased, so less waste is removed.

Correct answer: b Rationale: The constipation is most likely related to the sympathetic nervous system activation from the stress related to the accident. SNS activation can decrease peristalsis. Even without oral intake for a short time, stool will be formed. The parasympathetic system stimulates peristalsis. The circulation to the GI system is decreased with stress.

As gastric contents move into the small intestine, the bowel is normally protected from the acidity of gastric contents by the a. inhibition of secretin release. b. release of bicarbonate by the pancreas. c. release of pancreatic digestive enzymes. d. release of gastrin by the duodenal mucosa.

Correct answer: b Rationale: The hormone secretin stimulates the pancreas to secrete fluid with a high concentration of bicarbonate. This alkaline secretion enters the duodenum and neutralizes acid in the chyme.

The nurse has completed initial instruction with a patient regarding a weight loss program. The nurse determines that the teaching has been effective when the patient makes which statement? a. "I plan to lose 4 lb a week until I have lost the 60-pound goal." b. "I will keep a diary of weekly weights to illustrate my weight loss." c. "I will restrict my carbohydrate intake to less than 30 g/day to maximize weight loss." d."I should not exercise more than my program requires since increased activity increases the appetite."

Correct answer: b Rationale: The patient should monitor and record weight once per week. This prevents frustration at the normal variations in daily weights and may help the patient to maintain motivation to stay on the prescribed diet. Weight loss should occur at a rate of 1 to 2 lb/week. The diet should be well balanced rather than lacking in specific components that may cause an initial weight loss but is not usually sustainable. Exercise is a necessary component of any successful weight loss program.

Which assessment should the nurse prioritize in the care of a patient who has recently begun receiving parenteral nutrition (PN)? a. Skin integrity and bowel sounds b. Electrolyte levels and daily weights c. Auscultation of the chest and tests of blood coagulability d. Peripheral vascular assessment and level of consciousness (LOC)

Correct answer: b Rationale: The use of PN necessitates frequent and thorough assessments. Key focuses of these assessments include daily weights and close monitoring of electrolyte levels. Assessments of bowel sounds, integument, peripheral vascular system, LOC, chest sounds, and blood coagulation may be variously performed, but close monitoring of fluid and electrolyte balance supersedes these in importance.

Which female patient is most likely to have metabolic syndrome? a. BP 128/78 mm Hg, triglycerides 160 mg/dL, fasting blood glucose 102 mg/dL b. BP 142/90 mm Hg, high-density lipoproteins 45 mg/dL, fasting blood glucose 130 mg/dL c. Waist circumference 36 in, triglycerides 162 mg/dL, high-density lipoproteins 55 mg/dL d. Waist circumference 32 in, high-density lipoproteins 38 mg/dL, fasting blood glucose 122 mg/dL

Correct answer: b Rationale: Three of the following five measures are needed for a woman to be diagnosed with metabolic syndrome: waist circumference >35 in, triglycerides >150 mg/dL, high-density lipoproteins <50 mg/dL, BP >130 mm Hg systolic or >85 mm Hg diastolic, fasting blood glucose >110 mg/dL. Although the other options have some abnormal measures, none has all three measures in the diagnostic ranges. The criteria for metabolic syndrome for both women and men are listed in Table 41-10.

Which statement about obesity is explained by genetics? a. Older obese patients have exacerbated changes of aging. b. Android body shape and weight gain are influenced by genetics. c. White Americans have a higher incidence of obesity than African Americans. d. Men have a harder time losing weight, as they have more muscle mass than women.

Correct answer: b Rationale: Twin studies and studies with adopted children have shown that body shape and weight gain are influenced by genetics but more research is needed. Older obese people do have exacerbated aging problems related to declines in physical function. African Americans and Hispanics have a higher incidence of obesity than whites. Women have a higher incidence of obesity and more difficulty losing weight than men because women have a higher percentage of metabolically less-active fat.

Inspection of an older patient's mouth reveals the presence of white, curd-like lesions on the patient's tongue. What is the most likely etiology for this abnormal assessment finding? a. Herpesvirus b. Candida albicans c. Vitamin deficiency d. Irritation from ill-fitting dentures

Correct answer: b Rationale: White, curd-like lesions surrounded by erythematous mucosa are associated with oral candidiasis. Herpesvirus causes benign vesicular lesions in the mouth. Vitamin deficiencies may cause a reddened, ulcerated, swollen tongue. Irritation from ill-fitting dentures will cause friable, edematous, painful, bleeding gingivae.

Before administering a bolus of intermittent tube feeding to a patient with a percutaneous endoscopic gastrostomy (PEG), the nurse aspirates 220 mL of gastric contents. How should the nurse respond? a. Return the aspirate to the stomach and recheck the volume of aspirate in an hour. b. Return the aspirate to the stomach and continue with tube feeding as planned. c. Discard the aspirate to prevent over distending the stomach when the new feeding is given. d. Notify the health care provider that the feedings have been scheduled too frequently to allow for stomach emptying.

Correct answer: b Rationale: With intermittent feedings, less than 250 mL residual does not require further action. With continuous feedings and a residual of 250 mL or more after the second residual check, a pro motility agent should be considered.

Which digestive substances are active or activated in the stomach (select all that apply)? a. Bile b. Pepsin c. Gastrin d. Maltase e. Secretin f. Amylase

Correct answer: b, c Rationale: Pepsinogen is changed to pepsin by acidity of the stomach, where it begins to break down proteins. Gastrin stimulates gastric acid secretion and motility and maintains lower esophageal sphincter tone. The stomach also secretes lipase for fat digestion.Bile is secreted by the liver and stored in the gallbladder for emulsifying fats. Maltase is secreted in the small intestine and converts maltose to glucose. Secretin is secreted y the duodenal mucosa and inhibits gastric motility and acid secretion. Amylase is secured in the small intestine and by the pancreas for carbohydrate digestion.

A patient with stage I colorectal cancer is scheduled for surgery. Patient teaching for this patient would include an explanation that a. chemotherapy will begin after the patient recovers from the surgery. b. both chemotherapy and radiation can be used as palliative treatments. c. follow-up colonoscopies will be needed to ensure that the cancer does not recur. d. a wound, ostomy, and continence nurse will visit the patient to identify an abdominal site for the ostomy.

Correct answer: c Rationale: Stage 1 colorectal cancer is treated with surgical removal of the tumor and reanastomosis, and so there is no ostomy. Chemotherapy is not recommended for stage I tumors. Follow-up colonoscopy is recommended because colorectal cancer can recur.

The patient tells the nurse she had a history of abdominal pain, so she had a surgery to make an opening into the common bile duct to remove stones. The nurse knows that this surgery is called a a. colectomy b. cholecystectomy c. choledocholithotomy d. choledochojejunostomy

Correct answer: c Rationale: A choledocholithotomy is an opening into the common bile duct for the removal of stones. A colectomy is the removal of the colon. The cholecystectomy is the removal of the gallbladder. The choledochojejunostomy is an opening between the common bile duct and the jejunum.

What is a normal finding during physical assessment of the mouth? a. A red, slick appearance of the tongue b. Uvular deviation to the side on saying "Ahh" c. A thin, white coating of the dorsum of the tongue d. Scattered red, smooth areas on the dorsum of the tongue

Correct answer: c Rationale: A thin white coating of the dorsum (top) of the tongue is normal. A red, slick appearance is characteristic of cobalamin deficiency and scattered red, smooth areas on the tongue are known as geographic tongue. The uvula should remain in the midline while the patient is saying "Ahh"

The nurse recognizes that the majority of patients' caloric needs should come from which source? a. Fats b. Proteins c. Polysaccharides d. Monosaccharides

Correct answer: c Rationale: Carbohydrates should constitute between 45% and 65% of caloric needs, compared with 20% to 35% from fats and 10% to 35% from proteins. Polysaccharides are the complex carbohydrates that are contained in breads and grains. Monosaccharides are simple sugars.

In developing an effective weight reduction plan for an overweight patient who states a willingness to try to lose weight, it is most important for the nurse to first assess which factor? a. The length of time the patient has been obese b. The patient's current level of physical activity c. The patient's social, emotional, and behavioral influences on obesity d. Anthropometric measurements, such as body mass index and skinfold thickness

Correct answer: c Rationale: Eating patterns are established early in life, and eating has many meanings for people. To establish a weight reduction plan that will be successful for the patient, the nurse should first explore the social, emotional, and behavioral influences on the patient's eating patterns. The duration of obesity, current physical activity level, and current anthropometric measurements are not as important for the weight reduction plan.

The health care team is assessing a male patient for acute pancreatitis after he presented to the emergency department with severe abdominal pain. Which laboratory value is the best diagnostic indicator of acute pancreatitis? a. Gastric pH b. Blood glucose c. Serum amylase d. Serum potassium

Correct answer: c Rationale: Elevated serum amylase levels indicate early pancreatic dysfunction and are used to diagnose acute pancreatitis. Serum lipase levels stay elevated longer than serum amylase in acute pancreatitis. Blood glucose, gastric pH, and potassium levels are not direct indicators of acute pancreatic dysfunction.

What is an indication for parenteral nutrition that is not an appropriate indication for enteral tube feedings? a. Head and neck cancer b. Hypermetabolic states c. Malabsorption syndrome d. Protein-calorie malnutrition

Correct answer: c Rationale: In malabsorption syndrome, foods that are ingested into the intestinal tract cannot be digested or absorbed and tube feedings infused into the intestinal tract would not be absorbed. All of the other conditions can be treated with enteral or parenteral nutrition, depending on the patient's needs.

The nurse is evaluating the nutritional status of a 55-year-old man who is undergoing radiation treatment for oropharyngeal cancer. Which laboratory test would be the best indicator to determine if the patient has protein-calorie malnutrition? a. Serum transferrin b. C-reactive protein c. Serum prealbumin d. Alanine transaminase (ALT)

Correct answer: c Rationale: In the absence of an inflammatory condition, the best indicator of protein-calorie malnutrition (PCM) is prealbumin; prealbumin is a protein synthesized by the liver and indicates recent or current nutritional status. Decreased albumin and transferrin levels are other indicators that protein is deficient. C-reactive protein (CRP) is elevated during inflammation and is used to determine if prealbumin, albumin, and transferrin are decreased related to protein deficiency or an inflammatory process. Other indicators of protein deficiency include elevated serum potassium levels, low red blood cell counts and hemoglobin levels, decreased total lymphocyte count, elevated liver enzyme levels (ALT), and decreased levels of both fat-soluble and water-soluble vitamins.

Which statement accurately describes vitamin deficiencies? a. The two nutrients most often lacking in the diet of a vegan are vitamin B6 and folic acid. b. Vitamin imbalances occur frequently in the United States because of excessive fat intake. c. Surgery on the GI tract may contribute to vitamin deficiencies because of impaired absorption. d. Vitamin deficiencies in adults most commonly are clinically manifested by disorders of the skin.

Correct answer: c Rationale: Patients who have surgery on the GI tract may be at risk for vitamin deficiencies because of inability to absorb or metabolize them. The strict vegan diet most often lacks cobalamin (vitamin B12) and iron. Although the high intake of fat is a major nutritional problem in the United States, vitamin deficiencies are rare in developed countries except in those with eating disorders or chronic alcohol abusers. Some vitamin deficiencies in adults have neurologic manifestations.

The nurse monitors the laboratory results of the patient with protein-calorie malnutrition during treatment. Which result is an indication of improvement in the patient's condition? a. Decreased lymphocytes b. Increased serum potassium c. Increased serum transferrin d. Decreased serum prealbumin

Correct answer: c Rationale: Serum transferrin is a protein that is synthesized by the liver and used for iron transport and decreases when there is protein deficiency. An increase in the protein would indicate a more positive nitrogen balance with amino acids available for synthesis. Decreased lymphocytes and serum prealbumin are indicators of protein depletion and increased serum potassium shows continuing failure of the sodium-potassium pump.

During care of the severely obese patient, what is most important for the nurse to do? a. Avoid reference to the patient's weight to avoid embarrassing the patient. b. Emphasize to the patient how important it is to lose weight to maintain health. c. Plan for necessary modifications in equipment and nursing techniques before initiating care. d. Recognize that a full assessment of each body system might not be possible because of numerous layers of skinfolds.

Correct answer: c Rationale: Special considerations are needed for the care of the severely obese patient because most hospital units are not prepared with beds, chairs, BP cuffs, and other equipment that will need to be used with the very obese patient. Consideration of all aspects of care should be made before implementing care for the patient, including extra time and perhaps assistance for positioning, physical assessment, and transferring the patient.

When caring for the patient with heart failure, the nurse knows that which gastrointestinal process is most dependent on cardiac output and may affect the patient's nutritional status? a. Ingestion b. Digestion c. Absorption d. Elimination

Correct answer: c Rationale: Substances that interface with the absorptive surfaces of the GI tract (primarily in the small intestine) diffuse across the intestinal membranes into intestinal capillaries and are then carried to other parts of the body for use in energy production. The cardiac output provides the blood flow for this absorption of nutrients to occur.

How will an obstruction at the ampulla of Vater affect the digestion of all nutrients? a. Bile is responsible for emulsification of all nutrients and vitamins. b. Intestinal digestive enzymes are released through the ampulla of Vater. c. Both bile and pancreatic enzymes enter the duodenum at the ampulla of Vater. d. Gastric contents can ply pass to the duodenum when the ampulla of Vater is open.

Correct answer: c Rationale: The ampulla of Vater is the site where the pancreatic duct and common bile duct enter the duodenum and the opening and closing of the ampulla is controlled by the sphincter of Oddi. Because bile from the common bile duct is needed for emulsification of fat to promote digestion and pancreatic enzymes from the pancreas are needed for digestion of all nutrients, a blockage at this point would affect the digestion of all nutrients. Gastric contents pass into the duodenum through the pylorus or pyloric valve.

A patient's serum liver enzyme tests reveal an elevated aspartate aminotransferase (AST). The nurse recognizes what about the elevated AST? a. It eliminates infection as a cause of liver damage. b. It is diagnostic for liver inflammation and damage. c. Tissue damage in organs other than the liver may be identified. d. Nervous system symptoms related to hepatic encephalopathy may be the cause.

Correct answer: c Rationale: The aspartate aminotransferase (AST) level is elevated in liver disease but it is important to note that it is also elevated in damage to the heart and lungs and is not a specific test for liver function. Measurements of most of the transaminases involves nonspecific tests unless isoenzyme fractions are determined. Hepatic encephalopathy is related to elevated ammonia levels.

The nurse is assessing a 50-year-old woman admitted with a possible bowel obstruction. Which assessment finding would be expected in this patient? a. Tympany to abdominal percussion b. Aortic pulsation visible in epigastric region c. High-pitched sounds on abdominal auscultation d. Liver border palpable 1 cm below the right costal margin

Correct answer: c Rationale: The bowel sounds are more high pitched (rushes and tinkling) when the intestines are under tension, as in intestinal obstruction. Bowel sounds may also be diminished or absent with an intestinal obstruction. Normal findings include aortic pulsations on inspection and tympany with percussion, and the liver may be palpable 1 to 2 cm along the right costal margin.

The nurse is caring for a patient admitted to the hospital for asthma who weighs 186 lb (84.5 kg). During dietary counseling, the patient asks the nurse how much protein he should ingest each day. How many grams of protein does the nurse recommend should be included in the diet based on the patient's current weight? a. 24 b. 41 c. 68 d. 93

Correct answer: c Rationale: The daily intake of protein should be between 0.8 and 1 g/kg of body weight. Thus this patient should take in between 68 and 84 g of protein per day in the diet.

A patient receives atropine, an anticholinergic drug, in preparation for surgery. The nurse expects this drug to affect the GI tract by doing what? a. Increasing gastric emptying b. Relaxing pyloric and ileocecal sphincters c. Decreasing secretions and peristaltic action d. Stimulation the nervous system of the GI tract

Correct answer: c Rationale: The parasympathetic nervous system stimulates activity of the gastrointestinal (GI) tract, increasing motility and secretions and relaxing sphincters to promote movement of contents. A drug that blocks this activity decreases secretions and peristalsis, slows gastric emptying, and contracts sphincters. The enteric nervous system of the GI tract is modulated by sympathetic and parasympathetic influence.

Which explanation about weight reduction should be included when teaching the obese patient and her obese husband? a. Weight gain is caused by psychologic factors. b. Daily weighing is recommended to monitor weight loss. c. Fat is not burned until the glycogen-water pool is depleted. d. Men lose weight less quickly than women because they have a higher percentage of metabolically less-active fat.

Correct answer: c Rationale: With reducing diets that severely restrict carbohydrates, the body's glycogen stores become depleted within a few days. The glycogen normally binds to water in fat cells and it is this water loss that causes weight loss in the first few days. Fat is not burned until the glycogen-water pool is depleted. Although psychosocial components (i.e., using food for comfort or reward and inability to buy high-nutritional quality food) may have an influence on weight gain, these factors along with lack of physical exercise, underestimation of portion size, and genetics contribute to weight gain. Weekly weighing is recommended as a more reliable indicator of weight loss because daily weighing shows frequent fluctuation from retained water (including urine) and elimination of feces. Men are able to lose weight more quickly than women because women have a higher percentage of metabolically less-active fat.

The obesity classification that is most often associated with cardiovascular health problems is a. primary obesity. b. secondary obesity. c. gynoid fat distribution. d. android fat distribution.

Correct answer: d Rationale: Individuals with fat located primarily in the abdominal area (i.e., whose body is apple-shaped) are at greater risk for obesity-related complications (e.g., heart disease) than are those whose fat is primarily located in the upper legs (i.e., whose body is pear-shaped). Individuals whose fat is distributed over the abdomen and upper body (i.e., neck, arms, and shoulders) are classified as having android obesity.

Which nursing actions are indicated for a liver biopsy (select all that apply)? a. Observe for white stools b. Monitor for rectal bleeding c. Monitor for internal bleeding d. Position to right side after test e. Ensure bowel preparation was done f. Check coagulation status before test

Correct answer: c, d, f Rationale: Because the liver is a vascular organ, vital signs are monitored to assess for internal bleeding. Prevention of bleeding is the reason for positioning on the right side for at least 2 hours and for splinting the puncture site. Again, because of the vasculature of the liver, coagulation status is checked before the biopsy is done. White stools occur with upper gastrointestinal (UGI) or barium swallow tests. No smoking is to be done after midnight before the study with an UGI. The bowel must be cleared before a lower GI or barium enema, a virtual colonoscopy, or a colonoscopy. Rectal bleeding may occur with a sigmoidoscopy or colonoscopy. A perforation may occur with an esophagogastroduodenoscopy (EGD), ERCP, or peritoneoscopy.

When teaching a patient about weight reduction diets, the nurse teaches the patient that an appropriate single serving of a food is a. a 6-inch bagel. b. 1 cup of chopped vegetables. c. a piece of cheese the size of three dice. d. a chicken breast the size of a deck of cards.

Correct answer: d Rationale: A chicken breast the size of a deck of cards is about 3 oz, a recommended portion size of meat. Other normal portions include a 3-inch bagel, 1/2 cup of chopped vegetables, and a piece of cheese the size of six dice.

Which patient is at highest risk for developing metabolic syndrome? a. A 62-year-old white man who has coronary artery disease with chronic stable angina b. A 54-year-old Hispanic woman who is sedentary and has nephrogenic diabetes insipidus c. A 27-year-old Asian American woman who has preeclampsia and gestational diabetes mellitus d. A 38-year-old Native American man who has diabetes mellitus and elevated hemoglobin A1C

Correct answer: d Rationale: African Americans, Hispanics, Native Americans, and Asians are at an increased risk for development of metabolic syndrome. Other risk factors include individuals who have diabetes that cannot maintain a normal glucose level, have hypertension, and secrete a large amount of insulin, or who have survived a heart attack and have hyperinsulinemia.

In the immediate postoperative period a nurse cares for a severely obese 72-year-old man who had surgery for repair of a lower leg fracture. Which assessment would be most important for the nurse to make? a. Cardiac rhythm b. Surgical dressing c. Postoperative pain d. Oxygen saturation

Correct answer: d Rationale: After surgery an older and/or severely obese patient should be closely monitored for oxygen desaturation. The body stores anesthetics in adipose tissue, placing patients with excess adipose tissue (e.g., obesity, older) at risk for resedation. As adipose cells release anesthetic back into the bloodstream, the patient may become sedated after surgery. This may depress the respiratory rate and result in a drop in oxygen saturation.

To evaluate the effect of nutritional interventions for a patient with protein-calorie malnutrition, what is the best indicator for the nurse to use? a. Height and weight b. Body mass index (BMI) c. Weight in relation to ideal body weight d. Mid-upper arm circumference and triceps skinfold

Correct answer: d Rationale: Anthropometric measurements, including mid-upper arm circumference and triceps skinfold measurements, are good indicators of lean body mass and skeletal protein reserves and are valuable in evaluating persons who may have been or are being treated for acute protein malnutrition. The other measurements do not specifically address muscle mass.

At the first visit to the clinic, the female patient with a BMI of 29 kg/m2 tells the nurse that she does not want to become obese. Which question used for assessing weight issues is the most important question for the nurse to ask? a. "What factors contributed to your current body weight?" b. "How is your overall health affected by your body weight?" c. "What is your history of gaining weight and losing weight?" d. "In what ways are you interested in managing your weight differently?"

Correct answer: d Rationale: Asking the patient about her desire to manage her weight in a different manner helps the nurse determine the patient's readiness for learning, degree of motivation, and willingness to change lifestyle habits. The nurse can help the patient set realistic goals. This question will also lead to discussing the patient's history of gaining and losing weight and factors that have contributed to the patient's current weight. The patient may be unaware of the overall health effects of her body weight, so this question is not helpful at this time.

After eating, a patient with an inflamed gallbladder experiences pain caused by contraction of the gallbladder. What is the mechanism responsible for this action? a. Production of bile by the liver b. Production of secretin by the duodenum c. Release of gastrin from the stomach antrum d. Production of cholecystokinin by the duodenum

Correct answer: d Rationale: Cholecystokinin is secreted by the duodenal mucosa when fats and amino acids enter the duodenum and stimulate the gallbladder to release bile to emulsify the fats for digestion. The bile is produced by the liver but stored in the gallbladder. Secretin is responsible for stimulating pancreatic bicarbonate secretion and gastrin increases gastric motility and acid secretion.

A severely obese patient has undergone Roux-en-Y gastric bypass surgery. In planning postoperative care, the nurse anticipates that the patient a. may have severe diarrhea early in the postoperative period. b. will not be allowed to ambulate for 1 to 2 days postoperatively. c. will require nasogastric suction until the incision heals. d. may have only liquids orally, and in very limited amounts, during the postoperative period.

Correct answer: d Rationale: During the immediate postoperative period, water and sugar-free clear liquids are given (30 mL every 2 hours while the patient is awake).

What is the most common cause of secondary protein-calorie malnutrition in the United States? a. The unavailability of foods high in protein b. A lack of knowledge about nutritional needs c. A lack of money to purchase high-protien foods d. An alteration in ingestion, digestion, absorption, or metabolism

Correct answer: d Rationale: In the United States, where rote in intake is high and of good quality, protein-calorie malnutrition most commonly results from problems of the GI system. In developing countries, adequate food sources might not exist, the inhabitants may not be well educated about nutritional needs, and economic conditions can prevent purchase of balanced diets.

The nurse has completed initial instruction with a patient regarding a weight-loss program. Which patient comment indicates to the nurse that the teaching has been effective? a. "I will keep a diary of daily weight to illustrate my weight loss." b. "I plan to lose 4 lb a week until I have lost the 60 lb I want to lose." c. "I should not exercise more than what is required so I don't increase my appetite." d. "I plan to join a behavior modification group to help establish long-term behavior changes."

Correct answer: d Rationale: People who have undergone behavior therapy are more successful in maintaining weight losses over time because most programs deemphasize the diet, focus on how and when the person eats and education, and provide support from others. Weighing daily is not recommended and plateaus may not allow for consistent weight loss. A goal for weight loss must be set and 1 to 2 pounds a week is realistic. A more rapid loss often causes skin and underlying tissue to lose elasticity and become flabby folds of tissue. Exercising more often depresses appetite and exercise need not be limited.

A patient is admitted to the hospital with a diagnosis of diarrhea with dehydration. The nurse recognizes that increased peristalsis resulting in diarrhea can be related to a. sympathetic inhibition. b. mixing and propulsion. c. sympathetic stimulation. d. parasympathetic stimulation.

Correct answer: d Rationale: Peristalsis is increased by parasympathetic stimulation.

The nurse is reviewing the laboratory test results for a 71-year-old patient with metastatic lung cancer. The patient was admitted with a diagnosis of malnutrition. The serum albumin level is 4.0 g/dL, and prealbumin is 10 mg/dL. What should this indicate to the nurse? a. The albumin level is normal, and therefore the patient does not have protein malnutrition. b. The albumin level is increased, which is a common finding in patients with cancer who have malnutrition. c. Both the serum albumin and prealbumin levels are reduced, consistent with the admitting diagnosis of malnutrition. d. Although the serum albumin level is normal, the prealbumin level more accurately reflects the patient's nutritional status.

Correct answer: d Rationale: Prealbumin has a half-life of 2 days and is a better indicator of recent or current nutritional status. Serum albumin has a half-life of approximately 20 to 22 days. The serum level may lag behind actual protein changes by more than 2 weeks and is therefore not a good indicator of acute changes in nutritional status.

A patient with anorexia nervosa shows signs of malnutrition. During initial referring, the nurse carefully assesses the patient for a. hyperkalemia. b. hypoglycemia. c. hypercalcemia. d. hypophosphatemia.

Correct answer: d Rationale: Refeeding syndrome is characterized by fluid retention, electrolyte imbalances (e.g., hypophosphatemia, hypokalemia, hypomagnesemia), and hyperglycemia. Conditions that predispose patients to refeeding syndrome include long-standing malnutrition states such as those induced by chronic alcoholism, vomiting and diarrhea, chemotherapy, and major surgery. Refeeding syndrome can occur any time a malnourished patient is started on aggressive nutritional support. Hypophosphatemia is the hallmark of refeeding syndrome, and it is associated with serious outcomes, including cardiac dysrhythmias, respiratory arrest, and neurologic disturbances (e.g., paresthesias).

The best nutritional therapy plan for a person who is obese is a. the Zone diet. b. the Atkins diet. c. Sugar Busters. d. foods from the basic food groups.

Correct answer: d Rationale: Restricted food intake is a cornerstone for any weight loss or maintenance program. A good weight loss plan should include foods from the basic food groups.

Which patient has the highest morbidity risk? a. Male 6 ft. 1 in. tall, BMI 29 kg/m2 b. Female 5 ft. 6 in. tall, weight 150 lb. c. Male with waist circumference 46 in. d. Female 5 ft. 10 in. tall, obesity Class III

Correct answer: d Rationale: The patient in Class III obesity has the highest risk for disease because Class III denotes severe obesity or a BMI greater than 40 kg/m2. The patient with waist circumference 46 in. has a high risk for disease, but without the BMI or obesity class, a more precise determination cannot be made. The female who is 5 ft. 6 in. tall has a normal weight for her height. The male patient who is over 6 ft. tall is overweight, which increases his risk of disease, but a more precise determination cannot be made without the waist circumference.

What is a postoperative nursing intervention for the obese patient who has undergone bariatric surgery? a. Irrigating and repositioning the nasogastric (NG) tube as needed b. Delaying ambulation until the patient has enough strength to support self c. Keeping the patient positioned on the side to facilitate respiratory function d. Providing adequate support to the incision during coughing, deep breathing, and turning

Correct answer: d Rationale: Turning, coughing, and deep breathing are essential to prevent postoperative complications. Protecting the incision from strain is important since wound dehiscence is a problem for obese patients. If a nasogastric (NG) tube that is present following gastric surgery for severe obesity becomes blocked or needs repositioning, the health care provider should be notified. Ambulation is usually started on the evening of surgery and addition help will be needed to support the patient. Respiratory function is promoted by keeping the head of the bed elevated at an angle of 35 to 40 degrees.

A 68-year-old patient is in the office for a physical. She notes that she no longer has regular bowel movements. Which suggestion by the nurse would be most helpful to the patient? a. Take an additional laxative to stimulate defecation. b. Eat less acidic foods to enable the gastrointestinal system to increase peristalsis. c. Eat less food at each meal to prevent feces from backing up related to slowed peristalsis. d. Attempt defecation after breakfast because gastrocolic reflexes increase colon peristalsis at that time.

Correct answer: d Rationale: When food inters the stomach and duodenum, the gastrocolic and duodenocolic reflexes are initiated and are more active after the first daily meal. Additional laxatives or laxative abuse contribute to constipation in older adults. Decreasing food intake is not recommended, as many older adults have a decreased appetite. Fibre and fluids should be increased.

The ED nurse has inspected, auscultated, and palpated the abdomen with no obvious abnormalities, except pain. When the nurse palpates the abdomen for rebound tenderness, there is severe pain. The nurse should know that this could indicate what problem? a. Hepatic cirrhosis b. Hypersplenomegaly c. Gall bladder distention d. Peritoneal inflammation

Correct answer: d Rationale: When palpating for rebound tenderness, the problem area of the abdomen will produce pain and severe muscle spasm when there is peritoneal inflammation. Hepatic cirrhosis, hypersplenomegaly, and gall bladder distention do not manifest with rebound tenderness.

Wich of the following criteria must be met for a diagnosis of metabolic syndrome (select all that apply)? a. Hypertension b. Elevated triglycerides c. Elevated plasma glucose d. Increased waist circumference e. Decreased low-density lipoproteins

Correct answers: a, b, c, d Rationale: Three of the following five criteria must be met for a diagnosis of metabolic syndrome: • Waist circumference of 40 inches or more in men and 35 inches or more in women • Triglyceride levels higher than 150 mg/dL, or need for drug treatment for elevated triglyceride levels • High-density lipoprotein (HDL) cholesterol levels lower than 40 mg/dL in men and lower than 50 mg/dL in women, or need for drug treatment for reduced HDL cholesterol levels • Blood pressure: 130 mm Hg or higher systolic or 85 mm Hg or higher diastolic, or need for drug treatment for hypertension • Fasting blood glucose level of 110 mg/dL or higher, or need for drug treatment for elevated glucose levels

When a 35-year-old female patient is admitted to the emergency department with acute abdominal pain, which possible diagnosis should you consider that may be the cause of her pain (select all that apply)? a. Gastroenteritis b. Ectopic pregnancy c. Gastrointestinal bleeding d. Irritable bowel syndrome e. Inflammatory bowel disease

Correct answers: a, b, c, d, e Rationale: All these conditions could cause acute abdominal pain.

Normally, which hormones and peptides affect appetite (select all that apply)? a. Leptin b. Insulin c. Ghrelin d. Peptide YY e. Neuropeptide Y f. Cholecystokinin

Correct answers: a, b, c, d, e, f Rationale: Normally ghrelin and neuropeptide Y stimulate appetite. Leptin suppresses appetite and hunger. Insulin decreases appetite. Peptide YY and cholecystokinin inhibit appetite by slowing gastric emptying and sending satiety signals to the hypothalamus.

A normal physical assessment finding of the GI system is/are (select all that apply) a. nonpalpable liver and spleen. b. borborygmi in upper right quadrant. c. tympany on percussion of the abdomen. d. liver edge 2 to 4 cm below the costal margin. e. finding of a firm, nodular edge on the rectal examination.

Correct answers: a, c Rationale: Normal assessment findings for the gastrointestinal system include a nonpalpable liver and spleen and generalized tympany on percussion. Normally, bowel sounds are high pitched and gurgling; loud gurgles indicate hyperperistalsis and are called borborygmi (stomach growling). If the patient has chronic obstructive pulmonary disease, large lungs, or a low-set diaphragm, the liver may be palpated 0.4 to 0.8 inch (1 to 2 cm) below the right costal margin. On palpation, the rectal wall should be soft and smooth and should have no nodules.

Health risks associated with obesity include (select all that apply) a. colorectal cancer. b. rheumatoid arthritis. c. polycystic ovary syndrome. d. nonalcoholic steatohepatitis. e. systemic lupus erythematosus.

Correct answers: a, c, d Rationale: Health risks associated with obesity include cardiovascular disease (related to increased low-density lipoprotein levels, increased triglyceride levels, and decreased high-density lipoprotein levels), hypertension, sleep apnea, obesity hypoventilation syndrome, reduced chest wall compliance, increased work of breathing, decreased total lung capacity and functional residual capacity, type 2 diabetes mellitus (i.e., hyperinsulinemia and insulin resistance), osteoarthritis, hyperuricemia, gout, gastroesophageal reflux disease, gallstones, nonalcoholic steatohepatitis, fatty liver and cirrhosis, cancer (mainly breast, endometrial, kidney, colorectal, pancreas, esophagus, and gallbladder), psychosocial problems (employment, education, and health care), low self-esteem, withdrawal from social interactions, and major depression.

Which teaching points are important when providing information to a patient with metabolic syndrome (select all that apply)? a. Stop smoking. b. Monitor weight daily. c. Increase level of activity. d. Decrease saturated fat intake. e. Reduce weight and maintain lower weight. f. Check blood glucose each morning prior to eating.

Correct answers: a, c, d, e Rationale: Patients with metabolic syndrome need to lower their risk factors by reducing and maintaining weight, increasing physical activity, establishing healthy diet habits, and smoking cessation. Some patients with metabolic syndrome are diabetic and would need to monitor glucose levels frequently. When monitoring weight reduction, it is recommended to check weight weekly, not daily.

A community health nurse is conducting an initial assessment of a new patient. Which assessments should the nurse include when screening the patient for metabolic syndrome (select all that apply)? a. Blood pressure b. Resting heart rate c. Physical endurance d. Waist circumference e. Fasting blood glucose

Correct answers: a, d, e Rationale: The diagnostic criteria for metabolic syndrome include elevated blood pressure, fasting blood glucose, waist circumference, triglycerides, and HDL cholesterol. Resting heart rate and physical endurance are not part of the diagnostic criteria.

What characteristics describe adjustable gastric banding (select all that apply)? a. 85% of the stomach is removed. b. Stomach restriction can be reversed. c. Eliminates hormones that stimulate hunger. d. Malabsorption of fat-soluble vitamins occurs. e. Inflatable band allows for modification of gastric stoma size. f. Stomach with a gastric pouch surgically anastomosed to the jejunum.

Correct answers: b, e Rationale: The adjustable gastric banding procedure is reversible and allows a change in gastric stoma size by inflation or deflation of the band around the funds of the stomach. The vertical sleeve gastrectomy removes 85% of the stomach and eliminates the hormones produced in the stomach that stimulate hunger. The biliopancreatic diversion is a maladaptive surgery that prevents absorption of nutrients, including fat-soluble vitamins. The Roux-en-Y gastric bypass reduces the stomach size with a gastric pouch anastomosed to the small intestine, so it is both restrictive and malabsorptive.

Which clinical manifestations of inflammatory bowel disease are common to both patients with ulcerative colitis (UC) and Crohn's disease (select all that apply)? a. Restricted to rectum b. Strictures are common. c. Bloody, diarrhea stools d. Cramping abdominal pain e. Lesions penetrate intestine.

Correct answers: c, d Rationale: Clinical manifestations of UC and Crohn's disease include bloody diarrhea, cramping abdominal pain, and nutritional disorders. Intestinal lesions associated with UC are usually restricted to the rectum before moving into the colon. Lesions that penetrate the intestine or cause strictures are characteristic of Crohn's disease.

A 72-year-old patient was admitted with epigastric pain due to a gastric ulcer. Which patient assessment warrants an urgent change in the nursing plan of care? a) Chest pain relieved with eating or drinking water b) Back pain 3 or 4 hours after eating a meal c) Burning epigastric pain 90 minutes after breakfast d) Rigid abdomen and vomiting following indigestion

D A rigid abdomen with vomiting in a patient who has a gastric ulcer indicates a perforation of the ulcer, especially if the manifestations of perforation appear suddenly. Midepigastric pain is relieved by eating, drinking water, or antacids with duodenal ulcers, not gastric ulcers. Back pain 3-4 hours after a meal is more likely to occur with a duodenal ulcer. Burning epigastric pain 1-2 hours after a meal is an expected manifestation of a gastric ulcer related to increased gastric secretions and does not cause an urgent change in the nursing plan of care.

Why are adenomatous polyps removed during a colonoscopy? A. They eventually cause intestinal obstruction. B. They are prone to bleeding and lead to anemia. C. They lead to familial adenomatous polyposis (FAP). D. They are closely linked to colorectal cancer.

D Rationale Adenomatous polyps are neoplastic; 85% of colorectal adenocarcinomas arise from them. Removing adenomatous polyps decreases the occurrence of colorectal cancer. All polyps are considered abnormal and should be removed. FAP is a genetic disorder that is autosomal dominant and not related to the typical appearance of adenomatous polyps. Reference: 1034

You are conducting a community education session. Which option is correct information related to diverticulosis? A. It is commonly seen in young people. B. The classic presentation is right-sided abdominal pain. C. Adequate protein can prevent its occurrence. D. It is a result of aging and decreased stool size.

D Rationale Although the exact cause is unknown, diverticulosis is thought to result from high intraluminal pressure on weakened areas of the bowel, often resulting from inadequate fiber. The decreased stool raises intraluminal pressure. Diverticular disease is common, and the incidence increases with age. In Western civilization, the classic anatomic location of diverticulum is in the sigmoid colon or left-sided abdomen. It usually is asymptomatic unless inflammation (diverticulitis) is present. Preventive measures include increasing the bulk in diet (fresh fruits and vegetables) and decreasing intake of red meats and fats. Reference: 1046

Which is the best method for evaluation and treatment of large intestine polyps? A. Sigmoidoscopy B. Barium enema C. Digital examination D. Colonoscopy

D Rationale Colonoscopy is preferred because it allows evaluation of the total colon and polyps can be immediately removed. Only polyps in the distal colon and rectum can be detected and removed during sigmoidoscopy. Polyps can be detected but not removed during barium enema and radiography. Digital examination is used for prostate evaluation but not for the diagnosis of colon polyps because it only assesses the rectal area and not the colon. Reference: 1034

You are caring for a 68-year-old patient admitted with abdominal pain, nausea, and vomiting. The patient has an abdominal mass and a bowel obstruction is suspected. You are auscultating the abdomen listening for which types of bowel sounds that are consistent with the patient's clinical picture? A. Low pitched and rumbling above the area of obstruction B. High pitched and hypoactive below the area of obstruction C. Low pitched and hyperactive below the area of obstruction D. High pitched and hyperactive above the area of obstruction

D Rationale Early in intestinal obstruction, the patient's bowel sounds are hyperactive and high pitched, sometimes referred to as "tinkling" above the level of the obstruction. This occurs because peristaltic action increases to push past the area of obstruction. As the obstruction becomes complete, bowel sounds decrease and finally become absent. Reference: 1033

Which finding is most important to monitor in a patient with Crohn's disease? A. Elevated WBC level B. Frequent diarrhea C. Abdominal cramping D. Brown discharge in urine

D Rationale Fistulas can develop between bowel and bladder. Stool in the urine and urinary tract infections (UTIs) are the signs of a fistula. The other options are expected findings during an exacerbation of the condition. Reference: 1023, 1025

During the assessment of a patient with acute abdominal pain, what should you do? A. Perform deep palpation before auscultation. B. Obtain blood pressure and pulse rate to determine hypervolemic changes. C. Auscultate bowel sounds because hyperactive bowel sounds suggest paralytic ileus. D. Measure body temperature because an elevated temperature may indicate an inflammatory or infectious process.

D Rationale For the patient complaining of acute abdominal pain, you should take vital signs immediately. Increased pulse and decreasing blood pressure are indicative of hypovolemia. An elevated temperature suggests an inflammatory or infectious process. Intake and output measurements provide essential information about the adequacy of vascular volume. Inspect the abdomen first, and then auscultate bowel sounds. Palpation is performed next and should be gentle. Reference: 1016

Which discharge teaching should you provide to a patient with a herniorrhaphy for an inguinal hernia repair? A. Wear a truss continually. B. Call the primary provider if scrotal edema occurs. C. Cough frequently to prevent atelectasis. D. Do not do heavy lifting for 6 to 8 weeks.

D Rationale Heavy lifting should be avoided for 6 to 8 weeks after repair. Preoperatively a truss is used by some people to prevent a hernia from protruding. It is not worn after surgery. Scrotal edema is a painful but not serious complication after an inguinal hernia repair. Coughing is not encouraged because it increases pressure. The patient should be encouraged to do deep breathing and turning instead. Reference: 1048

The elderly male patient reports his stool is very narrow and thin like a pencil. What action should you take? A. Encourage more bulk in the diet. B. Stress the importance of adequate fluids. C. Assess for laxative abuse. D. Initiate evaluation for colorectal cancer.

D Rationale Left-sided colon cancerous lesions can change the stool caliber. This is the priority action. Reference: 1036

What is the most common symptom of an acute abdominal problem? A. Nausea B. Flatulence C. Fever D. Pain

D Rationale Pain is the most common symptoms of an acute abdominal problem. The other symptoms may be present, along with vomiting, diarrhea, constipation, flatulence, fatigue, or increased abdominal girth, but pain is most common. Reference: 1015

Which items are included in the Rome III symptom-based criteria for inflammatory bowel syndrome (IBS)? A. Positive occult blood stool specimen B. Unilateral abdominal pain C. Diarrhea 10 or more times/day D. Abdominal pain for at least 3 months

D Rationale The Rome III criteria include abdominal discomfort or pain for at least 3 months, with onset at least 6 months before that has at least two characteristics: (1) relieved with defecation, (2) onset associated with a change in stool frequency, and (3) onset associated with change in stool appearance. Positive occult blood in the stool can be an indication of colorectal cancer and that must be ruled out. Pain tends to be across the lower abdomen. While some people can have frequent diarrhea, others have constipation or intermittent diarrhea and constipation. Reference: 1018

Which is the best indication that the patient is adjusting emotionally to having a colostomy? A. Indicates his spouse will be taking care of things B. Agrees to attend a future support group meeting C. Reads a brochure about colostomy care D. Participates in changing the drainage bag

D Rationale The best indication that a patient is adjusting emotionally is involvement in his or her care. Participation is a better indication than relying on others, promises of future activity, or only reading about the care. Reference: 1042

Which alterations occur in IBS? A. Ulceration in the gastric mucosa B. Viral infection C. Protozoal infestation D. Altered bowel motility

D Rationale The cause of IBS is unknown, but altered bowel motility, heightened visceral sensitivity, inflammation, and psychological distress are likely to be involved. Ulceration is associated with inflammatory bowel disease (IBD). IBS is not associated with infection or protozoal infestation. Reference: 1018

After inserting a nasogastric tube for a 68-year-old patient with suspected bowel obstruction, you should write which priority nursing diagnosis on the patient's problem list? A. Anxiety related to nasogastric tube placement B. Abdominal pain related to nasogastric tube placement C. Risk of deficient knowledge related to nasogastric tube placement D. Altered oral mucous membrane related to nasogastric tube placement

D Rationale With nasogastric tube placement, the patient is likely to breathe through the mouth and may experience irritation in the affected nares. For this reason, you should plan preventive measures based on this nursing diagnosis. Reference: 1034

The health care provider orders lactulose for a patient with hepatic encephalopathy. The nurse will monitor for effectiveness of this medication for this patient by assessing which of the following? A. Relief of constipation B. Relief of abdominal pain C. Decreased liver enzymes D. Decreased ammonia levels

D) Hepatic encephalopathy is a complication of liver disease and is associated with elevated serum ammonia levels. Lactulose traps ammonia in the intestinal tract. Its laxative effect then expels the ammonia from the colon, resulting in decreased serum ammonia levels and correction of hepatic encephalopathy.

The nurse is caring for a 55-year-old man patient with acute pancreatitis resulting from gallstones. Which clinical manifestation would the nurse expect the patient to exhibit? Hematochezia Left upper abdominal pain Ascites and peripheral edema Temperature over 102o F (38.9o C)

Left upper abdominal pain Abdominal pain (usually in the left upper quadrant) is the predominant manifestation of acute pancreatitis. Other manifestations of acute pancreatitis include nausea and vomiting, low-grade fever, leukocytosis, hypotension, tachycardia, and jaundice. Abdominal tenderness with muscle guarding is common. Bowel sounds may be decreased or absent. Ileus may occur and causes marked abdominal distention. Areas of cyanosis or greenish to yellow-brown discoloration of the abdominal wall may occur. Other areas of ecchymoses are the flanks (Grey Turner's spots or sign, a bluish flank discoloration) and the periumbilical area (Cullen's sign, a bluish periumbilical discoloration).

Nursing management of the patient with acute pancreatitis includes (select all that apply) a. checking for signs of hypocalcemia. b. providing a diet low in carbohydrates. c. giving insulin based on a sliding scale. d. observing stools for signs of steatorrhea. e. monitoring for infection, particularly respiratory tract infection. (Lewis 1042)

a, e Rationale: During the acute phase, it is important to monitor vital signs. Hemodynamic stability may be compromised by hypotension, fever, and tachypnea. Intravenous fluids are ordered, and the response to therapy is monitored. Fluid and electrolyte balances are closely monitored. Frequent vomiting, along with gastric suction, may result in decreased levels of chloride, sodium, and potassium. Because hypocalcemia can occur in acute pancreatitis, the nurse should observe for symptoms of tetany, such as jerking, irritability, and muscular twitching. Numbness or tingling around the lips and in the fingers is an early indicator of hypocalcemia. The patient should be assessed for Chvostek's sign or Trousseau's sign. A patient with acute pancreatitis should be observed for fever and other manifestations of infection. Respiratory infections are common because the retroperitoneal fluid raises the diaphragm, which causes the patient to take shallow, guarded abdominal breaths.

When evaluating the patient's understanding about the care of the ileostomy, what statement by the patient indicates the patient needs more teaching? a. "I will be able to regulate when I have stools." b. "I will be able to wear the pouch until it leaks." c. "Dried fruit and popcorn must be chewed very well." d. "The drainage from my stoma can damage my skin."

a. "I will be able to regulate when I have stools." The ileostomy is in the ileum and drains liquid stool frequently, unlike the colostomy which has more formed stool the further distal the ostomy is in the colon. The ileostomy pouch is usually worn 4-7 days or until it leaks. It must be changed immediately if it leaks because the drainage is very irritating to the skin. To avoid obstruction, popcorn, dried fruit, coconut, mushrooms, olives, stringy vegetables, food with skin, and meats with casings must be chewed extremely well before swallowing because of the narrow diameter of the ileostomy lumen.

The nurse plans teaching for the patient with a colostomy but the patient refuses to look at the nurse or the stoma, stating, "I just can't see myself with this thing." What is the best nursing intervention for this patient? a. Encourage the patient to share concerns and ask questions. b. Refer the patient to a chaplain to help cope with this situation. c. Explain that there is nothing the patient can do about it and must take care of it. d. Tell the patient that learning about it will prevent stool leaking and the sounds of flatus.

a. Encouraging the patient to share concerns and ask questions will help the patient to begin to adapt to living with the colostomy. The other options do not support the patient and do not portray the nurse's focus on helping the patient or treating the patient as an individual.

Which method is preferred for immediate treatment of an acute episode of constipation? a. An enema c. Stool softeners b. Increased fluid d. Bulk-forming medication

a. Enemas are fast acting and beneficial in the immediate treatment of acute constipation but should be limited in their use. Bulk-forming medication stimulates peristalsis but takes 24 hours to act. Stool softeners have a prolonged action, taking up to 72 hours for an effect, and fluids can help to decrease the incidence of constipation

A patient with ulcerative colitis has a total proctocolectomy with formation of a terminal ileum stoma. What is the most important nursing intervention for this patient postoperatively? a. Measure the ileostomy output to determine the status of the patient's fluid balance. b. Change the ileostomy appliance every 3 to 4 hours to prevent leakage of drainage onto the skin. c. Emphasize that the ostomy is temporary and the ileum will be reconnected when the large bowel heals. d. Teach the patient about the high-fiber, low-carbohydrate diet required to maintain normal ileostomy drainage.

a. Initial output from a newly formed ileostomy may be as high as 1500 to 2000 mL daily and intake and output must be accurately monitored for fluid and electrolyte imbalance. Ileostomy bags may need to be emptied every 3 to 4 hours but the appliance should not be changed for several days unless there is leakage onto the skin. A terminal ileum stoma is permanent and the entire colon has been removed. A return to a normal, presurgical diet is the goal for the patient with an ileostomy, with restrictions based only on the patient's individual tolerances.

A patient with a gunshot wound to the abdomen complains of increasing abdominal pain several hours after surgery to repair the bowel. What action should the nurse take first? a. Take the patient's vital signs. b. Notify the health care provider. c. Position the patient with the knees flexed. d. Determine the patient's IV intake since the end of surgery.

a. It is likely that the patient could be developing a peritonitis, which could be life-threatening, and assessment of vital signs for hypovolemic shock should be done to report to the health care provider. If an IV line is not in place, it should be inserted and pain may be eased by flexing the knees.

A patient with inflammatory bowel disease has a nursing diagnosis of imbalanced nutrition: less than body requirements related to decreased nutritional intake and decreased intestinal absorption. Which assessment data support this nursing diagnosis? a. Pallor and hair loss c. Anorectal excoriation and pain b. Frequent diarrhea stools d. Hypotension and urine output below 30 mL/hr

a. Signs of malnutrition include pallor from anemia, hair loss, bleeding, cracked gingivae, and muscle weakness, which support a nursing diagnosis that identifies impaired nutrition. Diarrhea may contribute to malnutrition but is not a defining characteristic. Anorectal excoriation and pain relate to problems with skin integrity. Hypotension relates to problems with fluid deficit.

A postoperative patient has a nursing diagnosis of pain related to effects of medication and decreased GI motility as evidenced by abdominal pain and distention and inability to pass flatus. Which nursing intervention is most appropriate for this patient? a. Ambulate the patient more frequently. b. Assess the abdomen for bowel sounds. c. Place the patient in high Fowler's position. d. Withhold opioids because they decrease bowel motility.

a. The abdominal pain and distention that occur from the decreased motility of the bowel should be treated with increased ambulation and frequent position changes to increase peristalsis. If the pain is severe, cholinergic drugs, rectal tubes, or application of heat to the abdomen may be prescribed. Assessment of bowel sounds is not an intervention to relieve the pain and a high Fowler's position is not indicated. Opioids may still be necessary for pain control and motility can be increased by other means.

82-year-old man is admitted with an acute attack of diverticulitis. What should the nurse include in his care? a. Monitor for signs of peritonitis. b. Treat with daily medicated enemas. c. Prepare for surgery to resect the involved colon. d. Provide a heating pad to apply to the left lower quadrant.

a. The inflammation and infection of diverticula cause small perforations with spread of the inflammation to the surrounding area in the intestines. Abscesses may form or complete perforation with peritonitis may occur. Systemic antibiotic therapy is often used but medicated enemas would increase intestinal motility and increase the possibility of perforation, as would the application of heat. Surgery is only necessary to drain abscesses or to resect an obstructing inflammatory mass.

The wound, ostomy, and continence (WOC) nurse selects the site where the ostomy will be placed. What should be included in the consideration for the site? a. The patient must be able to see the site. b. Outside the rectus muscle area is the best site. c. It is easier to seal the drainage bag to a protruding area. d. The ostomy will need irrigation, so area should not be tender.

a. The patient must be able to see the site. In selection of the ostomy site, the WOC nurse will want a site visible to the patient so the patient can take care of it, within the rectus muscle to avoid hernias, and on a flat surface to more easily create a good seal with the drainage bag.

A patient is admitted to the emergency department with acute abdominal pain. What nursing intervention should the nurse implement first? a. Measurement of vital signs b. Administration of prescribed analgesics c. Assessment of the onset, location, intensity, duration, and character of the pain d. Physical assessment of the abdomen for distention, bowel sounds, and pigmentation changes

a. The patient with an acute abdomen may have significant fluid or blood loss into the abdomen and evaluation of blood pressure (BP) and heart rate (HR) should be the first intervention, followed by assessment of the abdomen and the nature of the pain. Analgesics should be used cautiously until a diagnosis can be determined so that symptoms are not masked.

The patient has not voided for 6 hours after a herniorrhaphy. What action should you take? A. Document the findings. B. Have patient sit on toilet and listen to running water. C. Obtain a residual urine sample. D. Apply pressure on the suprapubic region to promote urge.

b Rationale After a hernia repair, the patient may have difficulty voiding. Voiding is expected by 6 hours after the operation. Measures to help the patient void include using the normal sitting position, pouring warm water over the perineum, and listening to running water. Because it has been 6 hours, action beyond documentation is required. The residual urine is the amount remaining after the patient has voided. The bladder can be palpated for distention, but the first nursing action for someone who does not have an atonic bladder is not manual pressure. Reference: 1048

Which is correct information about an anal fissure? A. Classic symptom is black, tarry stool. B. Defecating is described as "passing broken glass." C. Avoid bulk to present large stool that irritates the fissure. D. Typical treatment includes packing the rectum to absorb drainage.

b Rationale An anal fissure is a crack in the lining of the anal wall, producing the key symptoms of bright red rectal bleeding and severe anal pain on defecation. Black, tarry stool indicates bleeding higher in the gastrointestinal tract. Treatment of an anal fissure includes fiber supplements, adequate fluid intake, sitz baths, and topical analgesics. Rectal packing is used after a hemorrhoidectomy. Reference: 1053

Which behavior can help people with lactase deficiency to tolerate milk? A. Drink the milk on an empty stomach 30 minutes before meals. B. Add Lactaid to the consumed milk. C. Consume the milk with calcium supplement. D. Use on buttermilk.

b Rationale The lactase enzyme (Lactaid) is available as an over-the-counter product to mix with milk and break down the lactose before the milk is ingested. Some can tolerate the lactose better if lactase is taken with meals. Although lack of milk consumption can lead to a calcium deficiency, consuming it with calcium will not make a difference. The symptoms result from an inability to digest lactose, not a calcium deficiency. Buttermilk has additional fat, but it also has lactose. Reference: 1051

Teaching in relation to home management after a laparoscopic cholecystectomy should include a. keeping the bandages on the puncture sites for 48 hours. b. reporting any bile-colored drainage or pus from any incision. c. using over-the-counter antiemetics if nausea and vomiting occur. d. emptying and measuring the contents of the bile bag from the T tube every day. (Lewis 1042)

b Rationale: The following discharge instructions are taught to the patient and caregiver after a laparoscopic cholecystectomy: First, remove the bandages on the puncture site the day after surgery and shower. Second, notify the surgeon if any of the following signs and symptoms occur: redness, swelling, bile-colored drainage or pus from any incision; and severe abdominal pain, nausea, vomiting, fever, or chills. Third, gradually resume normal activities. Fourth, return to work within 1 week of surgery. Fifth, resume a usual diet, but a low-fat diet is usually better tolerated for several weeks after surgery.

When considering the following causes of acute abdomen, the nurse should know that surgery would be indicated for (select all that apply)? a. pancreatitis c. foreign-body perforation b. acute ischemic bowel d. pelvic inflammatory disease e. ruptured ectopic pregnancy f. ruptured abdominal aneurysm

b, c, e, f. An immediate surgical consult is needed for acute ischemic bowel, foreign-body perforation, ruptured ectopic pregnancy, or ruptured abdominal aneurysm. A diagnostic laparoscopy may be done or a laparotomy may be done to repair a ruptured abdominal aneurysm or remove the appendix. Surgery is not needed for pancreatitis or pelvic inflammatory disease, as these can be diagnosed and treated without surgery.

On examining a patient 8 hours after having surgery to create a colostomy, what should the nurse expect to find? a. Hyperactive, high-pitched bowel sounds b. A brick-red, puffy stoma that oozes blood c. A purplish stoma, shiny and moist with mucus d. A small amount of liquid fecal drainage from the stoma

b. A normal new colostomy stoma should appear bright red, have mild to moderate edema, and have a small amount of bleeding or oozing of blood when touched. A purplish stoma indicates inadequate blood supply and should be reported. The colostomy will not have any fecal drainage for 2 to 4 days but there may be some earlier mucus or serosanguineous drainage. Bowel sounds after extensive bowel surgery will be diminished or absent

The patient asks the nurse to explain what the physician meant when he said the patient had an anorectal abscess. Which description should the nurse use to explain this to the patient? a. Ulcer in anal wall c. Sacrococcygeal hairy tract b. Collection of perianal pus d. Tunnel leading from the anus or rectum

b. An anorectal abscess is a collection of perianal pus. An ulcer in the anal wall is an anal fissure. Sacrococcygeal hairy tract describes a pilonidal sinus. A tunnel leading from the anus or rectum is an anorectal fistula.

What information should be included when the nurse teaches a patient about colostomy irrigation? a. Infuse 1500 to 2000 mL of warm tap water as irrigation fluid. b. Allow 30 to 45 minutes for the solution and feces to be expelled. c. Insert a firm plastic catheter 3 to 4 inches into the stoma opening. d. Hang the irrigation bag on a hook about 36 inches above the stoma.

b. Following infusion of the fluid into the stoma, the solution and feces will take about 30 to 45 minutes to return and the patient can plan to read or perform other quiet activities during the wait time. Between 500 and 1000 mL of warm tap water should be used. A cone tip on the end of the tubing prevents bowel damage that could occur if a stiff plastic catheter is used. Fluid should be elevated about 18 to 24 inches above the stoma, or to about shoulder level, to prevent too rapid infusion of the solution and cramping.

The nurse is planning care for a 68-year-old patient with an abdominal mass and suspected bowel obstruction. Which factor in the patient's history increases the patient's risk for colorectal cancer? a. Osteoarthritis b. History of colorectal polyps c. History of lactose intolerance d. Use of herbs as dietary supplements

b. History of colorectal polyps A history of colorectal polyps places this patient at risk for colorectal cancer. This tissue can degenerate over time and become malignant. Osteoarthritis, lactose intolerance, and the use of herbs do not pose additional risk to the patient.

Which of the following would be the highest priority information to include in preoperative teaching for a 68-year-old patient scheduled for a colectomy? a. How to care for the wound b. How to deep breathe and cough c. The location and care of drains after surgery d. What medications will be used during surgery

b. How to deep breathe and cough Because anesthesia, an abdominal incision, and pain can impair the patient's respiratory status in the postoperative period, it is of high priority to teach the patient to cough and deep breathe. Otherwise, the patient could develop atelectasis and pneumonia, which would delay early recovery from surgery and hospital discharge. Care for the wound and location and care of the drains will be briefly discussed preoperatively, but done again with higher priority after surgery. Knowing which drugs will be used during surgery may not be meaningful to the patient and should be reviewed with the patient by the anesthesiologist.

What is a nursing intervention that is indicated for a male patient following an inguinal herniorrhaphy? a. Applying heat to the inguinal area c. Applying a truss to support the operative site b. Elevating the scrotum with a scrotal support d. Encouraging the patient to cough and deep breathe

b. Scrotal edema is a common and painful complication after an inguinal hernia repair and can be relieved in part by application of ice and elevation of the scrotum with a scrotal support. Heat would increase the edema and the discomfort and a truss is used to keep unrepaired hernias from protruding. Coughing is discouraged postoperatively because it increases intraabdominal pressure and stress on the repair site.

Which patient is most likely to be diagnosed with short bowel syndrome? a. History of ulcerative colitis c. Diagnosed with irritable bowel syndrome b. Had extensive resection of the ileum d. Had colectomy performed for cancer of the bowel

b. Short bowel syndrome results from extensive resection of portions of the small bowel and would occur if a patient had an extensive resection of the ileum. The other conditions primarily affect the large colon and result in fewer and less severe symptoms.

A patient with ulcerative colitis undergoes the first phase of a total proctocolectomy with ileal pouch and anal anastomosis. On postoperative assessment of the patient, what should the nurse expect to find? a. A rectal tube set to low continuous suction b. A loop ileostomy with a plastic rod to hold it in place c. A colostomy stoma with an NG tube in place to provide pouch irrigations d. A permanent ileostomy stoma in the right lower quadrant of the abdomen

b. The initial procedure for a total proctocolectomy with ileal pouch and anal anastomosis includes a colectomy, rectal mucosectomy, ileal reservoir construction, ileoanal anastomosis, and a temporary ileostomy. A loop ileostomy is the most common temporary ileostomy and it may be held in place with a plastic rod for the first week. A rectal tube to suction is not indicated in any of the surgical procedures for ulcerative colitis. A colostomy is not used and an NG tube would not be used to irrigate the pouch. A permanent ileostomy stoma would be expected following a total proctocolectomy with a permanent ileostomy.

How is the most common form of malabsorption syndrome treated? a. Administration of antibiotics b. Avoidance of milk and milk products c. Supplementation with pancreatic enzymes d. Avoidance of gluten found in wheat, barley, oats, and rye

b. The most common type of malabsorption syndrome is lactose intolerance and it is managed by restricting the intake of milk and milk products. Antibiotics are used in cases of bacterial infections that cause malabsorption, pancreatic enzyme supplementation is used for pancreatic insufficiency, and restriction of gluten is necessary for control of adult celiac disease (celiac sprue, gluten-induced enteropathy).

The patient has persistent and continuous pain at McBurney's point. The nursing assessment reveals rebound tenderness and muscle guarding with the patient preferring to lie still with the right leg flexed. What should the nursing interventions for this patient include? a. Laxatives to move the constipated bowel b. NPO status in preparation for possible appendectomy c. Parenteral fluids and antibiotic therapy for 6 hours before surgery d. NG tube inserted to decompress the stomach and prevent aspiration

b. The patient's manifestations are characteristic of appendicitis. After laboratory test and CT scan confirmation, the patient will have surgery. Laxatives are not used. The 6 hours of fluids and antibiotics preoperatively would be used only if the appendix was ruptured. The NG tube is more likely to be used with abdominal trauma.

In contrast to diverticulitis, the patient with diverticulosis: a. has rectal bleeding b. often has no symptoms c. has localized cramping pain d. frequently develops peritonitis

b. often has no symptoms Rationale: Many people with diverticulosis have no symptoms. Patients with diverticulitis have symptoms of inflammation. Diverticulitis can lead to obstruction or perforation.

Which food item should a patient with celiac disease avoid? A. Steamed rice B. Yogurt C. Pancakes D. Raw pineapple

c Rationale A patient with celiac disease cannot digest wheat, rye, or barley. Pancakes are made with flour, which is made with wheat. Reference: 1049-1050

Which is a risk for developing hemorrhoids? A. Body mass index (BMI) of 17 kg/m2 B. Younger than 30 years C. Chronic constipation D. History of lactase deficiency

c Rationale Chronic constipation increases intraluminal pressure, which can predispose to hemorrhoids. Overweight is a risk, not underweight (normal BMI is 18.5 to 24.9 kg/m2). Older adults are at higher risk. Lactase deficiency is not related to hemorrhoids. Reference: 1052

The patient reports that she experiences cramping, flatulence, and abdominal cramping about an hour after consuming milk. What action should you pursue? A. Encourage patient to ensure she is not drinking milk past the expiration date. B. Assess the patient for the presence of dumping syndrome. C. Suggest a hydrogen breath test. D. Ask about recent stressors in the patient's life that could cause gastrointestinal upset.

c Rationale In lactase deficiency, the enzyme to digest lactose in milk is absent. Common symptoms include bloating, flatulence, cramping abdominal pain, and diarrhea 30 minutes to several hours after ingestion. It can be diagnosed with a lactose tolerance test, a hydrogen breath test to assess intolerance, or genetic testing using DNA. The most likely cause should be explored before assuming the food is spoiled or the reaction is a result of emotional stress. Dumping syndrome occurs when the stomach storage area has decreased from surgery and concentrated carbohydrates are consumed. Reference: 1051

Which is the best food for the patient with lactase deficiency? A. Skim milk B. Low-fat ice cream C. Live-culture yogurt D. Cheddar cheese

c Rationale Live-culture yogurt can be consumed if no milk products have been added. The other options all contain lactose. Reference: 1051

Which is the most common causative organism of an anorectal abscess? A. Giardia lamblia B. Clostridium difficile C. Escherichia coli D. The organism for tuberculosis

c Rationale The most common organisms causing perianal pus are E. coli, staphylococci, and streptococci. Reference: 1054

The patient had a colon resection and a colostomy today. You should call the primary provider after noticing which finding during the patient assessment? A. No drainage in the colostomy bag B. Moderate stoma swelling C. Dusky blue stoma D. Stoma that bleeds when touched

c Rationale The stoma should be pink or red to show adequate circulation. Drainage is not expected the first 24 hours because peristalsis has not returned. Mild to moderate swelling of the stoma is expected the first 2 to 3 weeks after surgery. The raw surgical mucosa bleeds with trauma during the first postoperative day. Reference: 1041

The patient is having an esophagoenterostomy with anastomosis of a segment of the colon to replace the resected portion. What initial postoperative care should the nurse expect when this patient returns to the nursing unit? a Turn, deep breathe, cough, and use spirometer every 4 hours. b Maintain an upright position for at least 2 hours after eating. c NG will have bloody drainage and it should not be repositioned. d Keep in a supine position to prevent movement of the anastomosis.

c The patient will have bloody drainage from the nasogastric (NG) tube for 8 to 12 hours, and it should not be repositioned or reinserted without contacting the surgeon. Turning and deep breathing will be done every 2 hours, and the spirometer will be used more often than every 4 hours. Coughing would put too much pressure in the area and should not be done. Because the patient will have the NG tube, the patient will not be eating yet. The patient should be kept in a semi-Fowler's or Fowler's position, not supine, to prevent reflux and aspiration of secretions.

The patient asks the nurse why she needs to have surgery for a femoral, strangulated hernia. What is the best explanation the nurse can give the patient? a. The surgery will relieve her constipation. b. The abnormal hernia must be replaced into the abdomen. c. The surgery is needed to allow intestinal flow and prevent necrosis. d. The hernia is because the umbilical opening did not close after birth as it should have.

c. A strangulated femoral hernia obstructs intestinal flow and blood supply, thus requiring emergency surgery. The other options are incorrect.

During a routine screening colonoscopy on a 56-year-old patient, a rectosigmoidal polyp was identified and removed. The patient asks the nurse if his risk for colon cancer is increased because of the polyp. What is the best response by the nurse? a. "It is very rare for polyps to become malignant but you should continue to have routine colonoscopies." b. "Individuals with polyps have a 100% lifetime risk of developing colorectal cancer and at an earlier age than those without polyps." c. "All polyps are abnormal and should be removed but the risk for cancer depends on the type and if malignant changes are present." d. "All polyps are premalignant and a source of most colon cancer. You will need to have a colonoscopy every 6 months to check for new polyps."

c. Although all polyps are abnormal growths, the most common type of polyp (hyperplastic) is non-neoplastic, as are inflammatory, lipomas, and juvenile polyps. However, adenomatous polyps are characterized by neoplastic changes in the epithelium and most colorectal cancers appear to arise from these polyps. Only patients with a family history of familial adenomatous polyposis (FAP) have close to a 100% lifetime risk of developing colorectal cancer.

A patient returns to the surgical unit with a nasogastric (NG) tube to low intermittent suction, IV fluids, and a Jackson-Pratt drain at the surgical site following an exploratory laparotomy and repair of a bowel perforation. Four hours after admission, the patient experiences nausea and vomiting. What is a priority nursing intervention for the patient? a. Assess the abdomen for distention and bowel sounds. b. Inspect the surgical site and drainage in the Jackson-Pratt. c. Check the amount and character of gastric drainage and the patency of the NG tube. d. Administer prescribed prochlorperazine (Compazine) to control the nausea and vomiting.

c. An adequately functioning nasogastric (NG) tube should prevent nausea and vomiting because stomach contents are continuously being removed. The first intervention in this case is to check the amount and character of the recent drainage and check the tube for patency. Decreased or absent bowel sounds are expected after a laparotomy and the Jackson-Pratt drains only fluid from the tissue of the surgical site. Antiemetics may be given if the NG tube is patent because anesthetic agents may cause nausea.

The patient is receiving the following medications. Which one is prescribed to relieve symptoms rather than treat a disease? a. Corticosteroids c. Antidiarrheal agents b. 6-Mercaptopurine d. Sulfasalazine (Azulfidine)

c. Antidiarrheal agents only relieve symptoms. Corticosteroids, 6-mercaptopurine, and sulfasalazine (Azulfidine) are used to treat and control inflammation with various diseases.

The nurse identifies a need for additional teaching when a patient with acute infectious diarrhea makes which statement? a. "I can use A&D ointment or Vaseline jelly around the anal area to protect my skin." b. "Gatorade is a good liquid to drink because it replaces the fluid and salts I have lost." c. "I may use over-the-counter Imodium or Parepectolin when I need to control the diarrhea." d. "I must wash my hands after every bowel movement to prevent spreading the diarrhea to my family."

c. Antiperistaltic agents, such as loperamide (Imodium) and paregoric, should not be used in infectious diarrhea because of the potential of prolonging exposure to the infectious agent. Demulcent agents may be used to coat and protect mucous membranes in these cases. The other options are all appropriate measures to use in cases of infectious diarrhea.

What should the nurse teach the patient with diverticulosis to do? a. Use anticholinergic drugs routinely to prevent bowel spasm. b. Have an annual colonoscopy to detect malignant changes in the lesions. c. Maintain a high-fiber diet and use bulk laxatives to increase fecal volume. d. Exclude whole grain breads and cereals from the diet to prevent irritating the bowel.

c. Formation of diverticula is common when decreased bulk of stool, combined with a more narrowed lumen in the sigmoid colon, causes high intraluminal pressures that result in saccular dilation or outpouching of the mucosa through the muscle of the intestinal wall. To prevent the high intraluminal pressure, fecal volume should be increased with use of high-fiber diets and bulk laxatives, such as psyllium (Metamucil). Anticholinergic drugs are used only during an acute episode of diverticulitis and the lesions are not premalignant.

The patient comes to the emergency department with intermittent crampy abdominal pain, nausea, projectile vomiting, and dehydration. The nurse suspects a GI obstruction. Based on the manifestations, what area of the bowel should the nurse suspect is obstructed? a. Large intestine c. Upper small intestine b. Esophageal sphincter d. Lower small intestine

c. Intermittent crampy abdominal pain, nausea, projectile vomiting, and dehydration are characteristics of mechanical upper small intestinal obstruction. With continued vomiting, metabolic alkalosis may occur. Large bowel obstruction is characterized by constipation, low-grade abdominal pain, and abdominal distention. Fecal vomiting is seen with lower small intestinal obstruction.

Following bowel resection, a patient has a nasogastric (NG) tube to suction, but complains of nausea and abdominal distention. The nurse irrigates the tube as necessary as ordered, but the irrigating fluid does not return. What should be the priority action by the nurse? a. Notify the physician. b. Auscultate for bowel sounds. c. Reposition the tube and check for placement. d. Remove the tube and replace it with a new one.

c. Reposition the tube and check for placement. The tube may be resting against the stomach wall. The first action by the nurse (since this is intestinal surgery and not gastric surgery) is to reposition the tube and check it again for placement. The physician does not need to be notified unless the tube function cannot be restored by the nurse. The patient does not have bowel sounds, which is why the NG tube is in place. The NG tube would not be removed and replaced unless it was no longer in the stomach or the obstruction of the tube could not be relieved.

A patient is diagnosed with celiac disease following a workup for iron-deficiency anemia and decreased bone density. The nurse identifies that additional teaching about disease management is needed when the patient makes which statement? a. "I should ask my close relatives to be screened for celiac disease." b. "If I do not follow the gluten-free diet, I might develop a lymphoma." c. "I don't need to restrict gluten intake because I don't have diarrhea or bowel symptoms." d. "It is going to be difficult to follow a gluten-free diet because it is found in so many foods."

c. The autoimmune process associated with celiac disease continues as long as the body is exposed to gluten, regardless of the symptoms it produces, and a lifelong gluten-free diet is necessary. The other statements regarding celiac disease are all true

In planning care for the patient with Crohn's disease, the nurse recognizes that a major difference between ulcerative colitis and Crohn's disease is that Crohn's disease: a. frequently results in toxic megacolon b. causes fever nutritional deficiencies than ulcerative colitis c. often recurs after surgery, whereas ulcerative colitis is curable with a colectomy d. is manifested by rectal bleeding and anemia more frequently than is ulcerative colitis

c. often recurs after surgery, whereas ulcerative colitis is curable with a colectomy Rationale: Ulcerative colitis affects only the colon and rectum; it can cause megacolon and rectal bleeding, but not nutrient malabsorption. Surgical removal of the colon and rectum cures it. Crohn's disease usually involves the ileum, where bile salts and vitamin cobalamin are absorbed. After surgical treatment, disease recurrence at the site is common.

A patient with oral cancer is not eating. A small-bore feeding tube was inserted and the patient started on enteral feedings. Which patient goal would indicate improvement? a Weight gain of 1 kg in 1 week b Administer tube feeding at 25 mL/hr. c Consume 50% of clear liquid tray this shift. d Monitor for tube for placement and gastrointestinal residual

correct answer a The goal for a patient with oral cancer that is not eating would be to note weight gain rather than loss. Consuming 50% of the clear liquid tray is not a realistic goal. Administering feedings, monitoring tube placement, and tolerance are interventions used to achieve the goal.

The nurse determines a patient has experienced the beneficial effects of therapy with famotidine when which symptom is relieved? Nausea Belching Epigastric pain Difficulty swallowing

correct answer c Famotidine is an H2-receptor antagonist that inhibits parietal cell output of HCl acid and minimizes damage to gastric mucosa related to hyperacidity, thus relieving epigastric pain. It is not indicated for nausea, belching, and dysphagia.

A patient was admitted with epigastric pain because of a gastric ulcer. Which patient assessment warrants an urgent change in the nursing plan of care? a Back pain 3 or 4 hours after eating a meal b Chest pain relieved with eating or drinking water c Burning epigastric pain 90 minutes after breakfast d Rigid abdomen and vomiting following indigestion

correct answer d A rigid abdomen with vomiting in a patient who has a gastric ulcer indicates a perforation of the ulcer, especially if the manifestations of perforation appear suddenly. Midepigastric pain is relieved by eating, drinking water, or antacids with duodenal ulcers, not gastric ulcers. Back pain 3 to 4 hours after a meal is more likely to occur with a duodenal ulcer. Burning epigastric pain 1 to 2 hours after a meal is an expected manifestation of a gastric ulcer related to increased gastric secretions and does not cause an urgent change in the nursing plan of care.

A patient is seeking emergency care after choking on a piece of steak. The nursing assessment reveals a history of alcoholism, cigarette smoking, and hemoptysis. Which diagnostic study is most likely to be performed on this patient? a Barium swallow b Endoscopic biopsy c Capsule endoscopy d Endoscopic ultrasonography

correct answer:b Because of this patient's history of excessive alcohol intake, smoking, and hemoptysis and the current choking episode, cancer may be present. A biopsy is necessary to make a definitive diagnosis of carcinoma, so an endoscope will be used to obtain a biopsy and observe other abnormalities as well. A barium swallow may show narrowing of the esophagus, but it is more diagnostic for achalasia. An endoscopic ultrasonography may be used to stage esophageal cancer. Capsule endoscopy can show alterations in the esophagus but is more often used for small intestine problems. A barium swallow, capsule endoscopy, and endoscopic ultrasonography cannot provide a definitive diagnosis for

The patient who is admitted with a diagnosis of diverticulitis and a history of irritable bowel disease and gastroesophageal reflux disease (GERD) has received a dose of Mylanta 30 mL PO. The nurse will determine the medication was effective when which symptom has been resolved? a Diarrhea b Heartburn c Constipation d Lower abdominal pain

correct answer:b Mylanta is an antacid that contains both aluminum and magnesium. It is indicated for the relief of gastrointestinal discomfort, such as heartburn associated with GERD. Mylanta can cause both diarrhea and constipation as a side effect. Mylanta does not affect lower abdominal pain.

A patient with cholelithiasis needs to have the gallbladder removed. Which patient assessment is a contraindication for a cholecystectomy? Low-grade fever of 100° F and dehydration Abscess in the right upper quadrant of the abdomen Activated partial thromboplastin time (aPTT) of 54 seconds

ctivated partial thromboplastin time (aPTT) of 54 seconds Multiple obstructions in the cystic and common bile duct An aPTT of 54 seconds is above normal and indicates insufficient clotting ability. If the patient had surgery, significant bleeding complications postoperatively are very likely. Fluids can be given to eliminate the dehydration; the abscess can be assessed, and the obstructions in the cystic and common bile duct would be relieved with the cholecystectomy.

What should a patient be taught after a hemorrhoidectomy? A. Do not use the Valsalva maneuver. B. Eat a low-fiber diet to rest the colon. C. Administer oil-retention enema to empty the colon. D. Use prescribed pain medication before a bowel movement.

d Rationale After a hemorrhoidectomy, the patient usually dreads the first bowel movement and often resists the urge to defecate. Pain medication may be given before the bowel movement to reduce discomfort. The patient should avoid constipation and straining. A high-fiber diet can reduce constipation. A stool softener such as docusate (Colace) is usually ordered for the first few postoperative days. If the patient does not have a bowel movement within 2 to 3 days, an oil-retention enema may be given. Reference: 1053

The person with lactase deficiency is at risk for which condition? A. Colorectal cancer B. Pancreatitis C. Osteoarthritis D. Osteoporosis

d Rationale Avoidance of milk and milk products can lead to calcium deficiency, which can lead to osteoporosis. Lactase deficiency is not linked to the other options. Reference: 1051

You are answering a patient's questions about celiac or gluten-sensitive enteropathy disease. Which option is the correct information to provide? A. Celiac is also known as tropical sprue. B. Celiac is only seen in children. C. Its symptoms mimic inflammatory bowel disease (IBD). D. It is an autoimmune disease.

d Rationale Celiac is an autoimmune disease in people who have a genetic predisposition, consume gluten, and an immune-mediated response. Celiac disease is different from tropical sprue, which is a chronic disorder acquired in tropical areas and treated with folic acid and tetracycline. Celiac disease is a relatively common disease that occurs in all ages. The symptoms mimic irritable bowel syndrome (IBS). Reference: 1049

After a fistulectomy, the patient is prescribed sitz baths. What is the main rationale for a sitz bath? A. To prevent hemorrhoids B. To promote defecation C. To relieve pressure on the area D. To provide comfort

d Rationale Sitz baths are started 1 to 2 days after surgery to provide comfort and keep the anal area clean. A sponge ring may be used to relieve pressure. Reference: 1054, 1053

The nursing management of the patient with cholecystitis associated with cholelithiasis is based on the knowledge that a. shock-wave therapy should be tried initially. b. once gallstones are removed, they tend not to recur. c. the disorder can be successfully treated with oral bile salts that dissolve gallstones. d. laparoscopic cholecystectomy is the treatment of choice in most patients who are symptomatic. (Lewis 1042)

d Rationale: Laparoscopic cholecystectomy is the treatment of choice for symptomatic cholelithiasis.

The nurse is caring for a 68-year-old patient admitted with abdominal pain, nausea, and vomiting. The patient has an abdominal mass and a bowel obstruction is suspected. The nurse auscultating the abdomen listens for which of the following types of bowel sounds that is consistent with the patient's clinical picture? a. Low pitched and rumbling above the area of obstruction b. High pitched and hypoactive below the area of obstruction c. Low pitched and hyperactive below the area of obstruction d. High pitched and hyperactive above the area of obstruction

d. High pitched and hyperactive above the area of obstruction Early in intestinal obstruction, the patient's bowel sounds are hyperactive and high-pitched, sometimes referred to as "tinkling" above the level of the obstruction. This occurs because peristaltic action increases to "push past" the area of obstruction. As the obstruction becomes complete, bowel sounds decrease and finally become absent.

A nurse is doing a nursing assessment on a patient with chronic constipation. What data obtained during the interview may be a factor contributing to the constipation? a. Taking methylcellulose (Citrucel) daily c. History of hemorrhoids and hypertension b. High dietary fiber with high fluid intake d. Suppressing the urge to defecate while at work

d. Ignoring the urge to defecate causes the muscles and mucosa in the rectal area to become insensitive to the presence of feces and drying of the stool occurs. The urge to defecate is decreased and stool becomes more difficult to expel. Taking a bulk-forming agent with fluids or highfiber diet with fluids prevent constipation. Hemorrhoids are the most common complication of chronic constipation, caused by straining to pass hardened stool. The straining may cause problems in patients with hypertension but these do not cause constipation. Other things that may cause constipation are a history of diverticulosis, which is seen in individuals with low fiber intake, small stool mass, and hard stools. Chronic laxative use and chronic dilation and loss of colonic tone may also cause chronic constipation.

The nurse is preparing to administer a dose of bisacodyl (Dulcolax). In explaining the medication to the patient, the nurse would explain that it acts in what way? a. Increases bulk in the stool b. Lubricates the intestinal tract to soften feces c. Increases fluid retention in the intestinal tract d. Increases peristalsis by stimulating nerves in the colon wall

d. Increases peristalsis by stimulating nerves in the colon wall Bisacodyl is a stimulant laxative that aids in producing a bowel movement by irritating the colon wall and stimulating enteric nerves. It is available in oral and suppository forms. Fiber and bulk forming drugs increase bulk in the stool; water and stool softeners soften feces, and saline and osmotic solutions cause fluid retention in the intestinal tract.

A male patient who has undergone an anterior-posterior repair is worried about his sexuality. What is an appropriate nursing intervention for this patient? a. Have the patient's sexual partner reassure the patient that he is still desirable. b. Reassure the patient that sexual function will return when healing is complete. c. Remind the patient that affection can be expressed in ways other than through sexual intercourse. d. Explain that physical and emotional factors can affect sexual function but not necessarily the patient's sexuality.

d. Sexual dysfunction may result from an anterior-posterior repair but the nurse should discuss with the patient that different nerve pathways affect erection, ejaculation, and orgasm and that a dysfunction of one does not mean total sexual dysfunction and also that an alteration in sexual activity does not have to alter sexuality. Simple reassurance of desirability and ignoring concerns about sexual function do not help the patient to regain positive feelings of sexuality.

In instituting a bowel training program for a patient with fecal incontinence, what should the nurse first plan to do? a. Teach the patient to use a perianal pouch. b. Insert a rectal suppository at the same time every morning. c. Place the patient on a bedpan 30 minutes before breakfast. d. Assist the patient to the bathroom at the time of the patient's normal defecation.

d. The first intervention to establish bowel regularity includes promoting bowel evacuation at a regular time each day, preferably by placing the patient on the bedpan, using a bedside commode, or walking the patient to the bathroom. To take advantage of the gastrocolic reflex, an appropriate time is 30 minutes after the first meal of the day or at the patient's usual individual time. Perianal pouches are used to protect the skin only when regularity cannot be established and evacuation suppositories are also used only if other techniques are not successful.

The patient with a new ileostomy needs discharge teaching. What should the nurse plan to include in this teaching? a. The pouch can be worn for up to 2 weeks before changing it. b. Decrease the amount of fluid intake to decrease the amount of drainage. c. The pouch can be removed when bowel movements have been regulated. d. If leakage occurs, promptly remove the pouch, clean the skin, and apply a new pouch

d. The ileostomy drainage is extremely irritating to the skin, so the skin must be cleaned and a new solid skin barrier and pouch applied as soon as a leak occurs to prevent skin damage. The pouch is usually worn for 4 to 7 days unless there is a leak. Because the initial drainage from the ileostomy is high, the fluid intake must not be decreased. The pouch must always be worn, as the liquid drainage, not formed bowel movements, is frequent.

A 22-year-old patient calls the outpatient clinic complaining of nausea and vomiting and right lower abdominal pain. What should the nurse advise the patient to do? a. Use a heating pad to relax the muscles at the site of the pain. b. Drink at least 2 quarts of juice to replace the fluid lost in vomiting. c. Take a laxative to empty the bowel before examination at the clinic. d. Have the symptoms evaluated by a health care provider right away.

d. The patient is having symptoms of an acute abdomen and should be evaluated by a health care provider immediately. The patient's age, location of pain, and other symptoms are characteristic of appendicitis. Heat application and laxatives should not be used in patients with undiagnosed abdominal pain because they may cause perforation of the appendix or other inflammations. Fluids should not be taken until vomiting is controlled, nor should they be taken in the event that surgery may be performed.

In report, the nurse learns that the patient has a transverse colostomy. What should the nurse expect when providing care for this patient? a. Semiliquid stools with increased fluid requirements b. Liquid stools in a pouch and increased fluid requirements c. Formed stools with a pouch, needing irrigation, but no fluid needs d. Semiformed stools in a pouch with the need to monitor fluid balance

d. The patient with a transverse colostomy has semiliquid to semiformed stools needing a pouch and needs to have fluid balance monitored. The ascending colostomy has semiliquid stools needing a pouch and increased fluid. The ileostomy has liquid to semiliquid stools needing a pouch and increased fluid. The sigmoid colostomy has formed stools and may or may not need a pouch but will need irrigation.

When a patient returns to the clinical unit after an abdominal-perineal resection (APR), what should the nurse expect? a. An abdominal dressing c. A temporary colostomy and drains b. An abdominal wound and drains d. A perineal wound, drains, and a stoma

d. With an abdominal perineal-resection (APR), an abdominal incision is made and the proximal sigmoid colon is brought through the abdominal wall and formed into a permanent colostomy. The patient is repositioned, a perineal incision is made, and the distal sigmoid colon, rectum, and anus are removed through the perineal incision, which may be left open, packed, and have drains.


Set pelajaran terkait

Behavioral neuroscience - research methods

View Set

CMED 116: The Back Pt. 1 (The Vertebrae)

View Set

Chapter 17 Patho taken from http://thepoint.lww.com/Book LEVEL 3 MASTERY

View Set

Chapter 23: Children and Adolescents

View Set

Bio 142 - Exam 6 (Chapter 28 - Reproductive System)

View Set